ORTHOPEDIC MCQS BANK Hand and wrist 1A

  1. A 35-year-old woman reports wrist pain after a fall onto an outstretched hand. On exam, she has focal tenderness over the wrist snuffbox. A radiograph and CT image are shown in Figures A and B. What is the proper treatment of her injury?

     

     

     

     

     

     

     

    1. Rest, ice, elevation

    2. Removable splint for comfort

    3. Thumb spica cast

    4. Open reduction, internal fixation

    5. Vascularized bone grafting

    CORRECT ANSWER: 4

    The radiograph and CT scan show a displaced scaphoid waist fracture. Optimal treatment is ORIF with screw fixation.

     

    The usual mechanism of injury to the scaphoid is axial load across a hyperextended wrist. Pain with resisted pronation, snuffbox tenderness and scaphoid tuberosity tenderness should all raise suspicion for a scaphoid fracture. AP and lateral X-rays, as well as PA view with the hand in ulnar

    deviation and an oblique 45 degree view with the hand in pronation can help to identify the fracture. Bone scan, CT and MRI can also be used to make the diagnosis if radiographs are indeterminate. ORIF is recommended for any fracture displaced more than 1mm, with a radiolunate angle greater than 15 degrees, with intrascaphoid angle greater than 35 degrees, associated with perilunate dislocation or with a proximal pole fracture. Optimal treatment is ORIF with screw fixation. For minimally displaced fractures, percutaneous or mini-open fixation allows minimal dissection and preservation of extrinsic ligaments.

     

    Incorrect Answers:

    Answer 1,2,3: Nonoperative management is not indicated in displaced scaphoid fractures

    Answer 5: Vascularized bone grafting is reserved for cases of scaphoid nonunion.

     

     

     

  2. Which of the following complications is most likely to occur following the procedure on the middle finger of a right hand shown in Figure A?

     

     

     

     

    1. Flap failure

    2. Cosmetic mismatch of the skin

    3. Median nerve motor branch injury

    4. Recipient site sensitivity

    5. Flexion contracture at the recipient proximal interphalangeal joint

    CORRECT ANSWER: 5

    The image shows a thenar flap. The digit is flexed at the PIPJ and extended at the DIPJ during the period prior to flap division, leading to PIPJ stiffness and flexion contracture.

     

    Thenar flaps can be used for coverage of digital tip injuries where there is exposed bone or extensive pulp loss. Advantages include more subcutaneous fat than a cross finger flap, good color and texture match, and primary closure of the donor site. Other disadvantages include limited flap size and donor site tenderness. Contraindications include RA, Dupuytren’s contracture and advanced age with degenerative disease as these predispose to joint stiffness.

     

    Fassler et al. reviewed fingertip injuries. The thenar flap can be used for any finger, although the small finger can be difficult to position comfortably. The flap can be as wide as 2 cm and should be 1.5 times as wide as the

    defect so as to restore the normal rounded contour to the tip. To decrease the amount of PIPJ flexion required, the MCPJ and DIPJ should be flexed as much as possible.

     

    Figure A demonstrates a thenar flap of the middle finger of the right hand. Illustration A shows a preoperative image with surgical planning marks on the right hand.

     

    Incorrect Answers:

    Answer 1: The flap is attached by a pedicle prior to flap division. After division, there is generally good flap take because of adequate neovascularization of the flap.

    Answer 2: Thenar skin is a good cosmetic match for digital pulp skin (both volar skin).

    Answer 3: Injury to the recurrent motor branch of the median nerve is distinctly uncommon with this flap.

    Answer 4: Donor site sensitivity (not recipient site) is a known complication of this flap.

     

     

     

     

     

  3. A 64-year-old diabetic female presents with sudden catching and locking of her ring finger when trying to extend it. Attempts at finger extension are painful, and she notes tenderness in her distal palm. A clinical photo is shown in Figure A. Which of the following structures are implicated in the pathogenesis of this condition?

     

     

     

     

    1. Extensor digitorum tendon

    2. Grayson's ligament

    3. Oblique retinacular ligament

    4. A1 pulley

    5. Transverse carpal ligament

    CORRECT ANSWER: 4

    The scenario listed above points to a diagnosis of trigger finger. In this condition, the disparity in size between the flexor tendon and the surrounding retinacular pulley system, most commonly at the level of the first annular (A1) pulley, results in difficulty flexing or extending the finger and the “triggering” phenomenon. Metacarpophalangeal locking should be included in the differential, where the collateral ligament or volar plate tethers on a prominent metacarpal head or osteophyte.

     

    The referenced text notes that a series of two corticosteroid injections should be given before surgery is considered for A1 pulley release. Mention is also made of the possibility of diabetics being more resistant to injections, with surgical release being a cost-effective treatment for this patient population.

     

    Illustration A shows the clinical appearance of this disorder.

     

     

     

     

     

     

  4. All of the following factors are favorable for digit survival after replantation surgery EXCEPT?

    1. Sharply amputated digit

    2. Crushed amputated digit

    3. Warm ischemia time of 8 hours

    4. Patient age of 10 years

    5. Female gender CORRECT ANSWER: 2

    Waikakul et al. in a study of 1018 replantations found that type of injury was the most important factor influencing immediate and late outcome. Extensively crushed injuries had the worst outcome, followed by degloving and avulsion

    injuries. Sharp cut injuries fared the best. Regular cigarette smoking resulted in poor immediate survival rate and prolonged ischemia had a significant influence in final functional outcome, but neither were as important as mechanism of injury. Alcohol consumption was also a negative predictor.

     

    Favorable factors for replantation survival were female gender, age under 13 years old, and nonsmokers. Regarding ischemia time, <12 hours of warm ischemia or <24 hours of cold ischemia for a digit to obtain optimal outcomes.

     

     

     

  5. A 67-year-old male has soft tissue defect on the palmar aspect of his right hand following a resection of mass as shown in Figures A and

    B. Which of the following is most appropriate for achieving coverage of the defect?

     

     

     

     

    1. Vacuum-assisted wound closure only

    2. Flap coverage with full-thickness skin coverage

    3. Wet-to-dry dressings only

    4. Split-thickness skin coverage only

    5. Flap coverage with meshed split-thickness skin coverage

    CORRECT ANSWER: 2

     

    Soft tissue defects of the palm are most appropriately treated with flap coverage followed with full-thickness grafts. A flap is a unit of tissue supported by blood vessels and moved from a donor site to a recipient site to cover a defect in tissue.

     

    This patient's full-thickness coverage was created from a posterior interosseous artery island flap as shown in Illustrations A-C. The skin of the dorsal hand is similar to that of the rest of the body and thus may be adequately replaced by split-thickness skin grafts from the skin of most donor sites. In contrast, palmar hand skin differs from that of the dorsal hand in that it (1) lacks both hair and sebaceous glands and (2) has specialized encapsulated nerve endings (Meissner’s corpuscles and Vater-Pacini corpuscles) that confer enhanced sensation via mechanoreception. Full thickness skin grafts (FTSG) transfer all of the skin appendages and nerve endings except those sweat glands located in the subcutaneous tissue and some of the Vater-Pacini corpuscles of palmar and plantar skin. It is necessary to remove all fat and subcutaneous tissue from the undersurface of a full-thickness skin graft, as this will otherwise act as a barrier preventing vascularization and graft survival.

     

     

     

     

     

     

     

     

  6. Vitamin B12 deficiency is a known cause of which the following?

    1. Inability to whistle

    2. Peripheral sensory neuropathy

    3. Increased deep tendon reflexes

    4. Urinary retention

    5. Hydrophobia CORRECT ANSWER: 2

    Vitamin B12 deficiency is a known cause of peripheral sensory neuropathy and B12 levels should be evaluated in patients presenting with peripheral sensory neuropathy. It is associated with decreased deep tendon reflexes, pathologic reflexes like Babinski's sign, and fatigue/depression. The inability to whistle is associated with fascioscapular dystrophy. Hydrophobia is associated with rabies infection.

     

    Smith and Singleton evaluated 138 patients referred with predominantly sensory symptoms to identify a standardized approach to diagnosis. They recommend that patients be tested for glucose tolerance and vitamin B(12) concentration in all cases, but that other tests should be performed only when the clinical scenario is suggestive.

     

    Steiner et al. describe a case report of a patient with vitamin B12 sensory peripheral neuropathy and associated EMG evidence of nerve demyelination as the potential cause for the observed clinical symptoms.

     

     

     

  7. A 8-month-old male presents with the hand deformity seen in Figure A. Examination reveals restrictions in passive extension and abduction of the thumb. His mother tells you that her other two children had similar deformities that required operative management. What would you offer as the first-line treatment option for this child's deformity?

     

     

     

    1. Observation

    2. Serial splinting of thumb

    3. Thumb-index finger web space reconstruction

    4. Thenar muscle tendon transfer

    5. First metacarpophalangeal joint arthrodesis

    CORRECT ANSWER: 2

    This patient is presenting with a congenital clasped thumb. The first line treatment for all types of congenital clasped thumb is serial splinting of thumb for 3-6 months.

     

    Congenital clasped thumbs are flexion-adduction deformities of the thumb that persist beyond the 3rd or 4th month of life. They usually result from the absence or hypoplasia of the extensor pollicis brevis (EPB) and/or the extensor pollicis longus (EPL). The deformity was categorized by Tsuyugushi from Type I - III based on anatomy of deformity. All types are initially managed conservatively with serial splinting for 3-6 months. Surgical treatment with first webspace widening and tendon transfer are considered after failed conservative management.

     

    Tsuyuguchi et al. reviewed 75 hands with congenital clasped thumb. Three

    groups were identified based on presenting deformities. All patients with type I deformities showed good results with splinting. Type II and III deformities had poor results to splinting and required operative management.

     

    Hisham et al reviewed the characteristics in 40 patients with congenital clasped thumb. They concluded that conservative management should be initiated in all Type I and II patients initially.

     

    Ruland and Slakey do an excellent job of reviewing the critical differences between trigger thumb and congenital clasped thumb.

    Figure A shows a moderate clasped thumb deformity (Type II deformity). Incorrect Answers:

    Answer 1: The deformity is unlikely to correct with observation

    Answer 3: Thumb-index finger web space reconstruction is considered for Type II or III deformities when there is significant contracture of the skin in the web space.

    Answer 4: Thenar muscle tendon transfer can be considered with failed treatment of type I or II deformities.

    Answer 5: First metacarpophalangeal joint arthrodesis is a last resort treatment for deformities associated with significant MP joint instability and deformity.

     

     

     

  8. A 34-year-old male undergoes a thumb replantation after an industrial meat slicer accident. At 4 hours postoperatively there is a drop from 33 degrees Celsius to 29 degrees Celsius and the pulse oximetry monitor on the thumb reads 87%. All of the following are treatment options for the management of his arterial inflow insufficiency EXCEPT:

    1. Administer IV heparin

    2. Administer a stellate ganglion block

    3. Placement of leeches on the thumb

    4. Inspect the dressing for any constriction

    5. Place the thumb in a dependent position.

       

      CORRECT ANSWER: 3

       

      Arterial thrombosis after digit replantation typically occurs within the first 12 hours postoperatively whereas venous thrombosis/congestion occurs after the first 12 hours postoperatively. Leeches excrete Hirudin, which is 100 times

      more potent than heparin, but are typically used for the treatment of venous thrombosis/congestion and not arterial thrombosis.

       

      Miller's review states a drop in temperature >2 C in 1 hour or temperature below 30 C indicates decreased digital perfusion. If arterial insufficiency develops: release constrictive bandages, place the extremity in a dependent position, consider heparinization, consider stellate ganglion blockade, or explore early if these maneuvers do not work.

       

       

       

  9. A 16-year-old football player sustains an injury to his ring finger after making a tackle. A clinical photograph is shown in Figure A. What is the most likely diagnosis?

     

     

     

     

    1. Flexor digitorum superficialis avulsion

    2. Central slip rupture

    3. Sagittal band rupture

    4. Distal extensor tendon rupture

    5. Flexor digitorum profundus avulsion

    CORRECT ANSWER: 5

    The photograph demonstrates the inability to flex the ring finger DIP. Based on the mechanism and clinical findings this injury represents a "rugby jersey finger", which is an avulsion of the flexor digitorum profundus (FDP) tendon.

     

    Tuttle et al reviewed these injuries and concluded treatment for an acute injury is FDP tendon reinsertion. For chronic injuries, a 2-staged tendon grafting is required.

     

     

  10. What is the most important variable when deciding between reconstruction versus ablation of a hypoplastic thumb?

    1. contralateral thumb function

    2. thenar muscle function

    3. presence of an extensor pollicis longus tendon

    4. carpometacarpal joint stability

    5. metacarpophalangeal joint stability

    CORRECT ANSWER: 4

    The carpometacarpal (CMC) joint must be sufficiently stable to provide resistance during grasp and pinch. This is the main determinant of the feasibility of reconstruction of the hypoplastic thumb.

     

    To function correctly, the thumb must be positioned so that it can oppose the adjacent medial fingers and grasp objects securely from an antiposed (abducted, slightly extended, and pronated) position. Although normal motion is usually not required at the MP or interphalangeal joints, thumb function is greatly dependent on preserving a full arc of circumduction at the CMC joint. The Blauth's classification ranges from type I to V and Types IIIB to V are treated with pollicization. The key difference between a Blauth IIIA and IIIB is the presence of carpometacarpal joint stability in Blauth IIIA.

     

    Tay et al. review thumb hypoplasia with an emphasis on classification-based treatment. They outline the goals of reconstruction as follows: "achieving independent flexor and extensor function, adequate first web space for grasp and adequate length, as well as joint stability, sensibility, and cosmoses."

     

    Illustration A provides the details of the Blauth classification of thumb hypoplasia.

     

    Incorrect Answers:

    Answer 1. Contralateral extremity function is an important consideration in any upper extremity condition but does not dictate the treatment undertaken.

    Answer 2. Thenar muscle function must be addressed but is not the main determinant of treatment choice.

    Answer 3. Extrinsic muscle function must be addressed but is not the main determinant of treatment choice.

    Answer 5. Metacarpophalangeal joint stability must be addressed but is not the main determinant of treatment choice.

     

     

     

     

     

  11. In a patient with -2.5mm of ulnar variance, which of the following statements best describes the distribution of compressive load across the wrist?

    1. Approximately 50% of the wrist load is accepted by distal radius and 50% is accepted by the distal ulna

    2. Approximately 80% of the wrist load is accepted by the distal radius and 20% is accepted by the distal ulna

    3. Approximately 80% of the wrist load is accepted by the distal ulna and 20% is accepted by the distal radius

    4. Approximately 95% of the wrist load is accepted by the distal radius and 5% is accepted by the distal ulna

    5. Approximately 60% of the wrist load is accepted by the distal radius and 40% is accepted by the distal ulna

    CORRECT ANSWER: 4

     

    Ulnar variance describes the cranio-caudal position of the distal ulna in relation to the distal radius at the wrist. In neutral ulnar variance, 80% of the

    compressive load across the wrist is accepted by the distal radius, and 20% is accepted by the distal ulna. With -2.5mm of ulnar variance (negative ulnar variance), approximately 5% of the wrist load is accepted by the distal ulna. With +2.5mm of ulnar variance (positive ulnar variance), approximately 40% of the wrist load is accepted by the distal ulna.

     

    As discussed in the biomechanical study by Palmer and Werner, the loading characteristics of the wrist are dependent on the radio/ulnar variance.

    Specifically, a 2.5 mm increase in ulnar variance increases load accepted by ulno-carpal joint from 18% to 42%; a 2.5 mm decrease in the ulno-carpal variance will decrease the load accepted by the ulno-carpal joint to 4.3%.

     

    Friedman and Palmer review the clinical diagnosis, pathophysiology, and treatment of ulnar impaction syndrome.

     

     

     

  12. A 34-year-old male sustains amputations of the 4th and 5th fingers at the level of the middle phalanx after cutting them with a butcher knife. Which of the following techniques would most likely increase total surgical time during replantation?

    1. Utilizing an on-call surgical team experienced at digit replantations

    2. Digit-by-digit repair method

    3. Structure-by-structure repair method

    4. Bone shortening procedures to avoid the need for vein grafts

    5. Obtaining a thorough inventory of the digit structures that are deficient and the structures that are available for reconstruction

    CORRECT ANSWER: 2

     

    Surgical time in multiple digit replantation is increased by digit-by-digit repair techniques and decreased by structure-by-structure repair techniques.

     

    The Level 5 article by Morrison and McCombe reviews the indications and results of finger replantation. Results of replantation from the DIP to PIP joint typically have good outcomes whereas replantations at the proximal interphalangeal (PIP) joint to MCP joint have poor outcomes due to flexion contractures. The review article by Wang cites that tenolysis and tendon procedures were needed in 47.2% of the published cases of digit replantation and is the leading type of secondary operation.

     

    The classic article by Waikakul et al reviewed 1018 digital replantations in 552

    patients. They had a 92% rate of successful outcome and found that type of injury was the most important factor influencing immediate and late outcome.

     

     

     

  13. Which of the following structures is an anatomical component of the triangular fibrocartilage complex?

    1. Extensor carpi ulnaris tendon sheath

    2. Lunotriquetral interosseous ligament

    3. Extensor digiti minimi tendon sheath

    4. Radioscaphocapitate ligament

    5. Flexor carpi ulnaris tendon sheath

    CORRECT ANSWER: 1

    The extensor carpi ulnaris tendon sheath is part of the triangular fibrocartilage complex (TFCC).

     

    Palmer et al studied the anatomy and function of the TFCC through anatomical dissections and biomechanical testing. The TFCC was found to be composed of the sheath of the extensor carpi ulnaris (ECU), an articular disc, the dorsal and volar radioulnar ligaments, the meniscus homologue, and the ulnar collateral ligament. Biomechanically, they determined that the TFCC functions as a cushion at the ulnocarpal interface, and is a major stabilizer of the DRUJ.

     

    Nakamura et al histologically examined the origins and insertions of the TFCC in fresh-frozen cadaver wrists. They found that the floor of the ECU sheath originated from the dorsal side of the fovea of the ulna, through an arrangement of Sharpey's fibers.

     

    Illustration A shows the anatomy of the TFCC.

     

     

     

     

     

  14. A 25-year-old left hand dominant musician sustains an injury to the left thumb shown in Figure A. He is unable to extend the interphalangeal joint and has less than 2 second capillary refill at the thumb. What is the most appropriate method to achieve soft tissue coverage after extensor tendon repair or transfer?

     

     

     

     

    1. Moberg advancement flap

    2. Vacuum-assisted wound closure

    3. Wet-to-dry dressings

    4. First dorsal metacarpal artery (Kite) flap

    5. V-Y advancement flap CORRECT ANSWER: 4

    The clinical scenario is consistent with a dorsal thumb avulsion with missing extensor tendon and exposed bone necessitating soft tissue coverage. The first dorsal metacarpal artery (Kite) flap is the most appropriate flap for defects of the dorsal aspect of the thumb.

     

    Fassler et al in a Level 5 review state that the first dorsal metacarpal artery (Kite) flap is appropriate for defects of the dorsal aspect of thumb. The flap is performed in one stage with the skin over the dorsum of the proximal index finger elevated with incisions on all four sides. An incision is extended proximally over the dorsum of the first web space, and a pedicle containing the first dorsal metacarpal artery, the subcutaneous veins, and branches of the dorsal sensory branch of the radial nerve is isolated. The skin island with the attached pedicle is transferred to the thumb defect and sutured in place.

     

    Illustration A shows the technical steps of the first dorsal metacarpal artery (Kite) flap. Illustration B shows the final functional results of the first dorsal metacarpal artery (Kite) flap are shown in Illustration B.

     

    Incorrect Answers:

    Answer 1: Moberg advancement flaps are indicated for volar thumb defects. Answer 2 & 3: Wet to dry dressings or vaccuum-assisted wound closure would be inappropriate in this situation.

    Answer 5: V-Y advancement flaps are most appropriate for transverse or dorsal oblique fingertip amputations.

     

     

     

     

     

     

     

     

  15. A 54-year-old female presents with a hand deformity. A surgical procedure is being considered that relocates the lateral bands dorsally to counteract the pathophysiology of the deformity. Which of the following deformities does this patient most likely have?

    1. Boutonneire finger deformity

    2. Lumbrical plus finger deformity

    3. Mallet finger deformity

    4. Jersey finger deformity

    5. Swan neck finger defomity

    CORRECT ANSWER: 1

    Boutonniere deformity is characterized with the PIP in flexion and the DIP in hyperextension as shown in Illustration A. It is caused by central slip rupture or attenuation (secondary to capsular distention, e.g., rheumatoid arthritis), laceration, or traumatic disruption. Volar subluxation of the lateral bands due to incompetence or disruption of the triangular ligaments leads to increased deformity as the lateral bands become flexors of the PIP. Relocation of the lateral bands to their original dorsal position to counteract the pathophysiology of the deformity is an option for patients that have an approximately 40 degree active flexion contracture but full passive extension.

     

     

     

     

     

     

     

  16. Which of following most appropriately details the anatomic orientation of the digital nerve and artery in the finger?

    1. Artery palmar and radial to nerve

    2. Nerve dorsal and ulnar to artery

    3. Nerve medial to artery

    4. Artery ulnar to nerve

    5. Artery dorsal to nerve CORRECT ANSWER: 5

    The digital nerve is palmar to the artery in the finger. A helpful way of remembering this orientation is that sensation is performed with the pads (nerve is palmar) of your fingers and you test for cap refill at the fingernail (artery is dorsal).

     

    Illustration A shows a cross section of the finger and the relative position of the artery and the nerve. Illustrations B and C depict cross-sectional anatomy

    of the finger including Grayson's (involved in Dupuytrens) and Cleland's ligament (not involved in Dupuytren's)

     

     

     

     

     

     

     

     

     

     

     

  17. A 35-year-old butcher inadvertently lacerates his ring finger FDP tendon at the level of the DIP joint which is subsequently repaired. Following the operation he notes the inability to fully flex his long and small fingers at the DIP joints with attempted fist clenching as well as a weak grip. Which of the following intraoperative maneuvers was likely responsible for this?

    1. FDP reconstruction with a long tendon graft

    2. FDS to FDP transfer at level of the A2 pulley

    3. Inadequate repair of the C3, A4 and A5 pulleys

    4. Distal advancement of lumbricals

    5. Overtensioning of the FDP tendon

    CORRECT ANSWER: 5

    The clinical presentation is most consistent with the quadrigia effect which is caused by overtensioning of the FDP tendon during surgical repair.

     

    The FDP tendons share a common muscle belly and have many interconnections. Overtensioning one tendon has a reciprocal effect on the length-tension curve of the remaining three muscle-tendon units, weakening grip strength in these digits.

     

    Malerich et al performed a cadaveric study looking at FDP advancement on hand function. They determined advancement >1cm can lead to an imbalance of muscle function in the profundus system.

    Kaufmann et al. studied maximal grip strength and point of contact in the extrinsic system. They determined that FDP grip strength was optimized when the FDP point of contact was at the DIP. Thus moving the FDP insertion point distal or advancing a lacerated FDP tendon leads to a decrease in grip strength.

     

     

     

  18. A 23-year-old presents with a knife laceration in the flexor zone 2 of the hand. Examination of the wound is performed and a laceration of the flexor tendon one-half the width of the tendon is identified. There is no triggering present as the patient's finger is passively extended and flexed fully. The most appropriate treatment is:

    1. No tendon repair with early protected range of motion

    2. No tendon repair with splint immobilization for 2 weeks

    3. Tendon repair with 2 strand repair and early protected range of motion

    4. Tendon repair with 2 strand repair with splint immobilization for 2 weeks

    5. Tendon repair with 4 strand repair and early active range of motion

      CORRECT ANSWER: 1

      Nonsurgical treatment with early protected range of motion is indicated for flexor tendon lacerations one-half the width of the tendon. The article by Al-Qattan recommends that partial flexor tendon lacerations should be fixed if the laceration is greater than 60%. Furthermore, the patient should be witnessed under digital block that they have full extension and flexion without triggering which would be another indication to operate. Rehabilitation consists of early ROM, wrist and MP flexed in dorsal splint, PIP and DIP extended, Passive digital flexion with wrist flexed, and wait until eight weeks postop to begin strengthening. This concept was also tested in question 9 of the 2007 OITE with the cited reference by McGeorge and Stillwell comparing the results of repair with non-repair in humans for zone 2 injuries and concluded that tendons lacerated by 60% or less should not be repaired.

       

       

       

  19. Using the Blauth classification of thumb hypoplasia, grade IIIA can be treated with thumb reconstruction whereas grade IIIB is treated with thumb amputation & pollicization. What is the key difference between these two grades?

    1. Presence of complete osseous structures

    2. Presence of intact musculotendinous structures

    3. Carpometacarpal joint stability

    4. Metacarpophalangeal joint stability

    5. Presence of an extensor pollicis longus tendon

    CORRECT ANSWER: 3

    The key difference between a Blauth IIIA and IIIB is the presence of carpometacarpal joint stability in Blauth IIIA. The CMC joint must be sufficiently stable to provide resistance during grasp and pinch.

     

    Thumb hypoplasia is a rare congenital deformity affecting 1 in 100,000 live births. It is commonly associated with syndromes such as VACTERL, and is classified using the Blauth classification. To function correctly, the thumb must be positioned so that it can oppose the adjacent medial fingers and grasp objects securely from an antiposed (abducted, slightly extended, and pronated) position. The decision to treat with reconstruction vs. amputation and pollicization depends on carpometacarpal joint stability.

     

    Light and Gaffey discussed the management of the hypoplastic thumb. Types II and IIIA (intact CMC joint) can be treated with thumb reconstruction, whereas Types IIIB to V are treated with pollicization or amputation.

     

    Tonkin reviewed the classification of congenital thumb hypoplasia and suggested a modification to the Blauth Classification. Illustration A shows the Blauth Classification.

     

    Incorrect Answers:

    Answer 1: Type II has complete osseos structures. Types IIIA and IIIB both have osseous deficiencies.

    Answer 2: Type I has complete musculotendinous structures. Types IIIA and IIIB both have musculotendinous deficiencies.

    Answer 4: Types IIIA and IIIB both have absence of MCP stability. Answer 5: Types IIIA and IIIB both tendon deficiencies.

     

     

     

     

     

  20. All of the following have been implicated in the pathogenesis of Dupuytren's contracture EXCEPT?

    1. Fibroblast growth factor (FGF)

    2. Transforming growth factor- beta (TGF-beta)

    3. Myofibroblasts

    4. Platelet-derived growth factor (PDGF)

    5. CBFA-1 CORRECT ANSWER: 5

    Of the answers listed, only CBFA-1 has NO known role in the pathogenesis of Dupuytren's contracture. Core-binding factor alpha(1) (CBFA-1) is an essential transcription factor for osteoblastic differentiation and osteogenesis.

     

    Dupuytren contracture, a disease of the palmar fascia, results in the thickening and shortening of fibrous bands in the hands and fingers. The offending cells are thought to be myofibroblasts and fibroblasts. Growth factors such as basic fibroblast growth factor (FGF), platelet-derived growth factor (PDGF), and transforming growth factor-beta (TGF-beta) may signal the overproduction of the myofibroblasts and/or myofibroblastic activity of the fibroblasts. In addition, high levels of TGF-Beta may hinder apoptosis of the active myofibroblasts, unlike normal tissue healing.

     

    McGrouther discusses how the pathophysiology of Dupuytren's is related to the anatomy of the palmar ligaments. The article discusses the proposed etiology of Dupuytren's with a loss of normal motion between palmar fascial ligaments causes stress concentrations which stimulate fibrous tissue deposition and contracture.

     

    Baird et al performed a tissue analysis of 12 patients with Dupuytren's contracture compared to 12 control patients. They found that Dupuytren's expressed a higher percentage of peptide regulatory factors including interleukin-1 alpha, interleukin-1 beta, transforming growth factor beta, and basic fibroblast growth factor.

     

    Incorrect Answers:

    1-4: All of these answers are involved in the pathogenesis of Dupuytren's contracture.

     

     

     

  21. Syndactyly is associated with all of the following conditions EXCEPT?

    1. Apert syndrome

    2. Poland's syndrome

    3. Holt-Oram syndrome

    4. Carpenter syndrome

    5. Tay-Sach's Disease CORRECT ANSWER: 5

    Syndactyly is one of the most common congenital hand deformities with an incidence of 1 in 2000 live births and is associated with Poland's syndrome (chest wall deformity with limb hypoplasia), Apert syndrome (autosomal dominant inheritance with mental retardation, craniosynostosis), Holt-Oram syndrome (also known as "heart-hand syndrome" with abnormalities of the heart and upper limbs), and Carpenter syndrome (acrocephaly, peculiar facies). There is no association of syndactyly with Tay-Sach's disease.

     

    Ireland and colleagues reviewed 43 consecutive cases of Poland's syndrome and found that clinical features were variable but always included congenital aplasia and syndactyly which is usually incomplete and simple. It can involve all fingers and not uncommonly the thumb as well.

     

    Galland and Bora wrote a review on congenital disorders of the upper extremity, and describe that syndactyly may occur as part of a syndrome including Poland's syndrome (chest wall anomalies and syndactyly) or Apert's syndrome (severe syndactyly with craniosynostosis, mental retardation, ankylosed IP joints, flattened faces and hypotelorism). There is no known increased incidence in Tay-Sach's disease, however.

     

     

     

  22. A 32-year-old male sustains a type IIIb open proximal third tibia fracture. Four days after intramedullary nailing of the tibia, the wound is clean and ready for coverage with a medial gastrocnemius rotational flap. What is the dominant arterial blood supply to this flap?

    1. Superior and inferior genicular arteries

    2. Anterior tibial artery

    3. Posterior tibal artery

    4. Sural artery

    5. Saphenous artery CORRECT ANSWER: 4

    The dominant arterial blood supply to a medial gastrocnemius muscle flap is the sural artery.

     

    Rotational gastrocnemius flaps are useful for coverage of the proximal third of the tibia and some wounds/defects about the knee. Medial and lateral gastrocnemius arterial supply is from the medial and lateral sural arteries respectively. Coverage of the middle third of the tibia requires use of a rotational soleus muscle flap, supplied by the peroneal artery proximally and the posterior tibial artery distally. Coverage of the distal third of the tibia requires a free muscle flap transfer, based on a specific vascular pedicle.

     

    Illustration A depicts the medial gastrocnemius flap with its sural artery pedicle.

     

     

     

     

     

     

  23. Approximately what percentage of pre-operative grip strength would be expected 3 months after carpal tunnel release?

    1. 10%

    2. 25%

    3. 50%

    4. 100%

    5. 150%

     

    CORRECT ANSWER: 4

     

    Gellman et al quantified grip and pinch strength post-operatively after carpal tunnel release. They found grip strength was 28% of preoperative level at 3 weeks; 73% by 6 weeks, returned to the preoperative level by 3 months, and

    116% at 6 months. Pinch strength was 74% of preoperative level at 3 weeks, 96% at 6 weeks, 108% at 3 months, and 126% at 6 months.

     

     

     

  24. A 24-year-old professional baseball outfielder reports persistent pain in the hypothenar region when batting for the past year. His CT scan is shown in Figure A. What is the recommended treatment?

     

     

     

     

     

    1. pisiform excision

    2. hook of hamate excision

    3. carpal tunnel release

    4. decompression of Guyon's canal

    5. open reduction and internal fixation

    CORRECT ANSWER: 2

    The history is typical of a hook of the hamate fracture, which is confirmed on the CT image. A carpal tunnel view radiograph of this injury is shown in Illustration A. It commonly occurs in baseball players and golfers. Physical exam findings include point tenderness at the hamate, ulnar nerve paresthesias (hemorrhage within Guyon's canal), and pain with axial load of ring and little fingers. For cases seen late, with few exceptions, the recommended treatment has been excision of the hook fragment.

    Marchessault provides a review of diagnoses and treatment for carpal fractures. They discuss the treatment of these injuries, indicating that acute, nondisplaced fractures may be placed in a cast, and excised if nonunion develops. The authors go on to say that certain investigators recommend excision of asymptomatic nonunions to minimize the risk for flexor tendon rupture.

     

     

     

     

     

  25. Which of the following structures are slowly adapting skin receptors that detect pressure, texture, and low frequency vibration and are best evaluated by static two-point discrimination?

    1. Meissner's corpuscles

    2. Pacinian corpuscles

    3. Merkel's receptor

    4. Free nerve endings

    5. Ruffini corpuscles CORRECT ANSWER: 3

    Merkel's skin receptors are slowly adapting skin receptors that detect pressure, texture, and low frequency vibration and can be appropriately evaluated by static two-point discrimination. Merkel's disk receptors adapt slowly and sense sustained pressure, texture, and low-frequency vibrations.

     

    Szabo et al state in their review that static and moving two point discrimination are best to initially evaluate innervation density for both quickly and slowly adapting fibers. Vibratory moving 2 point discrimination is best for evaluation of quickly adapting fibers.

     

    Meissner corpuscle, a rapidly adapting sensory receptor, is very sensitive to touch. Pacinian corpuscles are ovoid in shape, measuring approximately 1 mm in length. They respond to high-frequency vibration and rapid indentations of the skin. Ruffini corpuscles are slowly adapting receptors that detect stretching of the skin.

     

    Illustration A demonstrates Meissner's corpuscles (A), Pacinian corpuscles (B),

    Merkel's receptor (C), free nerve ending (D), and Ruffini corpuscles (E). Illustration B displays the function and location of the receptors.

     

     

     

     

     

     

     

     

     

  26. Which of the following peripheral nerve structures functions to cushion the nerve against external pressure?

    1. Endoneurium

    2. Fibronectin

    3. N-cadherin

    4. Epineurium

    5. Perineurium CORRECT ANSWER: 4

    The epineurium is a supportive sheath surrounding peripheral nerves that cushions fascicles against external pressure. It is comprised of a loose meshwork of collagen and elastin fibers that are aligned parallel with the nerve fibers.

     

    Illustration A & B depicts the contents of a nerve including epineurium, perineurium, and endoneurium.

     

    Incorrect Answers:

    Answer 1: Endoneurium is a fibrous tissue that covers the axon, Schwann cell, and myelin of each nerve fiber.

    Answer 2: Fibronectin and laminin are extracellular matrix glycoproteins that facilitate directional nerve fiber branching.

    Answer 3: N-cadherin is an adhesive membrane glycoproteins on neural ectoderm and facilitate growing axons.

    Answer 4: Perineurium is a dense connective tissue which surrounds nerve fascicles. It provides high tensile strength. The perineurium also limits diffusion within the intraneural environment and subsequently prevents injury from edema.

     

     

     

     

     

     

     

     

  27. An ulnar shortening osteotomy would be MOST indicated for which of the following patients presenting with longstanding ulnar sided wrist pain refractory to conservative measures?

    1. 34-year-old female with an ulnar neutral wrist and distal radioulnar joint incongruity

    2. 34-year-old female with an ulnar positive wrist and distal radioulnar joint incongruity

    3. 34-year-old female with an ulnar negative wrist and distal radioulnar joint incongruity

    4. 78-year-old female with ulnar positive wrist and distal radioulnar joint arthritis

    5. 78-year-old female with ulnar negative wrist and distal radioulnar joint arthritis

    CORRECT ANSWER: 2

    Ulnar shortening osteotomy is the best procedure for young adults with longstanding ulnar sided wrist pain due to ulnar positive variance and associated distal radioulnar joint (DRUJ) incongruity. Ulnar positive variance causes an "ulnar impaction syndrome" as the distal ulnar styloid can cause damage to the triangular fibrocartilage complex (TFCC), and ulnocarpal joint (illustration A.)

     

    Advantages of an ulnar shortening osteotomy include preservation of ulnar dome articular cartilage and DRUJ joint, and also tightens the TFCC and ulnocarpal ligaments as the distal ulna is translated and fixed proximally after the osteotomy.

     

    It is also important to note that ulnar shortening in the setting of preoperative DRUJ incongruity may simultaneously decrease ulnocarpal abutment and improve congruity at the distal radioulnar articulation. One specific instance in which to avoid an ulnar shortening in an ulnar positive wrist with DRUJ incongruity is a joint with a reverse oblique inclination in the coronal plane.

    This may create abnormally high radioulnar contact and may lead to joint degeneration

     

     

     

     

     

     

  28. What is the name of the pathologic structure, identified by the white arrow in Figure A, that displaces the digital neurovascular bundle and places it at risk during during surgical treatment of Dupuytren's disease?

     

     

     

    1. Pretendinous cord

    2. Pretendinous band

    3. Spiral cord

    4. Spiral band

    5. Natatory cord CORRECT ANSWER: 3

    The spiral cord, shown by the white arrow in Figure A, can displace the neurvascular bundle (blue arrow) and places it at risk during surgical resection. Dupuytren's contracture is a rare and progressive condition characterized by contractures of the fascia of the hand as seen in Illustration

    A. The fascial components involved in the disease include the pretendinous bands, spiral bands, natatory bands, lateral digital sheets, and Grayson's ligament. The offending cell is the myofibroblast which causes the normal structures to become fibrosed. Once these normal bands become pathologically involved in the disease process, they are termed cords. An easy way to remember this is that bands are normal, and cords are abnormal. The spiral cord travels dorsal to the NVB and displaces it volarly, placing it at risk during surgical resection. Example is shown in Illustrations B. Of note, Cleland's ligament is not involved in this disease process.

     

     

     

     

     

     

     

     

  29. A 19-year-old football player suffers a fall onto a pronated, extended wrist. He has pain with resisted ulnar deviation and is tender to palpation just distal to the ulnar styloid. He has no tenderness over the extensor carpi ulnaris (ECU) tendon. Current radiographs are shown in in Figures A and B and and MRI of the wrist is shown in FIgure C. Which of the following is the most likely diagnosis?

     

     

     

     

     

     

     

     

     

    1. ECU tendon rupture

    2. Triangular fibrocartilaginous complex (TFCC) tear

    3. Hook of hamate fracture

    4. Scapholunate ligament injury

    5. Perilunate dislocation CORRECT ANSWER: 2

    Fall from standing onto an extended and pronated wrist is a risk factor for injuries to the soft tissues of the wrist. The structures at risk include the triangular fibrocartilaginous complex (TFCC), the lunotriquetral ligament, ulnolunate ligament, hook of hamate, ulnar styloid, and the extensor carpi ulnaris (ECU) tendon sheath. Pain with resisted ulnar deviation and ulnar catching are all concerning for injury to the TFCC. MRI is useful for diagnosing TFCC tears ( Illustration A shows another example).

     

    Papapetropoulos et al in their review article discuss the evaluation and arthroscopic treatment of TFCC injuries. Specifically they discuss that most tears in athletes are acute and amenable to repair by repair of the dorsal tear to the ECU tendon sheath.

     

    Cohen in his review of injuries in athletes discusses scapholunate ligament, lunotriquetral ligament, and midcarpal injuries. Of note he divides scapholunate and lunotriquetral ligament injuries into dissociative lesions (abnormal motion within proximal carpal bones) vs. midcarpal lesions which are generally considered nondissociative (abnormal motion between proximal and distal carpal bones).

     

    Rettig in his review of sports injuries of the extremities discusses the Palmer classification of TFCC tears. Specifically he notes that central tears are more associated with repetitive activities in patients with positive ulnar variance.

     

    Incorrect Answers:

    Answer 1: The patient is not tender in the region of the ECU tendon sheath. Answer 3: The carpal tunnel view radiograph shows no hook of hamate fracture.

    Answer 4 and 5: Wrist radiographs shows no scapholunate widening or perilunate dislocation. Physical exam in this case is more consistent with a TFCC injury.

     

     

     

     

     

  30. A 32-year-old professional baseball player presents with wrist pain after a fall on his outstretched wrist 10 days ago. He initially thought it was a sprain, but presents due to continued pain worsened by push-ups. His physical exam shows dorsal wrist tenderness and is positive for the provocative test shown in Figure V. Standard PA radiograph of the wrist is normal. Which of the following radiographic views shown in Figures A to E would be most helpful in establishing the diagnosis?

     

     

     

     

     

     

     

     

     

     

     

    1. A

    2. B

    3. C

    4. D

    5. E

     

    CORRECT ANSWER: 1

     

    The clinical description and video of the patient's physical exam are consistent with an acute scapho-lunate ligament tear. The video shown in the question stem demonstrates the Watson test. When positive, the patient will feel dorsal wrist pain and/or a "clunk" when the wrist is brought from extension/ulnar

    deviation to radial deviation. If plain radiographs are normal, a PA clenched fist radiograph as seen in Figure A should be performed.

     

    In patients with a acute scapho-lunate ligament tear, initial radiographs may not show the characteristic "Terry Thomas" sign, or widening of the SL gap > 3mm. When making a clenched fist, the capitate is drawn proximally, stressing the SL ligament. This is an easy view to obtain during the initial patient visit and should strongly be considered if this diagnosis is suspected.

     

    Walsh et al review the various aspects of scapholunate ligament injuries. While they agree imaging is helpful in establishing the diagnosis, they emphasize that wrist arthroscopy is the gold standard in the diagnosis of SL injuries.

     

    Illustration A shows demonstrates a clenched fist view with obvious widening of the scapho-lunate gap.

     

    Incorrect Answers

    Answer 2: Shows a lateral radiograph in 30 degrees of supination. It is excellent for assessment of pisotriquetral arthrosis.

    Answer 3: Shows a PA of the wrist in radial deviation. This view will actually close the SL gap.

    Answer 4: Shows a a carpal tunnel view, used for assessment of hook of hamate fractures.

    Answer 5: Shows a a stardard PA wrist in neutral aligment.

     

     

     

     

     

  31. A 44-year-old man presents with ulnar-sided right wrist pain and mild constant tingling in the fourth and fifth digits after injuring his wrist while playing golf. Although pain and function have improved with conservative treatment 6 months following the injury, he still reports difficulty with his golf game. Which of the following should initially be obtained in this patient to aide in the diagnosis?

    1. Bone scan of the wrist and hand

    2. EMG study of the affected extremity

    3. Carpal tunnel view radiograph

    4. CT scan of the distal forearm and wrist

    5. Contrast enhanced magnetic resonance angiogram

    CORRECT ANSWER: 3

    This patients clinical presentation is most consistent with a chronic hook of the hamate fracture, which should initially be evaluated with a carpal tunnel view radiograph. Hook of the hamate fractures typically are associated with pain localized to the hypothenar eminence, and chronic cases can be associated with neuropathy of the ulnar nerve. Excision of the hook through the fracture site usually yields satisfactory results in the presence of chronic injuries.

    Parker et al treated five patients with six hook of the hamate fractures over an eight year period. All patients ultimately underwent hook resection and returned to their previous level of activity in 6 to 8 weeks after surgery without loss of function. Based on their case series, they concluded that the entire hook should be resected to the base of the hamate as the primary form of treatment in hook of the hamate fractures.

     

    Illustration A: Patient positioning for carpal tunnel radiograph-wrist is extended 70 degrees, and beam is angled 25-30 deg to the long axis of the hand(arrow).

     

    Illustration B: Carpal tunnel view radiograph demonstrates a fracture at the base of the hook of the hamate(black arrow) and normal pisotriquetral joint space.

     

    Incorrect Answers:

    1. Bone scans are not typically indicated in the diagnostic setting of acute or chronic hook of the hamate fractures.

    2. Imaging should be obtained to rule out bony injury prior to obtaining an EMG study.

    1. CT scans can used to confirm the diagnosis of a hook of the hamate fracture after obtaining a carpal tunnel view radiograph.

    2. Contrast enhanced MRA of the wrist is typically used to diagnose hypothenar hammer syndrome or other vascular abnormalities.

     

     

     

     

     

     

     

     

  32. A 20-year-old park ranger trips and falls onto his right wrist with the wrist in extension and pronation. The local urgent care orders both radiographs and a CT, which you review and determine to be normal. The patient complains of ulnar-sided wrist pain. On exam, his tenderness is localized to the fovea. Ulnar deviation also causes him pain. There is no snapping sensation with wrist supination, flexion, and ulnar deviation. He otherwise has 5/5 strength to his first dorsal interosseous muscle with 4mm static two-point discrimination on the ulnar side of the 4th digit. Which of the following injuries is most likely responsible for his symptoms and exam?

    1. TFCC tear

    2. Ulnar styloid fracture

    3. Hook of hamate fracture

    4. Ulnar nerve injury in Guyon's canal

    5. ECU subluxation CORRECT ANSWER: 1

    The patient has sustained a TFCC injury, classically associated with a fall onto the wrist in pronation and extension. Symptoms include ulnar-sided wrist pain, discomfort with ulnar deviation, and most notably foveal tenderness.

     

    Radiographs are typically negative in TFCC injuries. Although radiographs must be critically assessed for ulnar positive variance, which can dramatically increase the load transmitted through the TFCC. Repair or debridement of the TFCC will have worse outcomes in the setting of ulnar positive variance. MRI has been shown to have a sensitivity between 74-100%. Arthroscopy is considered the gold standard for diagnosis of the presence, type, and extent of the tear. TFCC injuries are often treated initially with immobilization and/or a steroid injection. In patients who fail conservative measures, arthroscopic debridement or repair may be indicated, particularly in younger patients with acute tears. Ulnar, radial, or distal avulsions are also typically treated with arthroscopy while central tears may be more amenable to non-operative treatment. Repairs can be completed via an "outside-in" approach, affixing the TFCC to the dorsal capsule, or securing it back to the ulna via a transosseous tunnel.

     

    Papapetropoulos and Ruch reviewed 25 arthroscopic procedures in professional athletes. They noted the presence of TFCC tears, which were repaired by securing the avulsed articular disk back to the capsule and ECU subsheath.

    The authors did not report outcomes but had no complications. They advocated that athletes with TFCC injuries must be treated differently, with earlier arthroscopic repair, given that this patient population cannot afford extended time away from their sport.

     

    Palmer and Werner performed biomechanical testing on 61 specimens. The authors described TFCC to be comprised of an articular disc, meniscal homolog, dorsal and volar radioulnar ligaments, the ulnar collateral ligament complex, and the ECU sheath. They found TFCC perforations in 53% of the specimens, which they noted could potentiate ulna-lunate abutment and cartilage erosion given that the TFCC functions as a cushion for the ulnar carpus and a stabilizer of the distal radio-ulnar joint.

     

    Incorrect answers:

    Answer 2: An ulnar styloid fracture would be evident on X-Ray, which in this case is normal.

    Answer 3: A hook of the hamate fracture would be evident on a CT, which in this case is normal.

    Answer 4: An ulnar nerve injury in Guyon's canal may present with interossei weakness or numbness in the ulnar nerve distribution.

    Answer 5: If injured, the ECU may subluxate in supination, flexion, and ulnar deviation of the wrist. The patient may hear or feel a click.

     

     

     

  33. What is the most important factor influencing immediate and late outcome of digit replantation?

    1. Gender

    2. Anastamosing the proximal profundus tendon stump to the superficialis tendon of the amputated digit

    3. Regular cigarette smoking

    4. Utilization of composite skin and subcutaneous vein graft

    5. Mechanism of digit amputation injury

    CORRECT ANSWER: 5

    The Level 2 study by Waikakul et al is a cohort of 552 patients that underwent 1018 digit replantation. Mechanism of injury was the most important factor influencing the survival rate with an odds ratio of 46.3. Specifically, avulsion, degloving and extensive crushed amputation resulted in a low survival rate and poor functional outcome. Cigarette smoking and male gender were associated with worse results but not to the degree of the mechanism of injury. Utilization of composite skin and subcutaneous vein grafts as well as connecting the profundus to the superficialis at the anastomoses correlated to better outcomes. After the operation, 329 of 946 survived digits (34.7%) in 180 patients (35.4%) needed further reconstructive surgery to improve their function. Tenolysis was the most common procedure followed by staged tendon grafting and capsulotomy.

     

    The review article by Wang found that tendon procedures, specifically tenolysis, accounted for 47.2% secondary surgeries following digit replantation.

     

     

  34. A 22-year-old rugby player presents with a mass at the base of his ring finger 5 months after sustaining an injury while making a tackle. Physical examination demonstrates a lack of active distal interphalangeal joint flexion, but full passive range of motion of all joints of the ring finger. Radiographs are normal. What is the most appropriate treatment to regain normal finger function?

    1. excision of the palmar mass and 2-stage tendon grafting

    2. excision of the palmar mass and single stage tendon grafting

    3. excision of the palmar mass and distal interphalangeal joint fusion

    4. active silicone rod implantation

    5. flexor digitorum profundis repair

    CORRECT ANSWER: 1

    This patient has a chronic jersey finger (flexor digitorum profundis avulsion). With the chronicity of the injury, it would be more amenable to grafting rather than direct repair given retraction of the FDP tendon that occurs with time that makes direct repair impossible. According to Green’s text, active silicone tendon rod implants have not proven to be effective. A 2-stage tendon grafting is the treatment of choice in cases of neglected or chronic (>3mo) tendon injuries or when previous surgery has failed. Two-stage flexor tendon grafting involves implanting a silicone rod (flexible silicone–Dacron-reinforced gliding implant) in the first stage and a free tendon graft (usually palmaris longus or plantaris) through the pseudosheath formed around the silicone in the second stage as intitially described by Hunter and Salisbury in 1971. In single-stage flexor tendon grafting, the tendon graft notoriously adheres to the surrounding fibro-osseous tunnel significantly limiting range of motion, but in 2-staged tendon grafting the pseudosheath that is formed around the silicone implant in the first stage greatly reduces the formation of post-operative adhessions to the tendon graft in the second stage.

     

    Amadio et al showed at 6 month follow-up of staged flexor tendon reconstruction, patients expressed 54% good to excellent results, but 16% of patients required tenolysis following the second stage of the procedure.

    LaSalle et al followed 43 flexor tendon 2 stage reconstructions by comparing passive ROM after stage 1 to postop active ROM after stage 2. They reported 16% excellent results, 23% good, 26% fair, and 35% poor. They stated that tenolysis following the second stage improved results on the patients reporting poor outcomes. A distal interphalangeal fusion would be reserved for failed reconstruction or a patient that does not desire/will not be compliant with likely lengthy postoperative therapy needed for a staged tendon grafting.

     

     

  35. Ischemia induced by the conversion of hypoxanthine to xanthine is thought to be involved in the mechanism of reperfusion injury following restoration of circulation after replantation. Which of the following agents inhibits conversion to xanthine and has been shown to improve outcomes following digit replantation?

    1. Tissue plasminogen activator

    2. Heparin

    3. COX 2 inhibitor

    4. Allopurinol

    5. Leeches CORRECT ANSWER: 4

    Allopurinol is a xanthine oxidase inhibitor and may have a beneficial role in replantation. Inhibition of xanthine oxidase also decreases uric acid in patients with gout.

     

    Waikakul et al. published a randomized control trial with a 2-year follow-up comparing thumb replantation with and without adjunctive allopurinol. There were 60 patients in the trial group, and 38 patients in the control group. All were young, healthy laborers who had sharp or locally crushed amputations of the thumb at the proximal phalanx with a total ischemic time >10 hours. The standard management for thumb replantation was used in these patients, except that 300mg allopurinol was given orally in the trial group on admission and a further 300mg for another 5 days. After the operation, the trial group had a lower infection rate, and less postoperative pain and chronic swelling than the control group. Recovery of sensation was also better in the trial group.

     

     

     

  36. A 20-year-old college football quarterback reports a 5-month history of gradually increasing medial elbow pain that occurs with throwing. The pain occasionally refers distally along the ulnar aspect of the forearm. He denies any weakness; however, he notes occasional paresthesias on the volar and dorsal aspect of his small finger. A nerve conduction velocity study demonstrates only slightly increased latency across the cubital tunnel. What structure may be contributing to his symptoms?

    1. Supracondylar process

    2. Ulnar artery

    3. Lacertus fibrosis

    4. Arcade of Frohse

    5. Anconeus epitrochlearis

    CORRECT ANSWER:

    5

    Of the possible answers shown, the anconeus epitrochlearis (AE) is the only potential site of ulnar nerve compression.

     

    The AE is an accessory muscle that originates from the medial epicondyle and inserts on the olecranon in place of Osborne's ligament. The prevalence among those with surgically treated cubital tunnel syndrome has been cited to be 13-30%. The AE is thought to exert a mass effect on the ulnar nerve at the cubital tunnel and thereby contribute to the symptoms of cubital tunnel syndrome. The diagnosis of the AE as the cause of cubital tunnel syndrome is not often not made until surgery. As a result, management involves excision of the accessory muscle and in-situ ulnar nerve decompression with or without transposition.

     

    Maslow et al. obtained an elbow MRI on 199 patients who were indicated for a cubital tunnel release. The authors noted that an AE was present in 13.6% of patients. They found that patients with AE had a shorter post-operative recovery and had a more reliable improvement in symptoms as compared to those who did not have an AE.

     

    Kim et al. retrospectively reviewed 13 patients (16 elbows) who had an AE associated with cubital tunnel syndrome. Ten patients had unilateral ulnar neuropathy as supported by nerve conduction studies, and three had bilateral cubital tunnel syndrome (1 of which had normal nerve conduction studies). Of the 16 elbows, 8 underwent muscle excision and submuscular transposition and 8 underwent muscle excision and in-situ decompression. They found that all but one patient had symptom relief. The authors concluded that the presence of an AE is an incidental finding; however, if it is identified during cubital tunnel surgery, excision is recommended.

     

    Masear et al. reported on 5 cases of cubital tunnel syndrome associated with an AE. All patients underwent AE excision and were symptom-free postoperatively. Nine months after surgery, electrodiagnostic studies were obtained and all demonstrated improved nerve conduction velocities. The authors recommend for AE excision in symptomatic cubital tunnel syndrome but against transposition of the nerve.

     

    Illustration A is an intra-operative image of the AE encountered at the cubital tunnel during a routine ulnar nerve decompression.

    Incorrect Answers:

    Answer 1: The supracondylar process is a residual osseous structure on the medial aspect of the distal humerus that is present in 1% of patients and may precipitate median nerve symptoms.

    Answer 2: The ulnar artery can be a source of ulnar nerve compression within Zone III of Guyon’s canal in the wrist. Compression in zone III may be due to ulnar artery thrombosis or aneurysm and can cause sensory abnormalities but not on the dorsum of the hand.

    Answer 3: The lacertus fibrosis is a broad aponeurosis of the biceps brachii which can cause compressive neuropathy of the median nerve.

    Answer 4: The arcade of Frohse, the thick tendinous edge of the supinator, is the most common site of entrapment of the posterior interosseous nerve which classically presents without motor deficits.

     

     

     

     

     

     

     

  37. A 20-year-old man has pain, swelling, and popping over his index metacarpophalangeal joint after striking a wall 3 days ago. Radiographs are normal, but physical exam reveals a palpable defect over the dorsum of the joint with clenching of the fist, and this defect is resolved with extension of the metacarpophalangeal joint. What is the next most appropriate step in treatment?

    1. Trigger finger steroid injection

    2. Extension splinting of the metacarpophalangeal joint

    3. Metacarpophalangeal synovectomy

    4. Extensor hood reconstruction

    5. Metacarpophalangeal joint arthrodesis

    CORRECT ANSWER: 2

    “Boxer’s knuckle” refers to injury to the extensor hood mechanism that results following resisted extension ("flicking") of the finger or direct trauma to the MP joint, usually involving the radial sagittal band of the middle or ring finger.

    Often, both the sagittal band and the dorsal capsule are torn. The hallmark of the physical examination is pain over the MCP with a palpable defect in the dorsal capsule, and it is important to examine for EDC subluxation with MP flexion. Sagittal band injuries seen within 3 weeks of injury may be treated nonoperatively with an MP joint flexion blocking splint. Patients presenting later than 2 to 3 weeks after the injury or patients who failed a trial of splinting are candidates for surgical repair.

     

    Hame et al reviewed 27 patients who were treated for Boxer’s Knuckle. The authors concluded that in cases in which conservative treatment has failed, these injuries should be treated with sagittal band repair with centralization of the extensor tendon without repair of the capsule. In the acute period however (less than 3 weeks), as is the scenario for this patient, conservative management with extension splinting should be attempted first.

     

    Araki et al performed a study of 16 cases of rupture of the extensor hood initially treated conservatively with splinting. While 8/16 responded successfully to nonoperative management, the remaining 8 did not improve with conservative treatment and were eventually treated with surgical repair and closure of the joint capsule when injured.

     

    Illustration A shows a clinical image of a boxer's knuckle and Illustration B displays an axial T2 MRI with a sagittal band rupture. Video V demonstrates a sagittal band reconstruction.

     

     

     

     

     

     

  38. A 25-year-old left hand dominant woman presents to the emergency department with left hand pain, fevers, and chills. Three days prior, she was bit by her neighbor's cat. On physical exam, she

    experiences significant pain with volar palpation and passive extension of her long finger. A photograph of her injury is shown in Figure A. What is the next best step in treatment?

     

     

     

     

    1. MRI

    2. Closed catheter irrigation of the long finger flexor sheath and radial bursa

    3. Wound swab and culture-directed antibiotics

    4. Open incision and drainage of the flexor sheath

    5. Resting volar splint, elevation, and IV antibiotics

    CORRECT ANSWER: 4

    This patient has developed septic flexor tenosynovitis (FTS) following a cat bite. Of the choices provided, treatment at this time involves incision and drainage (I&D) of the flexor sheath, optional drain placement, and pathogen-directed antibiotic therapy.

     

    The surgical treatment of FTS is based on the chronicity and degree of infection, as well as surgeon preference. For earlier and less aggressive infections, minimally invasive drainage and catheter irrigation may suffice; small incisions are typically made proximal to the A1 pulley and distal to the A4 pulley, with subsequent advancement of an 18-gauge angiocatheter and

    irrigation. For more severe infections, those presenting >1 week from symptom onset, and cases involving necrosis, treatment can range from extensive, open debridement of the flexor sheath, utilizing mid-axial or volar Brunner incisions, to amputation. More invasive procedures carry increased morbidity and may result in flap necrosis, but allow direct visualization and more extensive debridement.

     

    Draeger et al. provided a review article on FTS. They stress the importance of early diagnosis and expeditious treatment, including surgical drainage of the flexor sheath with open or closed drainage and intravenous antibiotics in the majority of cases. Patients who present within 48 hours of their initial injury may be managed non-operatively with splinting, elevation, and intravenous antibiotics; if no improvement is found within 12-24 hours, they should undergo I&D. They conclude that patients may still experience digital stiffness even after prompt and appropriate treatment.

     

    Henry provided a case report and focused literature review of a patient with FTS. His preference is for surgical decompression in the setting of FTS and feels that expeditious treatment improves subsequent finger function. He has had some success treating patients presenting within 24 hours of symptom onset with intravenous antibiotic therapy alone.

     

    Figure A is a left-hand photograph demonstrating a puncture wound of the long finger at the volar, middle phalanx. There is fusiform swelling and significant erythema. The involved digit is held in a slightly flexed position. Illustration A demonstrates the flexor pulley system of the hand.

     

    Incorrect Answers:

    Answer 1: FTS is a clinical diagnosis; a radiograph may be appropriate to rule our foreign body or fracture based on history, but MRI is not required and may delay treatment.

    Answer 2: The radial bursa and long finger flexor tendon sheath do not connect anatomically.

    Answer 3: A wound swab is inappropriate, as this will likely grow mixed skin flora. Furthermore, this patient requires surgical treatment and cultures can be obtained at the time of surgery.

    Answer 5: Application of a volar resting splint, elevation, and intravenous antibiotics may be appropriate in a patient with FTS presenting <2 days from initial symptoms, however, cat bites often require operative intervention.

     

     

     

     

     

  39. Which of the following locations correctly identifies the correct position of the initial portal used in wrist arthroscopy? Lister's tubercle corresponds to the blue circle.

     

     

     

    1. 1

    2. 2

    3. 3

    4. 4

    5. 5

     

    CORRECT ANSWER: 5

     

    The first portal used in wrist arthroscopy is the 3,4 portal (number 5), which is located just distal (approximately 1cm) to Lister's tubercle.

     

    At least 11 portals have been described in wrist arthroscopy: 5 radiocarpal, 4 midcarpal, and 2 distal radioulnar portals. The five radiocarpal portals are named by their relationship to the extensor compartments. The 3-4 portal is between the third and fourth compartments and is the initial site for portal placement. It is also the workhorse for wrist arthroscopy. It is located 1 cm distal to Lister's tubercle and is not located near any significant neurovascular structures. It is just radial to the extensor digitorum communis tendons and ulnar to the extensor pollicis longus tendon.

     

    Gupta et al. performed a review on wrist arthroscopy. They report that only the deep branch of the radial artery and the superficial dorsal sensory branches of the radial, ulnar, and lateral antebrachial cutaneous nerves are located on the dorsal aspect of the wrist. This makes wrist arthroscopy a safe procedure that not only yields diagnostic information but may have therapeutic options as well. They conclude that compared with open techniques, arthroscopic procedures may have a shorter recovery time and an earlier return to work for the patient.

     

    Kiliç et al. performed a cadaveric study to determine the course of the superficial branch of the radial nerve and its location in relation to wrist arthroscopy portals. The distance of the closest nerve branch to the 3-4 portal was 9 mm. They found a large amount of anatomic variation and therefore making a superficial skin incision with use of blunt dissection prior to portal placement to avoid damage to the superficial branch of the radial nerve.

     

    Figure A is a photograph of the dorsum of the wrist with Lister's tubercle marked by the blue circle. Illustration A (Gupta et al.) demonstrates the standard wrist arthroscopy portals. Illustration B (Gupta et al.) demonstrates the standard midcarpal portals, (STT: scaphotrapeziotrapezoid; TH: triquetrohamate).

     

    Incorrect Answers:

    Answer 1: Number 1 corresponds to the 6U portal.

    Answer 2: Number 2 corresponds to the 6R portal. Answer 3: Number 3 corresponds to the 4-5 portal.

    Answer 4: Number 4 corresponds to the radial midcarpal portal.

     

     

     

     

     

     

     

     

  40. A 28-year-old male sustains a laceration to the dorsal aspect of his left hand during an assault as shown in Figure A. He is unable to actively extend his ring finger. He undergoes primary repair of the injured structure and is placed into the relative motion splint (yoke splint) shown in figure B. All of the following are benefits of this splint when compared to full-time extension splinting or dynamic splinting, EXCEPT:

     

     

     

     

     

     

     

    1. Interferes less with activities of daily living

    2. Better at limiting motion of the digit

    3. Decreased risk of adhesions

    4. Allows more range of motion of adjacent digits

    5. Higher patient compliance with therapy

      CORRECT ANSWER: 2

      The relative motion splint (yoke splint) allows for safe early active range of motion injured digit after an extensor tendon laceration.

      This patient sustained an extensor tendon laceration in extensor tendon zone

      VI. Following repair of complete lacerations, traditional treatment includes full-time extension splinting, but this treatment can lead to adhesions and limited motion. In addition, full extension splinting or dynamic splinting with outriggers interferes substantially with daily activities. The relative motion splint (yoke splint), holds the involved digits in relative hyperextension at the MCP joint which allows for safe active MCP/PIP/DIP motion immediately following surgery.

       

      Howell et al. review 140 cases over a 20 year period using the yoke splint. They report excellent motion with few extensor lags and no re-ruptures, as well as an early return to work and high patient compliance with the therapy.

       

      Sharma et al. performed a cadaveric biomechanical study looking at the relative motion splint. They determined that the relative motion splint reduces the effective strain on intact and repaired zone VI middle finger extensor tendons and supports its clinical use.

       

      Figure A is a clinical photograph with a laceration in extensor tendon zone VI on the dorsum of the hand. Figure B is an example of a yoke splint that holds the involved MCP in relative extension. This position decreases strain on the repair site while allowing motion to decrease adhesions.

       

      Incorrect answers:

      Answer 1: The yoke splint interferes less with ADL's than extension splinting and the bulkier dynamic splints with outriggers.

      Answer 3: By allowing early active ROM, the yoke splint decreases the risk of adhesions in comparison to full-time extension splinting for 4-6 weeks.

      Answer 4: The relative motion splint allows adjacent digits more range of motion that full-time extension splinting.

      Answer 5: The yoke splint has a high rate of patient compliance and satisfaction with therapy.

       

       

       

  41. A 55-year-old female presents with persistent right palm, thumb, index, and long finger paresthesias with associated weakness. She reports a carpal tunnel released 1 year ago with no change in her preoperative symptoms. She reports no nighttime symptoms at any point but reports trouble using a key and volar proximal forearm pain. Figure A and B shows her hand clinically and radiographically. On physical exam you note normal 2-point discrimination in all digits with

    weakness of FPL, FDP to the index, and FCR. She has a negative CMC grind test. Figure C shows radiographs of her elbow. Elbow exam is normal other than pain with palpation of the lacertus fibrosis and a positive Tinel's test over the medial antecubital fossa. Following failure of appropriate nonoperative measure, which of the following is the next best step in treatment?

     

     

     

     

     

     

     

     

     

    1. Decompression of median nerve at the elbow

    2. Decompression of the anterior interosseous nerve at the elbow

    3. Revision carpal tunnel release

    4. Decompression of ulnar nerve at the elbow

    5. Night-time bracing CORRECT ANSWER: 1

    The patient has median nerve compression at the elbow which should be treated with surgical decompression following failure of nonoperative treatments.

     

    Proximal median nerve entrapment (PMNE) or pronator syndrome is a cause of median nerve motor and sensory symptoms that may be confused with carpal tunnel syndrome. Findings specific to PMNE include lack of nighttime symptoms, paresthesias over the palmar aspect of the hand, pain at the volar forearm, pain over lacertus fibrosis, and weakness in forearm and hand muscles innervated by the median nerve. Dynamic testing with motions that produce repetitive pronation may elicit symptoms as well. Tests such as resisted elbow flexion with supination, resisted forearm pronation with elbow extension, and resisted FDS flexion of the long finger may identify the location of median nerve entrapment. Lastly, the scratch collapse test may be positive in patients with PMNE.

     

    Hagert surgically released 44 patients with PMNE under local anesthesia in a minimally invasive fashion. They found statistically significant improvements in quick DASH, work DASH, and activity DASH scores following surgical release.

    They conclude that PMNE at the level of lacertus fibrosis is a clinical diagnosis based on weakness, pain over the area of compression, and a positive scratch

    collapse test. They conclude surgical release under local anesthesia is a safe and cost-efficient procedure with low morbidity.

     

    Zancolli et al. described a minimally invasive decompression for pronator teres syndrome with release of the deep fascia of the superficial head of the pronator teres muscle. The three clinical tests they use for diagnosis include the compression test, forearm resisted pronation test, and middle finger flexor superficialis test. Lastly, in their series of 44 patients, all patients did have associated carpal tunnel syndrome.

     

    Figure A shows a clinical photograph of a hand without any signs of thenar atrophy. Figure B is a normal radiograph of a hand without signs of basilar joint arthritis. Figure C is a normal radiograph of a right elbow without evidence of a supracondylar process.

     

    Incorrect Answers:

    Answer 2: AIN syndrome is characterized by purely motor symptoms and would not address the patients paresthesias from more proximal nerve compression.

    Answer 3: The presentation is not consistent with carpal tunnel syndrome and the patient will likely remain unchanged with a revision carpal tunnel release. Answer 4: Ulnar nerve decompression is indicated when patients present with symptoms of cubital tunnel, however, this patient has no objective findings of ulnar nerve compression.

    Answer 5: This patient has failed nonoperative measures and continuing nonoperative measures will likely lead to no improvement of symptoms.

     

     

     

  42. A 57-year-old woman presents to clinic with left thumb pain. She reports difficulty with pinching and grasping. Physical examination is remarkable for a painful CMC grind test. A radiograph of her left hand is shown in Figure A. All of the following are possible sequelae of her disease EXCEPT:

     

     

     

    1. Thumb metacarpal adduction deformity

    2. Thumb interphalangeal (IP) joint flexion deformity

    3. Thumb carpometacarpal (CMC) joint extension deformity

    4. Thumb metacarpophalangeal (MCP) joint hyperextension deformity

    5. Thumb metacarpal palmar flexion deformity

    CORRECT ANSWER: 3

    The patient's clinical presentation and radiographs are consistent with CMC arthritis, which is not associated with the development of a thumb CMC joint extension deformity.

     

    Basilar joint arthritis is a common location of arthritis in the hand. Patients with basilar joint arthritis may attempt to avoid painful thumb abduction, leading to adduction and palmar flexion of the thumb and a subsequent 1st web space contracture. A secondary MCP hyperextension deformity may develop over time and may be compensated by a thumb IP flexion deformity. Hyperextension of the MCP joint is an important sequelae of CMC arthritis and must be addressed at the time the CMC joint is treated surgically, as it will produce a deforming force that will cause recurrent thumb metacarpal subluxation if left untreated.

    Van Heest et al. published a review of thumb CMC arthritis. They describe pertinent anatomy, pathophysiology, diagnosis, classification, and treatment of CMC arthritis. They note that metacarpal extension osteotomy is contraindicated in patients with an MCP hyperextension deformity >10°, as the volar aspect of the joint would still be overloaded.

     

    Armbruster et al. published an article discussing the evaluation and treatment of the metacarpophalangeal joint deformity in CMC arthritis. They report that the classic deformity that develops in advanced stages of CMC arthrosis is characterized by hyperextension of the thumb metacarpophalangeal (MCP) joint and adduction involving the first web space of the hand. They present a treatment algorithm based on the degree of MCP hyperextension.

    Figure A is a radiograph of the left hand demonstrating CMC arthritis. Incorrect Answers:

    Answers 1, 2, 4, 5: All of these are possible sequelae of CMC arthritis.

     

     

     

  43. A 62-year-old right hand dominant nurse comes to your clinic with complaint of 3 years of off and on bilateral numbness and tingling. She reports symptoms in her thumb, index, and middle fingers that wakes her from sleep. Figure A shows a clinical photograph of her hands. Flexion of her wrist worsens her symptoms and tapping over her carpal tunnel causes sharp pain into her thumb, index, and middle fingers. She has two point discrimination to 3mm in her ring and small fingers and 10mm in the remainder of her fingers. Her colleague in neurology suggested electrodiagnostic studies to confirm the suspected diagnosis. Which of the following is true regarding an electrodiagnostic studies in this scenario?

     

     

     

    1. Electrodiagnostic studies would be useful to confirm compression of the median nerve at the elbow

    2. Electrodiagnostic studies would be useful to confirm compression of the median nerve at the wrist

    3. Electrodiagnostic studies would be useful to rule out compression of the median nerve at the wrist

    4. Electrodiagnostic studies would be useful to rule out compression of the median nerve at the elbow

    5. Electrodiagnostic studies are unlikely to change the probability of median nerve compression at the wrist

    CORRECT ANSWER: 5

     

    Electrodiagnostic studies will not affect the probability of a patient having carpal tunnel syndrome when the clinical suspicion is high based on history and physical exam.

     

    Carpal tunnel syndrome is a clinical diagnosis based on a set of symptoms and objective findings. The CTS-6 is a scoring tool for carpal tunnel which categorizes patients into a high probability (>.80) and low probability (<0.25) of having carpal tunnel. In a high or low probability patient electrodiagnostic studies are unlikely to alter the probability that the patient does or does not have carpal tunnel syndrome.

     

    Graham et al. collected the 8 most highly ranked criteria used to diagnose carpal tunnel syndrome based on expert opinion. All possible combinations of these 8 criteria were ranked by another panel of experts. They found the correlation between model and panel in predicting CTS was 0.71. They concluded improving the consistency of diagnosis of CTS should lead to more effective treatment.

    Graham used the CTS-6 to compared the pretest probability of carpal tunnel syndrome and the post-test probability following electrodianogistic testing at a single institution. They found patients with a pretest probability of CTS of 0.80 had an average post-test probability change of -0.02 when stringent electrodiagnostic criteria were used. They concluded for the majority of patients with a high probability of carpal tunnel syndrome based on physical exam, electrodiagnostic studies do not change the probability of the diagnosis in a clinically meaningful manner.

     

    Keith et al provide clinical practice guidelines in diagnosis of carpal tunnel syndrome. They state physicians may obtain electrodiagnostic tests to differentiate among diagnoses or if there is the presence of thenar atrophy and/or persistent numbness. They state physicians should order electrodiagnostic tests if surgical management is being considered and clinical and provocative tests are positive.

     

    Figure A shows a patient with bilateral thenar atrophy secondary to compression of the median nerve. Illustration A shows the scoring for the CTS-6(Graham JSJS 2008) with a score 12 corresponding to an 80% probability of carpal tunnel and a score <5 corresponding to a pretest probability of 25%.

     

    Incorrect Answers:

    Answer 1 & 4: The patient's symptoms are coming from the level of the wrist so EMG would be unlikely to confirm or rule out nerve compression at the elbow.

    Answer 2 & 3: Electrodiagnostic studies would not affect the probability that nerve compression is at the wrist given the patient's classic symptoms and objective findings

     

     

     

     

     

  44. A 6-year-old right-hand-dominant male presents to the emergency department with increasing right hand pain and swelling. He sustained a palmar abrasion while playing outside three days prior. He was treated the following day at an outside urgent care center with oral antibiotics due to swelling. Images of his hand today are shown in Figures A and B. A radiograph is obtained and shown in Figure C. Of the following skin incisions shown in Figure D, which should be utilized to appropriately address this child's pathology?

     

     

     

     

     

     

     

     

     

     

     

    1. A

    2. A and B

    3. A, B, and C

    4. C and D

    5. E

     

    CORRECT ANSWER: 4

     

    This child has a collar button abscess between the ring and small finger web space. The treatment for this is surgical, with a dorsal and palmar incision avoiding the web space.

     

    A collar button abscess describes a deep web space infection of the hand typically resulting from a volar puncture wound. Instead of proximal migration, the infection typically spreads dorsal through a sinus in the superficial transverse metacarpal ligament; proximal spread is prevented by the palmar aponeurosis. Classically, patients present with pain and tenderness in the web space with abduction of the adjacent digits (V-sign). Treatment is always surgical and involves both a dorsal and volar incision with optional placement of a Penrose drain; avoidance of the web space is critical to avoid scar contracture and loss of abduction.

    Abrams et al. provide a general review article of hand infections and their treatment. They note the importance of expeditious treatment and the potential for great morbidity. An understanding of anatomy is critical to effective treatment. They also note the importance of recognizing deep space infections, their association with Staph and Strep organisms, and their treatment with surgical incision and drainage, often with dual incisions.

     

    Imahara et al. present a retrospective review highlighting the increase in community-acquired methicillin-resistant Staphylococcus aureus (CA-MRSA) in surgically treated hand infections. Thirty percent of the 159 study patients had an infection due to CA-MRSA, with a 41% increase in MRSA cases for each progressive year in their 11-year study. The only independent risk factor for MRSA in their study was a history of intravenous drug use.

     

    Figures A and B are clinical photos of a right hand demonstrating a collar button abscess of the ring and small finger web space; note the abduction of the adjacent digits. Figure C is a posterior-anterior right-hand radiograph.

    Figure D shows modified hand photographs demonstrating different surgical incisions utilized in hand surgery. Illustration A shows an operative photograph following the treatment of a collar button abscess with dorsal and volar incisions, along with placement of a Penrose drain.

     

    Incorrect Answers:

    Answers 1 and 2: These are Brunner-type incisions; these are not appropriate in the setting of a collar button abscess.

    Answer 3: Two vertical incisions, dorsal and palmar, avoiding the web space are the appropriate treatment when undertaking a surgical incision and drainage of a collar button abscess.

    Answer 5: The web space should be avoided when performing an incision and drainage of a collar button abscess to avoid contracture.

     

     

     

     

     

  45. A 19-year-old male presents to your clinic after sustaining a laceration to his left hand while trying to slice an avocado. On physical exam, his small finger DIP remains extended when his wrist is passively extended. Which of the following labeled structures shown in Figure A is likely injured?

     

     

     

    1. A

    2. B

    3. C

    4. D

    5. E

     

    CORRECT ANSWER: 4

     

    The flexor digitorum profundus, the structure indicated by the letter D in the diagram, was injured in this scenario.

     

    An accurate history and a careful physical exam are critical for correctly diagnosing flexor tendon injuries and developing a treatment plan. The tenodesis effect describes normal finger flexion with passive wrist extension. Finger extension with passive wrist extension suggests a flexor tendon injury, whereas finger flexion with passive wrist flexion suggests an extensor tendon injury. The zone of injury and isolated testing of the FDP and FDS tendons can aid to further identify the injured structure.

     

    Samora et al. provide a review article on flexor tendon reconstruction. They emphasize that improved methods of primary repair have largely replaced reconstructive surgeries, which play more of a role in delayed or failed treatment. They also discuss the role of one- and two-stage reconstructions.

     

    Kamal et al. also discuss the surgical management of flexor tendon injuries. They state that to examine the FDP tendon in isolation, the PIP joint should be held in extension while asking the patient to flex the involved digit DIP. They add that in zone 1 injuries, distal to the FDS insertion, only the FDP tendon is susceptible to injury.

     

    Figure A shows the flexor tendon anatomy and pulley system of the finger. Illustration A shows the different zones of flexor tendon injuries. Illustration B shows the tenodesis effect.

     

    Incorrect Answers:

    Answer 1: This structure is the A1 pulley, release of this is commonly performed for trigger finger

    Answer 2: This structure is the flexor digitorum superficialis; injury to this would not result in extension of the DIP joint with passive wrist extension. Answer 3: This structure is the A2 pulley; injury to this may result in bowstringing.

    Answer 5: This structure is the A4 pulley; injury to this may result in bowstringing.

     

     

     

     

     

     

     

     

  46. During an open carpal tunnel release, the recurrent motor branch of the median nerve is iatrogenically transected. What postoperative disability would be expected, and what is the most likely way this disability arose?

    1. Inability to adduct the thumb; cut the transverse carpal ligament radially

    2. Inability to oppose the thumb; cut the transverse carpal ligament ulnarly

    3. Inability to oppose the thumb; cut the transverse carpal ligament radially

    4. Inability to abduct the thumb palmarly; cut the transverse carpal ligament ulnarly

    5. Inability to extend the thumb; cut the transverse carpal ligament ulnarly

    CORRECT ANSWER: 3

    The recurrent motor branch of the median nerve innervates the opponens pollicis, which is responsible for thumb opposition; cutting the transverse carpal ligament radially increases the risk of transection in cases of a transligamentous recurrent motor branch variation (Answer 3).

     

    The recurrent motor branch of the median nerve innervates the majority of the thenar musculature, including opponens pollicis, abductor pollicis brevis, and most of flexor pollicis brevis. There are several described variations in the path of the recurrent branch of the median nerve, of which the three most common are the extraligamentous, subligamentous and transligamentous variations.

    While reports vary regarding the exact incidence of each variant, the variation that poses the most risk for transection occurs when this nerve branch passes through the actual fibers of the transverse carpal ligament (transligamentous variation).The recurrent branch originates from the central and radial aspect of the median nerve, and thus the transverse carpal ligament should be cut ulnarly to minimize the risk of transection.

     

    Kozin dissected 101 cadavers to describe the anatomy of the recurrent branch of the median nerve. In his series, 7% of the specimens had recurrent branches that were transligamentous, and none of the median nerves had an origin of the recurrent branch that was ulnar. Kozin concluded that the transligamentous variation of the recurrent motor branch of the median nerve was rare.

     

    Siverhus et al. dissected 72 cadaveric specimens to describe the anatomy of the recurrent branch of the median nerve. In their series, 86% of the specimens had "classic" recurrent motor branch anatomy, while 14% had recurrent motor nerve branches that were transligamentous anywhere from 2-6 mm proximal to the distal edge.

     

    Figure A shows the most common recurrent motor branch variations of the median nerve.

     

    Answer 1: Thumb adduction is performed by adductor pollicis which is innervated by the deep branch of the ulnar nerve.

    Answer 2: While the recurrent branch of the median nerve does innervate the opponens pollicis which permits thumb opposition, cutting the transverse carpal ligament ulnarly minimizes the risk of transection.

    Answer 4: While the recurrent branch of the median nerve does innervate the abductor pollicis brevis which permits palmar thumb abduction, cutting the transverse carpal ligament ulnarly minimizes the risk of transection.

    Answer 5: Thumb extension is performed by extensor pollicis brevis and longus, both of which are innervated the posterior interosseous nerve.

     

     

     

     

     

     

  47. You are planning on performing the flap reconstruction depicted in figure A. What injury pattern and patient characteristics best match the indication for this reconstruction technique?

     

     

     

    1. 66-year-old male with a volar oblique defect of the long finger

    2. 23-year-old female with a dorsal oblique defect of the long finger

    3. 29-year-old female with rheumatoid arthritis and a volar oblique defect of the index finger

    4. 45-year-old male laborer with a volar thumb tip defect

    5. 28-year-old male with a volar oblique defect of the index finger

    CORRECT ANSWER: 5

    Thenar flaps are best utilized for volar oblique defects of the long or index finger with exposed bone/tendon in a patient less than 30-years-old.

     

    Thenar flaps can be used for coverage of digital tip injuries where there is exposed bone or extensive pulp loss. Advantages include more subcutaneous fat than a cross-finger flap, good color and texture match, and primary closure of the donor site. Disadvantages include limited flap size and donor site tenderness. Contraindications include rheumatoid arthritis, Dupuytren’s contracture and advanced age with degenerative joint disease as these predispose the patient to developing proximal interphalangeal joint (PIP) joint stiffness/contracture. This risk is due to the fact that the technique involves suturing the involved digit to the palm for 2 to 3 weeks with the PIP joint in near-full flexion.

     

    Panattoni et al. describe the technique and indications for multiple flaps including the thenar flap. They add that elevation of the flap closer to the

    thumb metacarpophalangeal joint crease results in a better donor site because there is greater skin mobility allowing direct donor site wound closure and also necessitating less flexion on the recipient finger than the more proximal donor site that was originally described.

     

    Biswas et al. note that the thenar flap is frequently used for larger volar defects, particularly for the index and long fingers. Although the technique is an excellent method of restoring the soft tissue of more extensive defects of the fingertips, the procedure requires the digit to be immobilized in flexion and risks the development of PIP joint flexion contractures.

     

    Figure A is a clinical photograph of the donor site for a thenar flap used for a volar oblique skin defect of the long finger. Illustration A is a depiction of a thenar flap used for a defect of the index finger

     

    Incorrect answers:

    Answer A: Age >30 years is a relative contraindication to thenar flaps as these patients are more predisposed to PIP joint stiffness. A better treatment option for these patients is a cross-finger flap.

    Answer B: Dorsal defects with exposed bone are best treated with revision amputation +/- V-Y advancement and/or nail bed ablation, depending on the amount of bony support left for the nail bed, or with digital island artery flaps. Answer C: Rheumatoid arthritis is a relative contraindication to thenar flaps as these patients are more predisposed to PIP joint stiffness

    Answer D: Volar thumb defects are not treated with a thenar flap. They are best treated with a Moberg advancement flap for defects <2cm, first dorsal metacarpal artery flaps for defects >2cm, or neurovascular island flaps for defects up to 4cm.

     

     

     

     

     

  48. A 14-year-old gymnast presents to your clinic for a second opinion. Over the past few months, she has complained of medial elbow pain with paresthesias over the dorsal and volar aspects of her small finger. She demonstrates some intrinsic atrophy on examination. When she flexes her elbow the ulnar nerve demonstrates subluxation over the medial epicondyle. She attempted a trial of nighttime splinting, activity modification, and anti-inflammatory medications without significant relief. Which of the following pre-operative variables is a good prognostic indicator following surgical intervention?

    1. Ulnar nerve subluxation

    2. Gender

    3. Intrinsic muscle atrophy

    4. Valgus stress athlete

    5. Paresthesias CORRECT ANSWER: 1

    Young patients with cubital tunnel syndrome that demonstrate ulnar nerve instability (ability to subluxate or perch on the medial epicondyle with elbow flexion) show superior outcomes compared to patients without nerve instability.

     

    Cubital tunnel syndrome is a rare diagnosis in the pediatric and adolescent population. It can often be a result of trauma or repetitive use. The presenting symptoms are similar to those of an adult. They include paresthesias in the ulnar nerve distribution, elbow pain, and possible hand muscle weakness. Nonoperative treatment can be effective in up to 50% of patients and includes a trial of nighttime splinting, activity modification, and anti-inflammatory medications. Pediatric and adolescent patients fair well when surgical intervention is indicated. Ulnar nerve transposition is indicated in patients with an unstable nerve on examination (vs. in situ decompression).

     

    Stutz et al. reviewed 39 extremities of pediatric and adolescent patients with cubital tunnel syndrome. They found that 30 extremities that eventually underwent cubital tunnel decompression demonstrated significant improvements in DASH scores and VAS pain scores, while 30 extremities either initially or definitively treated with nonsurgical care had no symptomatic resolution. They conclude that surgical intervention is effective for symptomatic relief if nonsurgical care fails.

     

    Henn et al. reviewed the charts of 67 patients who were 30 years old or younger when they underwent primary cubital tunnel surgery. Thirty-four of

    the patients were diagnosed with unstable ulnar nerves, and these patients had significantly improved patient-reported outcome measures when compared to the stable nerve cohort at final follow-up. They conclude that surgical management of young patients with symptomatic, unstable ulnar nerves results in superior subjective outcomes compared to that of young patients with stable ulnar nerves.

     

    Incorrect Answers:

    Answers 2-5: These have not been shown to be good prognostic factors for surgical intervention in the pediatric population. Intrinsic muscle atrophy is considered a poor prognostic variable.

     

     

     

  49. An 18-year-old female military recruit falls and sprains her wrist during basic training. One month later she reports to clinic with a dorsal wrist mass. A wrist MRI is reviewed (Figures A and B). The histopathology of the lesion would reveal what cellular pattern?

     

     

     

     

     

     

     

    1. Uniform distribution of stromal cells and giant cells

    2. Mixture of mature fat cells and spindle cells

    3. Fibroblasts with mixed Schwann cells, mast cells, and lymphocytes

    4. Lobular pattern of vascular proliferation with inflammation

    5. Mucin-filled space with occasional spindled fibroblasts

    CORRECT ANSWER: 5

    This patient presents with clinical and radiographic findings typical of a dorsal wrist ganglion cyst. The histopathology characteristically shows a mucin-filled space with scattered spindled fibroblasts.

     

    Dorsal wrist ganglion cysts are commonly associated with wrist trauma and subsequent synovial herniation. These are the most common benign hand masses, typically arising from the SL interval. Radiographs often appear benign and the classic MRI findings are a low-signal homogenous mass on T1 and correspondingly high-signal on T2. These lesions do not require biopsy as the diagnosis is largely clinical, however the mass is always sent for pathology post-operatively. Histology will reflect a mucin-filled sac with spindled fibroblasts. Ganglion cysts are typically treated with observation unless chronic and persistently symptomatic, at which time aspiration or surgical excision is more appropriate.

     

    Balazs et al. retrospectively reviewed 125 active duty military personnel who underwent dorsal wrist ganglion excision. Post-operatively, 15% of patients required waivers for performing push-ups owning to persistent pain. The authors reported a 9% recurrence rate and found no specific demographic factors contributing to risk of recurrence. They cautioned that surgical excision may not return young active patients to the expected level of function.

     

    Head et al. compared surgical excision versus needle aspiration of 2,239 adult wrist ganglions in a meta-analysis of 35 studies. They authors showed that surgical excision resulted in a 76% reduction in recurrence compared to aspiration. They compared the mean recurrence rates for arthroscopic excision (6%), open surgical excision (21%) and aspiration (59%) as well as the mean complication rates for arthroscopic excision (4%) open surgical excision (14%) and aspiration (3%). Though data from arthroscopic excision was limited, the authors considered this a promising technique. They concluded that open surgical excision boasted a significantly lower recurrence rate as compared to aspiration.

     

    Pang et al. performed a cost analysis for patients undergoing open (5,119) or arthroscopic (20) ganglion excisions. The cost of an open excision ($1,821) was significantly lower than that of an arthroscopic excision ($3,668). The authors concluded that this should be weighed with the relative success of each procedure.

    Figures:

    Figures A and B are the axial T2 and T1 MRI sequences of the affected wrist demonstrating the characteristic location and appearance of a dorsal wrist ganglion cyst.

     

    Incorrect answers:

    Answer A: A uniform distribution of stromal and giant cells would be consistent with a benign giant cell tumor (GCT) of the tendon sheath. Recurrence after excision is always a concern with GCT of the tendon sheath. While clinical presentation may be similar, MRI would demonstrate low signal on both T1 and T2 sequences.

    Answer B: A mixture of fat and spindle cells describes the histology associated with a benign spindle-cell lipoma. MRI would demonstrate high signal on both T1 and T2 sequences.

    Answer C: Fibroblasts with mixed Schwann cells, mast cells, and lymphocytes would suggest a begin peripheral nerve sheath tumor. The signal would appear hypointense on T1 and hyperintense on T2 MRI sequences.

    Answer D: A lobular pattern of vascular proliferation with inflammation describes the histologic appearance of a begin pyogenic granuloma. This vascular, often pedunculated mass has characteristic MRI findings demonstrating iso-intensity with muscle on T1 and high signal intensity on T2 sequences, consistent with its hypervascular nature.

     

     

     

  50. A 30-year-old right-hand-dominant accountant has progressive wrist pain for the last 18 months. He was initially treated in a cast, however his symptoms have continuously worsened. Figures A and B are radiographs taken in the office today. He reports decreased grip strength, and physical exam is significant for decreased wrist extension and tenderness directly over the radiocarpal joint. Examination of the contralateral wrist is otherwise unremarkable. All of the following are described as etiologies for the above condition EXCEPT:

     

     

     

     

     

     

    1. Ulnar negative variance

    2. High interosseous pressure

    3. Underlying medical conditions

    4. Decreased radial inclination

    5. Increased carpal height

    CORRECT ANSWER:

    5

    The patient presents with advanced late-stage Kienbock disease, or avascular necrosis of the lunate. All of the above are risk factors for development of

    Kienbock disease except for increased carpal height. Conversely, a decrease in the carpal height is associated with avascular necrosis of the lunate.

     

    The etiology of Kienbock disease is multifactorial and includes ulnar negative variance, decreased radial inclination, vascular congestion from high interosseous pressure, and medical conditions including scleroderma, sickle cell anemia, systemic lupus erythematosus, and corticosteroid use. The condition is more prevalent in men ages 20-40 and may also be associated with a history of trauma. Immobilization is the treatment for early Kienbock disease, prior to lunate collapse, after which surgical intervention is indicated.

     

    Nakamura et al. compared 20 patients with Kienbocks who underwent a proximal row carpectomy (7) with those who underwent limited wrist fusion

    (13). They found that those who underwent a PRC had inferior range of motion, strength, and increased pain. The authors recommended consideration of scapho-trapezio-trapezoid (STT) fusion for patients with advanced Kienbock disease, as limited wrist fusion (STT or SC) or joint leveling procedures (radial shortening or capitate shortening) may help decompress the lunate. However, they concluded that late disease with lunate collapse may require proximal row carpectomy or total wrist fusion.

     

    Lichtman et al. reviewed a series of 38 patients who underwent silicone arthroplasty for Kienbock disease. Fourteen of 20 patients had satisfactory result when the surgery was done before the lunate collapsed. The authors stressed early diagnosis and early surgical intervention.

     

    Condit et al. reviewed 23 patients who underwent STT fusion or radial shortening osteotomy for Kienbock disease. They found that when the scaphoid was flexed (Lichtman stage IIIB), no patient had a good result regardless of surgical procedure. In general, outcomes in the radial shortening cohort were superior. The authors stressed consideration of radioscaphoid angle as preoperative predictor of surgical success.

     

    Goldfarb et al. evaluated wrist radiographs from patients with Kienbock disease. They found that the interobserver reliability of the Lichtman classification was generally very high (k=0.63). However, that for stage IIIA (lunate collapse without scaphoid flexion) was poor (k=0.38). The authors stressed the difference in IIIA and IIIB (lunate collapse) is the presence of fixed scaphoid flexion in IIIB.

     

    Figures:

    Figure A is an AP X-Ray of a patient with Kienbock disease, Lichtman stage IIIB. The lunate is collapsed and there is a loss of carpal height secondary to

    proximal capitate migration.

    Figure B is a lateral X-Ray of a patient with Kienbock disease, Lichtman stage IIIB, demonstrating a flexed scaphoid.

     

    Incorrect answers:

    Answer 1: Ulnar negative variance has been postulated as a cause which may put additional pressure on the lunate, leading to avascular necrosis.

    Answer 2: High interosseous pressure, particularly greater than 40mmHg as compared to the capitate, may predispose to avascular necrosis.

    Answer 3: A number of medical conditions may lead to avascular necrosis. Answer 4: A decreased radial inclination has been associated with lunate avascular necrosis by possibly increasing radio-lunate contact pressure

     

     

     

  51. A 20-year-old male military recruit slams his index finger on a tank hatch and sustains the injury seen in Figure A. An attempt at reduction and immobilization is made in the field by his unit physician assistant, and he returns to your office one week later. Indications to treat proximal phalanx fractures operatively include all of the following EXCEPT:

     

     

     

    1. Rotational deformity

    2. >2mm shortening

    3. 20° apex volar angulation

    4. 5° apex dorsal angulation

    5. Grossly contaminated open fracture

    CORRECT ANSWER: 4

    The patient has sustained an oblique proximal phalanx fracture. All of the above are operative indications with the exception of 5° apex dorsal angulation.

     

    Phalanx fractures account for 10% of all fractures and occur most frequently in young men while playing sports. These are generally treated with immobilization (buddy taping) followed by range of motion exercises.

    However, indications for surgery include failed conservative management secondary to any rotational deformity, >2mm of shortening, >10° angulation, open fractures, or displaced intra-articular fractures. For every 1mm shortening the PIPJ will develop a 12° extensor lag. Also, a 16° apex volar malunion is expected to produce a 10° extensor lag. Apex volar deformities are the most common displacement pattern for transverse proximal phalanx fractures secondary to the pull of the central slip on the distal fragment and the interossei on the proximal fragment. As such, apex dorsal angulation for proximal phalanx fracture would be atypical.

     

    Henry comprehensively reviewed proximal phalanx fractures and preferred methods of stabilization. He noted that closed management should always be considered first, even for unicondylar fractures or proximal phalangeal head fractures. However, he held that bicondylar proximal fractures or comminuted shaft fractures may require plate fixation, transverse or short oblique may be treated with pin fixation, and long oblique fractures are often amenable to fixation with multiple lag screws.

     

    Faruqui et al. compared 100 patients with proximal phalangeal base fractures, 50 of which received transarticular pinning and 50 treated with extra-articular pinning. The authors found that one-half of the patients in both groups experienced loss of flexion (average 20°) at the PIPJ. In addition, a third of both groups had a flexion contracture (average 15°). They found no difference in outcomes between the groups. The authors noted that closed pinning minimizes soft tissue disruption and neither method is necessarily superior.

     

    Figure A is an AP radiograph of a long oblique proximal phalangeal fracture. These are prone to shortening and rotational deformity.

    Incorrect answers:

    Answer 1: Rotational deformity are poorly tolerated and should be corrected with fixation.

    Answer 2: Fracture shortening by >2mm shortening would be an operative indication to prevent PIPJ extensor lag.

    Answer 3: Fracture angulation >10° apex volar angulation would be an operative indication to prevent PIPJ extensor lag.

    Answer 5: Grossly contaminated open fractures necessitate operative management for debridement at which time the fracture should be stabilized.

     

     

     

  52. A 67-year-old man presents with aseptic loosening of a primary total knee arthroplasty (TKA). He has a history of a prior tibial plateau fracture which was treated with open reduction and internal fixation via an anterolateral approach and subsequently was converted to a primary posterior-stabilized TKA utilizing a standard midline, medial parapatellar approach. He is scheduled for revision TKA and the more medial incision is used, leaving a small lateral skin bridge. He presents a few weeks later with wound necrosis and after multiple rounds of debridement and negative pressure wound therapy, he is left with the lateral defect seen in Figure A. Which of the following surgical options would best address his defect?

     

     

     

     

    1. Gastrocnemius flap

    2. Latissimus dorsi free flap

    3. Gracilis free flap

    4. Split-thickness skin graft

    5. Full-thickness skin graft CORRECT ANSWER: 1

    A gastrocnemius flap would be best suited for covering a the proximal-third tibial defect in the setting of a total knee arthroplasty (TKA) with a large soft-tissue defect like that presented in the stem.

     

    In the setting of soft tissue defects, the reconstructive ladder must be considered. This begins with the simplest method of wound closure (i.e. primary closure) and ascends up the ladder to include secondary healing, negative-pressure wound therapy, skin grafting, local flaps, and finally free flaps. When the soft tissue defect about the proximal third of the tibia exposes deep tissues including tendon, bone, or prosthesis, reconstruction should begin with a local flap (i.e. gastrocnemius flap). For wound defects up to 15 cm from the knee joint, both medial and lateral gastrocnemius local flaps may be utilized, although the lateral gastrocnemius is generally smaller and has less excursion. In instances where additional length is required, the surgeon may perform fascial pie-crusting or even release the GC proximally from the femoral condyle and create an island flap, adding an additional 5cm of length.

     

    Houdek et al. reviewed the outcomes following gastrocnemius flaps for coverage of soft-tissue defects about TKA. The authors reported a 10-year revision and amputation-free survival rate after gastrocnemius flap coverage of 68% and 79%, respectively. However they found that the risk of implant failure was increased by morbid obesity and a history of 5 or more prior surgical procedures about the knee. The authors concluded that the gastrocnemius flap reliably provides coverage of components used in complex knee reconstruction.

     

    Busfield et al. published their experience with the medial gastrocnemius flap technique for reconstruction of the chronically disrupted extensor mechanism in patients with and without TKA after failed extensor mechanism allograft reconstruction, infection, or with a compromised soft tissue envelope. They reported that all patients in their cohort regained sufficient extensor mechanism strength to return to independent ambulation, and all infections resolved after treatment. This lead the authors to advocate for the gastrocnemius rotational flap for salvage of failed extensor mechanism allograft and as an alternative to allograft reconstruction in patients with poor soft tissue coverage, previous infection, or immunocompromise.

     

    Figure A demonstrates a significant lateral soft tissue defect over the proximal third of the tibia in the setting of a TKA, with exposed bone and prosthesis.

    Illustration A is a schematic of the gastrocnemius flap, with the medial sural artery as the arterial supply.

    Illustration B depicts a representative reconstructive ladder which may be used to guide reconstructive options.

    Incorrect Answers:

    Answer 2: While a latissimus dorsi free flap is an alternative, there is associated donor site morbidity and it is further up the reconstruction ladder. A gastrocnemius flap should be attempted first.

    Answer 3: While a gracilis free flap is an alternative, there is associated donor site morbidity with this as well and it too is further up the reconstruction ladder than a local rotational flap.

    Answer 4: A split-thickness skin graft would be inappropriate for a deep wound defect like that depicted in Figure A.

    Answer 5: A full-thickness skin graft would also be inappropriate for a deep wound defect with exposed prosthesis like that depicted in Figure A.

     

     

     

     

     

     

  53. A 25-year-old soldier fell during combat training and sustained the injury seen in Figure A. Operative and non-operative management were discussed with the patient who elected for non-operative treatment. Which of the following should be immobilized for optimal management of the injury?

     

     

     

    1. Thumb, wrist, and elbow

    2. Wrist and elbow

    3. Thumb and wrist

    4. Wrist

    5. Immobilization is not necessary

    CORRECT ANSWER: 4

    Cast immobilization of the wrist alone is necessary and sufficient for stabilizing scaphoid fractures. Additional stabilization of adjacent joints does not add any benefit but instead only results in undue stiffness.

     

    Over 75% of the scaphoid is covered in articular cartilage. In addition, the vascularity is poor, as the scaphoid is perfused in a retrograde fashion from the most dorsal-radial aspect of the bone. Minimally-displaced scaphoid waist fractures may be treated with either internal fixation or non-operatively with prolonged immobilization. Non-operative management for scaphoid waist fractures involves prolonged cast immobilization, often for a minimum of 3-4 months. The specific type of cast immobilization has been investigated, and immobilization of the wrist alone has been shown to provide sufficient stability.

     

    Kawamura and Chung comprehensively reviewed treatment of scaphoid fractures and nonunions. Despite improved fixation methods, the nonunion rate after operative fixation remains 10%. Scaphoid fractures may

    demonstrate a humpback deformity if presenting in a delayed fashion. Casting may be appropriate for minimally displaced fractures but requires 8-12 weeks of immobilization. Union rate after immobilization is high (88-95%) so long as treatment is started within three weeks. Scaphoid fractures that undergo internal fixation, as opposed to casting, have better early grip strength, but higher complications. The authors favored surgical fixation for displaced fractures.

     

    Schramm et al. performed a cadaveric study testing the stability of various immobilization techniques. The authors assessed the angulation of the scaphoid fractures after each immobilization technique was stressed biomechanically. They found no significant difference in fracture angulation or rotation between thumb spica and short arm casts. However casting was clearly superior to no immobilization. The authors determined that wrist immobilization alone was sufficient in preventing fracture displacement.

     

    Buijze et al. performed a prospective trial which randomized 62 patients with scaphoid fractures to below-elbow casts including or excluding the thumb. A CT scan was obtained at 10 weeks on all patients. The authors found that while the overall union rate was uniformly high (98%), at 10 weeks the extent of union was paradoxically higher in the wrist-only immobilization cohort (85% vs. 70 %). There were otherwise no significant differences in outcomes between the groups in terms of Mayo modified wrist score, Disabilities of the arm, shoulder, and hand score, or pain. The authors contend that immobilization of the thumb is cumbersome for the patient and unnecessary for non-displaced or minimally displaced scaphoid waist fractures.

     

    Doornberg et al. performed a meta-analysis including 523 patients comparing above versus below-elbow casting for acute scaphoid fractures. The authors found no significant differences in any of the outcome measures including union rate, grip strength, time to union, or osteonecrosis based on the extent of casting or position of the wrist within the cast, concluding that any type of immobilization was sufficient so long as the wrist was immobilized.

    Figure A is an AP radiograph with a non-displaced scaphoid waist fracture. Incorrect answers:

    Answer 1: Including the thumb and elbow in the cast is unnecessary in the

    setting of conservative management of a scaphoid fracture.

    Answer 2: There is no difference between above- and below-elbow immobilization in terms of outcomes following conservative management of scaphoid fractures. Inclusion of the elbow only risks additional undue stiffness. Answer 3: Inclusion of the thumb with a spica component has not shown any

    benefit toward improved stability in immobilization of scaphoid waist fractures. Answer 5: Cast immobilization is appropriate so long as the wrist is immobilized. The position of the wrist has additionally not been shown to impact outcomes following conservative management of scaphoid fractures.

     

     

     

  54. A 3-year-old child presents with a painless mass on his right hand which has been present for 3 months. Per the parents, the mass does not seem to be changing in size. The child has an unremarkable birth and developmental history. He plays happily in the office and does not seem to be bothered by the mass. The non-tender mass is overlying the palmar aspect of the base of the right index finger and is approximately 8x8mm in size (Figure A). The parents are very concerned about the presence of the mass. An ultrasound is shown in Figure B. How should you counsel the parents?

     

     

     

     

     

     

    1. Ultrasound-guided biopsy is needed next to identify a diagnosis

    2. Excisional biopsy is needed to remove the benign mass

    3. MRI of the left hand is needed next to identify the diagnosis

    4. Laboratory work-up to include chromosomal-breakage analysis is needed next to identify the diagnosis

    5. This benign mass will likely resolve after a period of observation

    CORRECT ANSWER: 5

    This patient presents with a volar ganglion cyst of the tendon sheath. The majority will resolve spontaneously with observation, therefore further workup or management is unnecessary.

     

    While ganglion cysts of the wrist and hand are most often dorsal in adults,

    volar ganglion cysts are more common in children. In addition, children have a higher female-to-male prevalence. Observation and splinting is the initial treatment of choice, since nearly 80% of these benign masses resolve without surgical intervention in children. Aspiration is often considered to avoid taking a child to the operating room. However, recurrence rates vary widely between 13-85%. Recurrence after aspiration in children is especially high in situations such as this when the ganglion cyst arises off of a tendon. Recurrence after surgical excision in children varies by report (6-35%).

     

    Wang and Hutchinson followed 14 children under the age of 10 with ganglia about the hand or wrist. The average age was 38 months (2 months to 9 years). At presentation, the masses were present for an average of 3 months. No ganglion was painful. At 33-month follow-up, 79% resolved spontaneously. The authors advocated for observation in children presenting with a characteristic history, as well as clinical and radiographic findings consistent with a benign ganglion of the hand.

     

    Coffey et al reviewed 48 children aged 12 years and younger with a hand or wrist ganglion cyst treated at their institution. The authors found that patients were mostly female (1.8:1) and that volar cysts were most common (1.2:1). Ganglions arising from tendons had a higher incidence (33%) than previously reported. The authors recommend observation initially, as only a minority will require surgery. Should symptomatic ganglion cysts persist, surgical excision is recommended.

     

    Figure A is a clinical photograph of a volar ganglion cyst within the palm of a child. Figure B is an ultrasound of child’s palm demonstrating hypo-echogenicity of the mass, consistent with a benign, fluid-filled ganglion cyst.

     

    Incorrect answers:

    Answer 1: An ultrasound-guided biopsy is not required to make a diagnosis for a ganglion cyst.

    Answer 2: Excisional biopsy is not required since the vast majority resolve without incident. This patient has furthermore not exhausted conservative management.

    Answer 3: Further advanced imaging is not required since the history, clinical exam, and ultrasound are all consistent with a benign mass which often resolves spontaneously.

    Answer 4: A laboratory analysis with chromosomal breakage test would be appropriate for the work-up of Fanconi’s anemia. Fanconi’s anemia is associated with longitudinal deficiency of the radius and aplastic anemia.

     

     

  55. A 26-year-old male construction worker presents with a six-month history of paresthesias in the small and ring fingers. Physical examination reveals weakness of the first dorsal interosseous muscle. An MRI is demonstrated in Figure A. Which additional finding is characteristic of this pathology?

     

     

     

     

    1. Abnormal sensation over the dorsal ulnar hand

    2. Wartenberg syndrome

    3. Inability to flex the thumb interphalangeal joint (IPJ) without flexing the distal IPJ joint of the index finger

    4. Thumb and index finger IPJ flexion when attempting to pinch a piece of paper

    5. Inability to flex both the thumb IPJ and index finger IPJ

    CORRECT ANSWER: 4

    The patient has a ganglion cyst within Guyon's canal resulting in compressive ulnar neuropathy. As a result, distal motor function is impaired, leading to a positive Froment's sign, or obligatory thumb and index finger IPJ flexion to compensate for weakness of the adductor pollicis.

     

    Guyon’s canal is bordered by the transverse carpal ligament (floor), the volar carpal ligament (roof), the pisiform and abductor digiti minimi muscle (ulnarly), and the hook of the hamate (radially). Guyon’s canal is comprised of three zones. Zone 1 is proximal to the bifurcation of the ulnar nerve, and a mass here may cause both motor and sensory abnormalities to the volar aspect of the 4th and 5th digits. Zone 2 surrounds the deep motor branch and will cause isolated motor symptoms. Lastly, zone 3 is further distal, surrounding the superficial sensory branch, in which compression would only

    cause the aforementioned volar sensory symptoms. A cyst in Guyon's canal may cause motor, sensory, or mixed motor-sensory ulnar nerve symptoms.

     

    Kenny and Hammert reviewed the physical exam of the hand. The authors underscore that ulnar nerve compression within Guyon's canal will not cause dorsal ulnar hand numbness, as would be the case with more proximal compression such as in cubital tunnel syndrome.

     

    Murata et al. performed a retrospective review of 31 cases of ulnar tunnel syndrome. These were characterized as idiopathic (14) or due to trauma (8), thrombosis of the ulnar artery (2), proliferative synovium (2), hook of the hamate fracture (2), aberrant fibrous band (1), or a ganglion cyst (1). The authors found no definite relationship to carpal tunnel syndrome. They found that all cases improved after surgery.

     

    Chen and Tsai comprehensively review ulnar tunnel syndrome. The authors highlight the improvement in diagnostic accuracy, which is due in part to advanced imaging and electrodiagnostics. They emphasize that ganglion cysts, tumors, anomalous muscles, ulnar artery pathology, fractures, and injury during wrist arthroscopy may all potentiate ulnar nerve symptoms within Guyon's canal.

     

    Figure A is an axial cut, T2 sequence, of a wrist MRI demonstrating a ganglion cyst within Guyon's canal.

    Illustration A demonstrates the anatomy of Guyon's canal.

     

    Incorrect Answers:

    Answer 1: The dorsal cutaneous branch of the ulnar nerve, which branches off from the ulnar nerve 5 cm proximal to the ulnar styloid, innervates the dorsal ulnar hand. Abnormal sensation on the dorsal hand would be caused from cubital tunnel syndrome. Conversely, volar paresthesias of the ulna one-and-a-half digits may be seen with ulnar tunnel syndrome.

    Answer 2: Wartenberg syndrome, or cheiralgia paraesthetica, is caused by entrapment of the superficial sensory branch of the radial nerve. Wartenberg sign is ulnar drift or abduction of the small finger due to interossei weakness (ulnar nerve) and unopposed action of the extensor digiti quinti (PIN).

    Answer 3: Limburg-Comstock syndrome is caused by an anomalous tendinous connection between the flexor pollicis longus and the flexor digitorium profundus of the index finger. This has been reported as frequently as 20% of the population and is diagnosed when a patient is unable to independently flex the thumb IPJ and index finger DIPJ.

    Answer 5: Parsonage-Turner Syndrome is caused typically by viral brachial neuritis and can be associated with shoulder pain, and an inability to flex both

    the thumb IPJ and index finger IPJ.

     

     

     

     

     

     

  56. A 76-year-old female has been seen in your office on multiple occasions. She has persistent pain and discomfort when pinching with her left thumb. In addition, she has a 20-degree extension deformity of her ipsilateral MCPJ. She has received injections and has attempted nighttime splinting for 3 months. Due to recalcitrant symptoms, she elects for surgical management. You discuss with her the risks and benefits of various surgical options. In addition to MCPJ stabilization, you discuss treatment options for her CMCJ. Specifically, with regard to the benefit of ligament reconstruction and tendon interposition (LRTI) as compared to trapeziectomy, you council her that:

     

     

     

    1. LRTI offers improved pain relief

    2. LRTI offers improved range of motion

    3. LRTI offers decreased symptomatic metacarpal subsidence

    4. LRTI offers improved subjective patient-centered outcome scores

    5. No benefit of LRTI over trapeziectomy

    CORRECT ANSWER: 5

    This patient presents with basilar thumb arthritis. In regard to the variety of possible surgical treatments, LRTI has shown no significant advantages over trapeziectomy alone.

     

    Basilar thumb arthritis is present in 25% of men and 40% of women >75 years old and is more prevalent in Caucasians. The anterior oblique ligament becomes attenuated which leads to carpometacarpal joint instability, subluxation of the base of the metacarpal, and development of arthritis. While the arthritis itself is often asymptomatic, patients with persistent discomfort with pinching maneuvers are typically managed initially with night splints and at times, injections. When non-operative management fails, there are several surgical options to include trapeziectomy, trapeziectomy with tendon interposition (FCR or PL), trapeziectomy with pinning (hematoma arthroplasty), trapeziectomy with tight rope, metacarpal base arthroplasty,

    and trapeziectomy with trapezium implant. Despite the number of different techniques, literature to date has not shown any definitive benefit of any of these specific technique over trapeziectomy alone.

     

    Kriegs-Au et al. performed a randomized comparative study evaluating the difference in outcomes between trapeziectomy and LRTI. At 48 month followup, the authors found that trapeziectomy alone had statistically improved subjective outcome scores, better palmar and radial abduction, cosmetic appearance, and willingness to undergo the surgery again. The groups otherwise demonstrated similar overall satisfaction, grip strength, pain, and dexterity, with no difference in proximal metacarpal migration. The authors concluded that tendon interposition is not necessary and that migration did not affect outcomes.

     

    Naram et al performed a review of 179 patients who underwent trapeziectomy or LRTI. Total compilations were greater in patients undergoing LRTI as compared to trapeziectomy (Odds ratio: 4.3). The authors concluded that a trapeziectomy is favored for basilar thumb arthritis for the lower complication profile.

     

    Li et al. performed a systematic review comparing LRTI vs. trapeziectomy. There were differences in grip strength, pinch strength, pain, DASH, or number of adverse events. The authors concluded that LRTI is not superior to trapeziectomy alone.

     

    Qadir et al. reviewed a surgical technique of volar capsulodesis and soft tissue stabilization of hyper-extended MCPJs in 14 patients undergoing surgery for concomitant basilar thumb arthritis. All patients had complete resolution of hyperextension and only one patient reported pain.

     

    Davis and Pace randomized patients to LRTI+k-wire vs. Trapezium. Pain relief, DASH, pinch/grip strength did not differ at 3 or 12 months.

     

    Figure A is a radiograph of the hand demonstrating advanced basilar thumb arthritis. Radially there are large osteophytes and ulnarly, there is a large osteophyte which protrudes into the intermetacarpal space. In addition, the 1st CMCJ is subluxated radially, forcing the metacarpal into adduction.

     

    Incorrect Answers:

    Answer 1: LRTI has not been shown to have improved pain relief over trapeziectomy.

    Answer 2: LRTI has not been found to result in improved range of motion over trapeziectomy.

    Answer 3: LRTI and trapeziectomy have similar outcomes in terms of similar radiographic subsidence of the metacarpal. This has furthermore been shown to be largely asymptomatic.

    Answer 4: LRTI and trapeziectomy have been found to result in comparable patient centered outcomes, with no significant benefit to LRTI.

     

     

     

  57. A 34-year-old right-hand-dominant squash player falls onto an outstretched hand during a qualifying match. Since this fateful match, he has had ulnar-sided wrist pain, decreased range of motion, and periodic clicking in his wrist. The physical exam finding in Figure A is noted, in which you are able to manipulate and easily translate the ulna against the radius. This finding is not present on his contralateral wrist. The deep portion of the injured ligamentous structure inserts on what anatomical landmark?

     

     

     

     

    1. Ulnar styloid

    2. Ulnar fovea

    3. Dorsal radio-ulnar capsule

    4. Dorsal oblique band of the interosseous membrane

    5. Radial styloid CORRECT ANSWER: 2

    The patient has a distal radio-ulnar joint (DRUJ) injury with concordant pain, instability, loss of motion, and a positive piano key sign. The deep portion of the radio-ulnar ligament attaches to the ulnar fovea.

     

    The triangular fibrocartilage complex (TFCC) is a key stabilizer of the DRUJ. This complex is comprised primarily of an articular fibrocartilaginous disk as

    well dorsal and volar radio-ulnar (RU) ligaments, which are each composed of superficial and deep fibers. In addition, the extensor carpi ulnaris (ECU) subsheath contributes fibers, and the volar portion of the RU ligament gives rise to the ultotriquetral and ulnolunate ligaments. In the setting of DRUJ injury, radiographs are most often unremarkable but may demonstrate a widened DRUJ or dorsal displacement of the ulna relative to the radius on a true lateral view. An MRI is recommended to better delineate the extent of soft tissue injury. So long as the DRUJ is reducible and stable, immobilization is usually sufficient. However, in the setting of acute TFCC injury with dorsal or dorsal-ulnar tears and DRUJ instability, repair of the deep portion of the RU ligaments (as opposed to the superficial RU ligaments) is advocated, as these are critical to stabilizing the DRUJ.

     

    Stuart et al. performed a biomechanical analysis on cadaveric DRUJs. The authors found the primary check against volar subluxation was the dorsal RU ligaments and the primary restraint against dorsal subluxation was the volar RU ligaments. The ulnocarpal ligaments and ECU subsheath did not contribute to DRUJ stability. They additionally found that 20% of the DRUJ constraint was provided by the articular contact of the radius and ulna within the sigmoid notch.

     

    Moritomo described the distal interosseous membrane (DIOM) anatomy and biomechanics. The DIOM contains a distal oblique band (DOB) which originates at the distal one-sixth of the ulna and inserts on the inferior rim of the sigmoid notch of the radius. They discussed that the DOB contributes to DRUJ stability and was an isometric stabilizer of the forearm during pronosupination. As a result, the authors noted that ulnar shortening with the osteotomy performed proximal to the attachment of the DIOM better preserved stability of the DRUJ.

     

    Mulford and Axelrod comprehensively reviewed traumatic injuries of the DRUJ and describe a step-wise approach to treatment. For acute injuries with mild instability, the authors recommend immobilization in the position of stability for 4-6 weeks. For complex injuries that are irreducible, open reduction and stabilization is required. For complex, reducible injuries with an ulnar styloid fracture, the authors recommend open reduction and internal fixation. Without an ulnar styloid fracture, the authors recommend TFCC repair, pinning the reduced DRUJ, and immobilization for 4-6 weeks.

     

    Abe et al. reviewed 29 patients who underwent open or arthroscopic repair of foveal tears of the TFCC. The groups did not differ in pain, ROM, stability, or subjective outcome scores. The authors found that operative time was significantly lower in the arthroscopic group but upheld that either technique is efficacious so long as the TFCC is secured to the ulnar fovea.

    Figure A demonstrates a piano key test. In this case the physician is able to translate the ulna, with the radius stabilized, in an AP direction. Illustration A depicts the anatomy of the DRUJ.

     

    Incorrect answers:

    Answer 1: The ulnar styloid serves as the insertion point for the superficial radio-ulnar ligaments.

    Answer 3: The dorsal radio-ulnar capsule surrounds the DRUJ but is not the insertion point for the deep fibers of the radio-ulnar ligaments.

    Answer 4: The dorsal oblique band (DOB) of the IOM inserts on the dorsal-ulnar corner of the distal radius and is a secondary stabilizer of the DRUJ. The DOB can be a deforming force for this fragment in certain distal radius fracture patterns.

    Answer 5: The radial styloid does not participate in the stability of the DRUJ/TFCC.

     

     

     

     

     

     

  58. A 32-year-old parkour enthusiast presents after tumbling 6 months ago. He has had persistent left wrist pain since the injury. Examination today reveals a positive ballottement test, dorsal and ulnar carpal tenderness, and a painful snap with ulnar deviation, pronation, and axial compression of the wrist. His images are seen in Figures A and B. Which portion of the injured ligament is strongest?

     

     

     

     

     

     

    1. Dorsal fibers of scapholunate ligament

    2. Volar fibers of scapholunate ligament

    3. Dorsal fibers of lunotriquetral ligament

    4. Volar fibers of lunotriquetral ligament

    5. Short radiolunate ligament

    CORRECT ANSWER: 4

    The clinical presentation and imaging are consistent with a chronic lunotriquetral (LT) ligament injury. In contrast to the scapholunate (SL) ligament, the volar fibers of the LT ligament are the most robust.

     

    LT ligament tears present with ulnar-sided wrist pain generally after a fall and are notoriously difficult to diagnose. Physical exam can help delineate a LT disruption from more common causes of ulnar sided wrist pain. The LT shuck test is performed by moving the lunate in a palmar/dorsal direction between one’s thumb and index finger. A positive test would elicit pain and clicking. The Kleinman’s shear test involves loading the triquetrium in the dorsal/palmar plane to produce a shear across the LT articulation. A positive test would be noted in the presence of pain or a clunk. In addition to these exam findings, when the LT ligament is disrupted, the scaphoid’s influence on lunate position is unchecked, and the lunate gradually flexes with the scaphoid. This leads to volar intercalated segmental instability (VISI) posture of the wrist (seen in Figure A). While the normal SL angle is on average 47°, a VISI wrist will have a SL angle of <30°. In addition, on an AP X-Ray of a patient with a LT disruption, the triquetrium may migrate proximally, especially in ulnar inclination. The LT ligament has two transverse components (dorsal and palmar) and a fibrocartilaginous membrane proximally. Similarly, the SL ligament has a dorsal, palmar, and interosseous components.

     

    Berger reviewed the yield strength of the SL and LT ligaments of the wrist. He found that the dorsal aspect of the SL ligament (260 Newtons) was stronger than the volar aspect of the SL ligament (118N). In contrast, the LT ligament is thicker and stronger volarly (301 N) as compared to dorsally (121N).

     

    Atkinson and Watson comprehensively review the management options for LT tears. The authors found that cast immobilization was more successful when the injury was treated in the acute setting, and only a minority (37%) of cases were treated surgically. They note that arthroscopic debridement, direct repair, arthrodesis, capsulodesis, and ligament reconstruction are all surgical options. The authors highlight the need for objective, standard definition of complete tear and LT instability.

     

    Figures and Illustrations:

    Figure A is a lateral radiographic view of a wrist with a VISI posture, demonstrating a flexed lunate with a decreased SL angle.

    Figure B is an AP clenched fist view of the same wrist showing an incongruent LT articulation of the left wrist.

     

    Illustration A is a lateral radiograph of a patient with VISI posture demonstrating how to measure the SL angle between the lunate axis and the

    scaphoid axis.

     

    Incorrect Answers:

    Answer 1: The dorsal fibers of the SL ligament is the stronger portion of the SL ligament, which is not involved in this patient. A SL tear would result in different clinical exam findings, and radiographs with a widened SL interval and DISI posture.

    Answer 2: The volar fibers of the SL ligament are the weaker portion of the SL ligament and is similarly not involved in this presentation.

    Answer 3: Although the LT is involved in this case, the DORSAL aspect is the WEAKER portion of the LT ligament.

    Answer 5. The short radiolunate ligament remains attached to the lunate in perilunate dislocations, but is not involved with LT disruption.

     

     

     

     

     

     

  59. A 21-year-old right hand dominant male was slicing an avocado in his New York City apartment when he lacerated his left ring finger. At the time of injury he was unable to flex the interphalangeal joints of the finger. He underwent surgical repair of the injured flexor tendon, which was shortened by 12 mm, but repaired with 8 core-suture in a cruciate pattern. He was placed on an early-active range of motion protocol and quickly regained his motion. Three months after repair, the patient continues to complain of difficulty actively bringing his middle and small finger into his palm as seen in Figure A. He otherwise has full passive motion. What factor is precipitating the discrepancy between his active and passive motion?

     

     

     

    1. A common flexor muscle belly

    2. Inadequate number of core suture repair

    3. Lumbrical plus deformity

    4. Unrecognized anterior interosseous nerve injury

    5. Inadequate intrinsic muscle function

    CORRECT ANSWER: 1

    This patient presents with quadriga, resulting from flexor tendon repair with effective tendon shortening greater than 1cm. Because the flexor digitorum profundus (FDP) tendons share a common muscle belly, shortening one of the tendons will limit the flexion of the adjacent fingers to that of the affected finger.

     

    Quadriga is a described complication of flexor tendon repair with over-advancement of the profundus tendon. Other causes include flexor tendon adhesions, retraction of the FDP, or “over the top” FDP repair of the distal phalanx after revision amputation. Treatment may include observation or FDP release of the affected digit. The latter would remove the repaired tendon from its tether to the adjacent tendons, allowing full active flexion of the unaffected digits.

     

    Neu et al. reviewed 20 patients with quadriga, most of which were reportedly due to adhesions following finger crush injuries. The patients were ultimately treated with FDP release. The authors reported that 85% of patients had resolution of quadriga after release.

     

    Malerich et al. performed a cadaveric analysis of flexor tendon advancement. The authors evaluated flexion lag of the adjacent digits while the repaired

    profundus tendon was progressively shortened. At 1cm, the extension loss in the adjacent fingers was not deemed significant. However, at 1.5cm of advancement, the ulnar three digits lost 40° of flexion. This data formed the foundation of the tenet of not advancing flexor tendon repairs greater than 1cm during repair.

     

    Schreuders et al. presented a comprehensive review article on the anatomy and biomechanics of quadriga. The authors highlight the etiology of the condition, discussing that it stems from the interconnection of the FDP tendons as well as the common muscle belly of the profundus. They note that this ultimately makes the FDP tendons less capable of independent movement as compared to the FDS tendons. The authors conclude that an understanding of the phenomenon is paramount to proper clinical assessment, treatment, and post-operative rehabilitation.

     

    Figures:

    Figure A demonstrates a clinical photograph of a patient with quadriga attempting ot make a full composite first. As a result of the shorted FDP to the ring finger, the adjacent fingers (middle and small) cannot be tensioned enough to actively flex completely.

     

    Incorrect answers:

    Answer 2: The patient underwent an adequate repair. If the repair had failed, it would not be due to the number of core sutures, and the patient would not actively able to flex his ring finger distal interphalangeal joint as is seen in Figure A.

    Answer 3: A lumbrical plus deformity is often seen as the converse to quadrigia. This may occur when a revision amputation is shortened at or proximal to the level of the distal interphalangeal joint. As a result, the FDP tendon is detached. Since the lumbrical originates on the FDP, without a distal FDP insertion, attempted active flexion of the digit will result in activation of the lumbrical, leading to the paradoxical extension of the proximal interphalangeal joint.

    Answer 4: An injury to the anterior interosseous nerve would result in an inability to flex the interphalangeal joint of the thumb and distal interphalangeal joint of the index finger.

    Answer 5: Intrinsic muscle function may be impaired in the setting of a crush or ulnar nerve injury but would result in extension at the MCPJ and flexion at the PIPJ, or an intrinsic-minus posture of the hand.

     

     

  60. A 67-year-old woman presents with chronic insidiously progressive right wrist pain. Her history is only significant for a remote fall onto her wrist 25 years prior. A radiograph of her affected wrist is shown in Figure A. After extensive discussion, she elects to undergo a salvage motion-sparing procedure that relies on the intact cartilage of the capitate head. During the dissection, the most volar-radial ligament is excised. What is her anticipated outcome?

     

     

     

     

    1. Improved pain and increased range of motion by 3 months

    2. Ulnar translocation of the carpus

    3. Capitate avascular necrosis

    4. Distal radio-ulnar joint instability

    5. Progression of radio-styloid arthritis

    CORRECT ANSWER: 2

    The procedure described is a proximal row carpectomy (PRC), which was indicated for her scaphoid nonunion advanced collapse (SNAC). Excision of the most volar-radial ligament, or the radioscaphocapitate (RSC) ligament, compromises the stability of the radiocarpal joint and may result in ulnar translocation of the carpus.

    A PRC involves excising the scaphoid, lunate, and triquetrum and relies on preserved articular cartilage of the proximal aspect of the capitate to articulate with the radius. The stability of the carpus after a PRC is dependent on preservation of the RSC ligament. The RSC ligament is a stout carpal stabilizing structure which runs in an oblique fashion from the radial styloid to the carpus on the volar aspect of the wrist. The RSC is radial to both the long and short radiolunate ligaments and essential to preventing ulnar translocation.

     

    Green et al. presented a current concepts review of PRC. The authors delinate the key factors to success of the PRC: intact cartilage of the capitate and lunate fossa of the radius and preservation of the RSC ligament. The authors describe how the RSC serves as a check-reign to ulnar translation of the carpus after a PRC.

     

    van Kooten et al. review a series of four complex fracture dislocations of the wrist that were treated with early PRC. Three patients had an intact RSC and had good outcomes. However, one patient had a severely damaged RSC ligament in addition to his carpal fracture dislocation. At three month follow-up his capitate was subluxed in a volar and ulnar direction, resulting in a painful and stiff wrist. The patient ultimately required a wrist arthrodesis.

     

    Figure A is an AP radiograph of the right wrist demonstrating SNAC wrist with preservation of the capitolunate joint and lunate facet. Illustration A is an intraoperative view of the RSC, running in an oblique fashion, tethering the carpus to the radial aspect of the wrist, preventing ulnar translocation.

     

    Incorrect answers:

    Answer 1: The PRC is a salvage procedure which generally offers pain relief and motion preservation in an arthritic wrist. However, the procedure relies on an intact RSC. Inadvertent release of the RSC would destabilize the wrist and ulnar translocation of the carpus would ensue. As in the van Kooten series, patients undergoing a PRC with a transected RSC would have an exceptionally poor outcome.

    Answer 3: Capitate avascular necrosis is not a complication of a PRC. Capitate avascular necrosis would be a counter-indication for a PRC as a PRC relies on preserved articular cartilage to articulate with the radius.

    Answer 4: Distal radio-ulnar joint instability may be precipitated in a Galeazzi fracture pattern, particular when the radial shaft fracture is within 7.5cm from the joint. The DRUJ would not be affected by transection of the RSC ligament. Answer 5: Progression of radio-styloid arthritis is a sequalae of SNAC wrist.

    However, a PRC would be the treatment for this development.

     

     

     

     

     

  61. A 52-year-old farmer’s periodic wrist pain has been managed with non-operative modalities to include two injections in the last 8 months. A recent imaging study is seen in Figure A. The patient now reports increasing pain and inability to use his wrist. The next best step in management would be:

     

     

     

    1. Scapho-capitate fusion

    2. Distal pole scaphoid excision

    3. Total wrist arthroplasty

    4. Scaphoid excision and four-corner fusion

    5. Proximal row carpectomy

    CORRECT ANSWER:

    4

    This patient has radiographic evidence of stage III scapho-lunate advanced collapse (SLAC). Scaphoid excision with four-corner fusion would be the most appropriate management of the above answers for stage III SLAC.

     

    When the scapho-lunate ligament tears, the carpal kinematics are permanently altered, and the carpal bones no longer articulate in concert. This leads to a predictable pattern of arthritic progression. Stage I involves arthritis between the scaphoid and radial styloid. Stage II shows arthritis between the scaphoid and entire scaphoid facet, and stage III shows progression of arthritis between the capitate and the lunate. In addition to these radiographic findings of degenerative arthritis, patients will have a flexed scaphoid and extended lunate – leading to a dorsal intercalated segment instability (DISI) posture.

    Acute injuries can be treated by ligament repair or reconstruction. However, in chronic cases, with advanced arthritic changes and fixed DISI posture, a salvage procedure is needed. Typically a four corner fusion (4CF) or proximal row carpectomy (PRC) are considered. A PRC relies on healthy, intact cartilage

    between the lunate facet of the distal radius and the proximal capitate. However, in SLAC III, the proximal capitate is degenerative and this procedure should not be offered.

     

    Berkhout et al. retrospectively compared 4CF (8 patients) with PRC (14 patients) after an average 17-year follow-up. The authors indicated patients with SLAC, SNAC, Kienbock’s disease, lunotriquetral syostosis, and perilunar subluxation. They found slightly improved range of motion following PRC but otherwise no differences in grip strength, patient-reported outcomes, or degenerative changes. There were more complications reported in the 4CF cohort.

     

    Wagner et al. compared the outcomes of PRC to 4CF in 89 patients with wrist arthritis, all of whom were younger than 45 years old. The authors found that there were no differences in the development of moderate to severe arthritis or the need for revision surgery at long-term follow-up. There was however a 12% non-union rate in the 4CF cohort. They found that the PRC cohort additionally had an improved flexion-extension arc and DASH scores versus the 4CF cohort, though 4CF was associated with slightly better grip strength. The authors concluded that while both are reasonable options for young patients, PRC conveyed improved motion and fewer complications.

     

    Kitay and Wolfe comprehensively review scapholunate instability. The authors note that injuries to the SL joint are the most common cause of carpal instability and precipitate a predictable pattern of arthritis from abnormal joint mechanics. They held that the goals of surgical treatment are to restore anatomy and return appropriate carpal kinematics.

     

    Watson et al. also reviewed the natural progression of scaphoid instability. The authors discussed how scaphoid instability should be viewed on a spectrum, noting that minor instability may be asymptomatic and present in up to 20% of adults. They concluded that appropriate diagnosis is dependent on a keen understanding of carpal anatomy and kinematics.

     

    Figures:

    Figure A is an AP radiographic view of a grade III SLAC wrist, demonstrating advanced arthritic changes in the capitolunate joint with a large capitate cyst.

     

    Incorrect answers:

    Answer 1: Scapho-capitate fusion is a treatment for Kienbock’s disease. The goal of the procedure is to off load the congested lunate. This is not an appropriate treatment for a SLAC wrist.

    Answer 2: Distal pole scaphoid excision, or Malerich procedure, is a salvage

    treatment for Scaphoid Non-Union Advanced Collapse. This patient does not have a scaphoid fracture.

    Answer 3: Total wrist arthroplasty may be a functional choice for an older patient with less activity demands or in a patient with rheumatoid arthritis. However, this is not a good choice in a younger farmer.

    Answer 5: Proximal row carpectomy is incorrect because this patient has advanced arthritic changes at the capito-lunate joint and a large proximal capitate cyst. A PRC would create another arthritic articulation between the capitate and lunate, and is a contraindication with advanced – stage III SLAC wrists.

     

     

     

  62. A 38-year-old right-hand-dominant computer programmer trips over his cape and falls on his right hand at a renaissance fair. He experiences immediate pain and swelling about the base of his right thumb. He presents to your office where initial radiographs are obtained (Figure A). The muscles responsible for the characteristic deforming forces on the thumb metacarpal shaft are innervated by which nerve(s)?

     

     

     

     

    1. Ulnar nerve only

    2. Ulnar and anterior interosseous nerve (AIN)

    3. Ulnar and posterior interosseous nerve (PIN)

    4. Ulnar and recurrent branch of the median nerve

    5. Recurrent branch of the median nerve and AIN

    CORRECT ANSWER: 3

    The patient presents with a fracture at the base of his thumb metacarpal. The deforming forces on the metacarpal are supination-adduction secondary to the pull of the adductor pollicis (ulnar nerve) and dorsal-radial displacement secondary to the pull of the abductor pollicis longus and extensor pollicis brevis (PIN).

     

    Thumb metacarpal base fractures comprise nearly 80% of all thumb metacarpal fractures. The Bennet fracture in particular is caused by axial loading of the thumb, resulting in a simple intra-articular fracture. The volar-ulnar corner of the metacarpal base remains in situ via its attachment to the trapezium by the anterior oblique ligament. The thumb metacarpal shaft is instead displaced by the aforementioned deforming forces. An understanding of these force facilitates reduction, which involves longitudinal traction, palmar abduction and pronation. Extra-articular fractures with less than 30° of angulation or intra-articular fractures with <1mm of articular step off can be treated non-operatively. Fragments too small for screw fixation may be percutaneously pinned, otherwise open reduction with internal fixation may be performed.

     

    Carter and Nallamouthu comprehensively reviewed Bennett fractures. The authors note that injury most commonly occurs when a partially flexed metacarpal is axial loaded. The resulting fractures are unstable owning to their deforming forces and usually require inter-fragmentary fixation. The authors report positive outcomes when addressed with anatomic fixation.

     

    Pavic and Malovic reviewed 89 Bennett fractures treated with ORIF (K-wire: 26, plate: 19, screws: 44). They noted that that percutaneous pins were removed after 5-7 weeks and found that all fractures went onto uneventful union in 4-8 weeks. The authors report no complications but recommend starting therapy as soon as possible after fixation.

     

    Rivin et al. reviewed the treatment of Bennett fractures, including tension band wiring, external fixation, and arthroscopic assisted fixation. The authors preferred the technique of closed reduction and percutaneous pinning with 2

    K-wires, one from the metacarpal base to the trapezium and the second across the metacarpal base with splint immobilization for 4 weeks.

     

    Figure A demonstrates an intra-articular thumb metacarpal base fracture. The palmar/ulnar corner is held in place by the anterior oblique ligament.

    Incorrect answers:

    Answer 1: The ulnar nerve in the hand innervates the two ulnar lumbricals, opponens digiti minimi, abductor digiti minimi, interossei and the adductor pollicis, but not the APL/EPL.

    Answer 2: While the ulnar nerve innervates one of the muscle responsible for the deformity, the AIN does not. The AIN innervates the flexor pollicis longus and flexor digitorum profundus to the radial two digits.

    Answer 4: The ulnar nerve innervates the adductor pollicis, but the recurrent median nerve innervates the thenar musculature, which is otherwise uninvolved in the deformity with a Bennett fracture.

    Answer 5: Neither the recurrent branch of the medial nerve or AIN innervate muscles responsible for the characteristic deformity.

     

     

     

  63. A 19-year-old military service-member visits your office and complains of significant wrist pain. He was able to pass basic training but had considerable discomfort with pushups. When he was a senior in highschool he hurt his wrist playing football. He underwent surgery in his hometown but never felt like he got back to his pre-injury level of function. Radiographs are provided in Figures A and B. What modification of surgical technique may have better served this fracture pattern on the index procedure?

     

     

     

    1. Achieving a more center-center screw position

    2. Using a dorsal approach with an anterograde screw

    3. Using a non-cannulated screw

    4. Using a vascularized bone transfer

    5. Using a percutaneous approach

    CORRECT ANSWER: 1

    This patient has a scaphoid fracture non-union following open reduction and internal fixation from a volar, retrograde approach. Excision of a portion of the trapezium may be necessary to obtain an appropriate trajectory for a center-center position, maximizing construct stiffness and decreasing risk of nonunion.

     

    While union rates following operative treatment of scaphoid fractures are cited at 90-95%, screw position is critical. A center-center position has been shown to provide for maximal screw length purchase, and construct stiffness, thereby providing for the highest chance of union. The screw in this case is in a suboptimal position, with the tip completely dorsal to the proximal pole (Figure B) and the starting point too radial (Figure A). As a result the screw is not in a center-center position, or centered within the scaphoid on both AP and lateral projections. A dorsal approach is not required to obtain optimal positioning of the screw, and both dorsal and volar approaches are supported. However in

    order obtain a proper start-point via a volar approach, the proximal portion of the trapezium must be excised. This allows the start point to move more ulnar and dorsal. Without excising a portion of the trapezium, the start point of the screw will be too volar and radial, as in this case.

     

    Dodds et al. assessed the biomechanical stability of scaphoid fracture fixation in a cadaveric model. The authors found less fracture motion with longer screws. They recommended the longest possible screw placed along the central axis of the scaphoid in order to attain the stiffest construct.

     

    McCallister et al. evaluated biomechanical stability of scaphoid fracture fixation in a cadaveric model. The authors similarly found that central placement of the screw provided superior fixation as opposed to eccentric screw placement.

    They advocated for central placement of screw placement and stressed that clinical efforts and techniques which facilitate central placement.

     

    Trumble et al. retrospectively reviewed 34 patients who received either a Herbert screw or a cannulated screw for an acute scaphoid fracture. The authors found no difference in clinical or radiographic outcomes. The group with a cannulated screw additionally had a shorter time to union (3.6 months vs. 7.6 months). They showed that regardless of implant, time to union was shorter when the screw was placed in the central axis. Furthermore, 17/18 cannulated screws but only 7/16 Hebert screws were placed appropriately, leading the authors to conclude that cannulated screws may be easier to place and lead to at least the same outcomes.

     

    Figure A is an AP radiograph of the wrist demonstrating a scaphoid non-union. The starting point for the screw is too radial. Figure B is a lateral radiograph of the same wrist showing that the tip of the screw is completely dorsal to the proximal pole.

     

    Incorrect answers:

    Answer 2: Scaphoid waist fractures may be treated from a volar or dorsal approach. The volar approach is typically reserved for mal-unions or those fractures with a humpback deformity. The fracture could have been treated dorsally, but the decision on approach does not constitute an improvement in surgical technique in this case.

    Answer 3: All modern scaphoid screws are cannulated and this has not demonstrated any effect on healing.

    Answer 4: Vascularized bone transfer (Medial Femoral Condyle, or MFC, for scaphoid waist non-unions; or Medial Femoral Trochlea, or MFT, for scaphoid proximal pole non-unions) are not indicated in the acute setting.

    Answer 5: A percutaneous approach may be indicated in non-displaced

    fractures but the decision on approach does not constitute an improvement in surgical technique in this case.

     

     

     

  64. A 60-year-old right hand dominant female telemarketer has long standing radial sided wrist pain, particularly when making calls. The pain is sharp in quality and is localized 1.5cm proximal to her right radial styloid. When she clasps her thumb in her palm and her wrist is forced into ulnar deviation, her symptoms are worsened. She has tried immobilization and injections which have offered only temporary relief. You offer her surgical release of her first dorsal compartment. Which of the following is true and should be considered during your release?

    1. The EPB tendon is thicker than the APL

    2. The EPB tendon is more dorsal than APL

    3. The radial artery is superficial to the APL

    4. The EPB contain often contains multiple slips which may be in separate sub-sheathes

    5. The EPL often has a separate sub-sheath within the first dorsal compartment

    CORRECT ANSWER: 2

     

    During surgical release of the first dorsal extensor compartment for de Quervain tenosynovitis, the surgeon must ensure that both the APL and EPB are completely released. Considerations include that the EPB is more dorsal and may be contained within an accessory compartment, and the APL may have multiple slips.

     

    The diagnosis of de Quervain tenosynovitis is made with history and a physical exam. The condition typically presents in the 5th and 6th decade, is six times more common in women, and can be more prevalent among lactating females. Symptoms include sharp tenderness 1-2cm proximal to the radial styloid which is made worse when the thumb is clasped in the palm and the wrist is forced into ulnar deviation (Eichhoff maneuver). De Quervain tenosynovitis should be distinguished from the less common intersection syndrome, which causes pain and crepitus proximally and more dorsally between the 1st and 2nd dorsal compartments. There are no causal links between de Quervain tenosynovitis and occupational risk factors. Advanced imaging is not necessary. Aberrant tendon slips and variations of dorsal compartment sheath anatomy contribute to symptoms. The first dorsal compartment is the site of the most numerous variations in tendon structure in the upper limb. Immobilization has a high

    failure rate except in lactating females in which non-operative management is highly effective. Corticosteroid injections have a 50-80% success rate after 1-2 injections. Surgery involves releasing the 1st dorsal compartment and all sub-sheathes. The EPB is phylogenetically young, only present in humans and gorillas, and is absent in 7% of individuals. It is thinner, more dorsal, and liable to be missed during 1st dorsal compartment release. The separate compartment for EPB raises the likelihood of failure of conservative management.

     

    Weiss et al. compared treatment of de Quervain tenosynovitis with steroid injection, immobilization, and a combination of the two. The authors found no difference in symptomatic relief between the groups, and nearly half (20/45) of the wrists went onto require surgical release. The authors advocate for a steroid/lidocaine injection alone as the initial treatment of choice, and they recommend against immobilization as they do not believe that it benefits patients.

     

    Ilyas et al. comprehensively review de Quervain disease. They note that the condition is most common in middle aged women, and that diagnosis is clinical and made on physical exam. However, the authors recommended getting radiographs to rule out other pathology. They additionally stress the importance of identifying and protecting the radial sensory nerve during surgical release.

     

    Incorrect answers:

    Answer 1: The EPB is significantly thinner than APL and may even be absent. Answer 3: The radial artery dives deep to the first dorsal compartment distal to the scaphoid on its path to the first webspace.

    Answer 4: The APL often contains multiple slips. The EPB is typically one extremely thin tendon.

    Answer 5: The EPL is contained within the third dorsal compartment.

     

     

     

     

  65. A 45-year-old male tennis player presents with ulnar-sided wrist pain. He has no foveal tenderness and otherwise has a full range of motion about his wrist. He notices a clicking sensation on the ulnar aspect of his wrist with supination and flexion of the wrist. He elects to begin with non-operative management. In what position should his wrist be immobilized to reduce the subluxating structure?

    1. Pronation, extension, ulnar deviation

    2. Pronation, extension, radial deviation

    3. Pronation, neutral, ulnar deviation

    4. Supination, extension, ulnar deviation

    5. Supination, neutral, radial deviation

    CORRECT ANSWER: 2

    The patient presents with dynamic extensor carpi ulnaris (ECU) subluxation. The position of immobilization which confers the greatest stability to the ECU if that of pronation, extension, and radial deviation of the wrist.

     

    The ECU passes through the fibro-osseous subsheath comprising the 6th dorsal extensor compartment and attaches to the base of the 5th metacarpal. ECU subluxation may result from attenuation or traumatic rupture of the ECU subsheath. The condition is more common in tennis players and is typically made worse with overuse. Patients with ECU subluxation will often report clicking during supination, flexion, and ulnar deviation. An MRI may be obtained to evaluate the position of the ECU or for tendonitis, however, a click with pathognomonic motion is diagnostic. Treatment is typically conservative with immobilization in pronation, extension, and radial deviation. If nonoperative management fails, direct subsheath repair may be performed in the acute setting or extensor retinaculum flap reconstruction may be required for chronic cases.

     

    Ghatan et al. tested ECU stability in a cadaveric model. The authors found that pronation and extension was the most stable position for the ECU, while supination and flexion was the least stable position. The authors recommended against the latter position in patients with symptomatic ECU instability.

     

    Inoue and Tamura reviewed the outcomes in 12 patients with recurrent ECU subluxations treated with repair or reconstruction of the ECU subsheath. They outlined three types of subsheath injuries: (A) ulnar-sided rupture of the fibro-osseous sheath and the torn sheath sitting dorsal to the tendon, which was treated with sheath reconstruction using a piece of extensor retinaculum; (B) radial-sided rupture of the fibro-osseous sheath and the torn sheath sitting beneath the tendon, which was treated with suture repair of the sheath; and

    (C) detachment of the periosteum from the ulnar side of the ulna which formed a false membrane into which the tendon easily dislocated, which was treated with periosteal fixation. The authors found that by this classification and respective techniques, all had a satisfactory result.

     

    MacLennan et al. reviewed outcomes of 21 patients who underwent ECU stabilization procedures. The authors found a significant improvement in ROM,

    grip strength, DASH, pain, and satisfaction after EDU subsheath reconstruction.

     

    Allende and LeViet evaluated outcomes in 28 cases of ECU pathology treated with surgery including ECU tendinopathy (15), ECU dislocation (5), ECU subluxation (4), and ECU rupture (4). The authors noted that seventeen of these patients developed symptoms while playing sports. They found that isolated ECU pathology was rare, as there were 11 TFCC injuries and 4 LT tears. At the time of final follow-up, twenty-two patients returned to sports or prior activity following surgery.

     

    Incorrect answers:

    Answer 1: Ulnar deviation may lower the threshold for ECU subluxation Answer 3: Placing the wrist in slight wrist extension will prevent subluxation Answer 4: Supination may lower the threshold for ECU subluxation

    Answer 5: Supination may lower the threshold for ECU subluxation

     

     

     

  66. A 64-year-old right–hand-dominant woman has been experiencing radial-sided right wrist pain, particularly with opening jars and pinching. On exam she has tenderness directly over her thenar eminence and swelling distally along her FCR tendon. She has worn a splint on-and-off over the past year, has had multiple cortisone injections, and has modified her activity, all of which helped initially. She wants to move forward with surgical intervention. Given her history, physical exam, and radiographs shows in Figures A and B, what surgical procedure would best alleviate her symptoms AND reduce the need for a secondary procedure?

     

     

     

    1. Scaphotrapeziotrapezoidal (STT) arthrodesis

    2. Open scaphoid distal pole resection

    3. Open scaphoid distal pole resection with silicone interpositional spacer

    4. Arthroscopic scaphoid distal pole resection

    5. First carpometacarpal joint (CMCJ) arthrodesis

    CORRECT ANSWER: 1

    The patient in this vignette has symptomatic STT arthritis. STT arthrodesis provides the most reliable symptomatic relief for this pathology with the lowest risk for revision surgery.

     

    STT arthritis affects 16% of the population and is most often present in women over the age of 50. STT arthritis is less common than basilar thumb arthritis, but both are often found in association with one another and patients similarly most often complain of pain when pinching. Treatment of STT arthritis includes STT arthrodesis, scaphoid distal pole excision (open or arthroscopic), and excision with implant arthroplasty. However distal pole excision may potentiate pre-existing subtle mid-carpal instability, which can be seen on a lateral X-Ray with lunate extension and volar tilt of the capitate, as in this case (Figure B). If surgery is indicated, in the presence of pre-existing DISI deformity, STT arthritis should be treated with STT arthrodesis.

     

    Zimmerman and Weiss reviewed the treatments for STT arthritis. They discussed that while STT fusion is the most common treatment for this pathology, it is associated with an increased rate of complications, is more expensive, and hinges upon the union of small, sclerotic bones. Conversely, they held that distal pole excision was technically simple, more inexpensive, and may permit quicker return to activities. However, in the presence of carpal malalignment such as a DISI deformity, the authors strongly advocated for STT fusion.

     

    Tay et al. reviewed the outcomes of management of 24 wrists with STT arthritis and DISI deformity, sixteen of which also had CMCJ arthritis. They found that fifteen wrists went onto surgery (STT fusion - 7, STT resection arthroplasty - 1, Trapeziectomy - 4, Neurectomies - 1, Scaphoidectomy and four corner fusion - 2) and four of these required revision (all of which had a STT fusion as the index procedure). The authors recommend restoration of adequate scaphoid length and using an ample supply of bone graft to ensure restoration of carpal realignment. In the setting of STT arthritis, the presence of an extended lunate is not secondary to SL instability. Instead the scaphoid and lunate extend in concert. The authors caution that surgical intervention of STT arthritis may lead to progression of midcarpal instability.

    Kapoutsis et al. reviewed the etiology and treatment for STT arthritis. The authors found that type II lunates were far more likely to develop STT arthritis. They held that conservative management should be attempted first, including splint, anti-inflammatory medication, and steroid injections. They further noted that distal pole excision was contra-indicated in a wrist with a DISI posture as this worsens mid-carpal instability.

     

    Figures A and B are AP and lateral radiographs of a wrist demonstrating advanced concomitant STT and basilar thumb arthritis. The lateral shows an extended lunate in a DISI posture.

     

    Incorrect answers:

    Answer 2: Distal pole resection in this patient with underlying DISI deformity may potentiate mid-carpal instability and require a secondary intracarpal fusion.

    Answer 3: Because the elastic modulus of silicone is less than that of bone, the implant cannot withstand physiologic shear and compression. Pyrocarbon has an elastic modulus more similar to bone but the data is extremely limited in STT arthritis.

    Answer 4: Distal pole resection, whether performed open or arthroscopically, in this patient with a DISI posture of the wrist is contraindicated as it may potentiate mid-carpal instability and necessitate a revision surgery.

    Answer 5: First CMCJ arthrodesis would like make this patient’s symptoms worse as the STT joint would not be addressed and the joint may see higher shear forces.

     

     

     

  67. After a night out on the town, a 21-year-old college student strikes an inanimate object. The next morning the brash young man notes that his hand is swollen, tender, and painful to move. He presents that day to your clinic with the images seen in Figures A and

    B. You are able to reduce the injury and the reduction is initially maintained in an appropriately-molded splint. You obtain follow-up radiographs one week later which demonstrate recurrent dislocation. The next step in management should be:

     

     

     

    1. Discontinue immobilization, discharge non-weight-bearing but with early active range of motion

    2. Repeat reduction, follow up in 7-10 days for splint removal

    3. Repeat reduction, follow-up in 7-10 days for repeat images in plaster, immobilization for 4 weeks so long as reduction is maintained

    4. Schedule for closed reduction and percutaneous pinning

    5. Schedule for open reduction, CMCJ arthrodesis with plate application

    CORRECT ANSWER: 4

    This patient presents with acute fracture-dislocations of the 4th and 5th carpometacarpal joints (CMCJs). Given the instability and failure of closed management, the next best step would be closed reduction and percutaneous pin fixation.

     

    While CMCJ fracture-dislocations injuries represent <1% of all hand trauma, they are often missed because of significant diffuse swelling, misunderstanding of the injury, and misinterpretation of radiographs. These injuries are most often sustained after an axial load is applied to a closed first (such as when striking an object). The 5th and at times the 4th ray are most often involved.

    The deforming force for the 5th metacarpal base fracture is the ECU. Treatment is reduction and splinting. When seen acutely, these can be reduced

    and treated closed. However, if irreducible, the reduction cannot be maintained, or if the presentation is delayed, operative fixation is required. Surgical treatment may include closed reduction and pinning, open reduction and pinning, or arthrodesis. Reduction, either open or closed, with internal fixation (pinning) is the recommended treatment for the majority of these injuries. Closed reduction with pinning is most often performed for acute injuries, generally due to the success in treatment via this method and frequent absence of large osseous fragments to fix. Open reduction with pinning is performed for those injuries that cannot be reduced by closed means or those with a delayed presentation.

     

    Storken et al. review four cases of acute (<8 hour) reduction of ulnar-sided CMCJ fracture-dislocations. They found that all four cases remained reduced and concentric at follow-up. However, the authors caution that pinning should be considered in chronic subluxations or those in which closed reduction cannot be maintained. The authors describe one case which displaced 4 months after initial successful reduction.

     

    Zhang et al. compared the results of 20 acute and 6 chronic 4th/5th fracture dislocations. The 20 acute injures were treated with reduction/splint and all had good results. Three chronic injuries were treated with fixation and did well, however three chronic injuries were treated conservatively had noticeable deformity, pain, limited ROM, and decreased grip strength. They concluded that patients who present with a delayed CMCJ fracture-dislocation cannot be managed effectively with closed means.

     

    Kato et al. describe a technique of 5th CMCJ stabilization for chronic fracture-dislocations using an ECU-based suspension arthroplasty. The authors support that this procedure is advantageous in that the mobility of the ulnar CMCJs and length of the respective metacarpals is preserved. They conclude that this may be a good option for degenerative arthrosis in the setting of chronic 5th CMCJ fracture-dislocations.

     

    Figure A is an AP radiograph of a 4th and 5th CMCJ fracture-dislocation. Note the relative shortening of the affected metacarpals. Figure B is a lateral radiograph of a 4th and 5th CMCJ fracture-dislocation, revealing dorsal displacement.

    Illustration A shows a 5th metacarpal CMCJ fracture-dislocation which was treated with closed reduction and percutaneous pinning utilizing one of many standard techniques.

     

    Incorrect answers:

    Answer 1: This patient’s fracture is unstable, and the reduction was not

    maintained. Discontinuing immobilization would result in persistency dislocated 4th and 5th CMC joints and significant disability.

    Answer 2: This patient’s fracture is unstable and failed closed reduction in a well-molded splint. There is no reason that repeat reduction would be successful. Furthermore, immobilization should be maintained for at least 4 weeks.

    Answer 3: This patient’s fracture is unstable and failed closed reduction in a well-molded splint. He has a very high risk of re-displacement in the subsequent 4 weeks following a repeat reduction and is therefore indicated for operative stabilization.

    Answer 5: Joint fusion may be indicated in chronic or irreducible cases. However, in the acute setting, closed reduction and percutaneous pinning should be attempted first to minimize soft tissue disruption.

     

     

     

     

     

     

  68. A 70-year-old right-hand-dominant retired female postal worker has severely symptomatic end-stage arthritis of her left middle finger proximal interphalangeal joint (PIPJ). Your skillful hand surgery partner discusses treatment options with the patient, and they decide

    together to treat her PIPJ arthritis with implant arthroplasty. In counseling the patient on post-operative outcomes, which of the following would be accurate in regard to the surgical approach and implant selection for this case?

    1. Surface implant has a lower revision rate regardless of approach

    2. Silicone implant with volar approach has best range of motion and lowest revision rate

    3. Silicone implant with volar approach has best range of motion with the same revision rate

    4. Volar approach, regardless of implant, has the best range of motion and lowest revision rate

    5. There is no difference between approach and implant type

    CORRECT ANSWER: 2

    This older patient with end-stage arthritis of a non-border PIPJ without stated angular or rotational deformity is a perfect candidate for PIPJ arthroplasty. Of the possible implants and surgical approaches that could be utilized, silicone implants placed through a volar approach have comparatively been shown to result in the best range of motion with the lowest rate of revision.

     

    PIPJ arthritis is present in nearly 17% of women and 13% of men and often presents with symptoms including flexion contracture, decrease motion, swelling, and pain. The radial digits are often treated with fusion due to issues with stability, but non-border digits may be better suited for arthroplasty. Pain relief is reported in 98% of patients who undergo silicone arthroplasty and the complication rate is relatively low. The dorsal approach is the historic gold standard and may be technically more straightforward. Surface implants (metal-on-plastic) as well as a silicone implants are available. However, more favorable outcomes have recently been reported with the volar approach and with the use of silicone implants.

     

    Yamamoto et al (2017) performed a systematic review evaluating the various implants and approaches used for PIPJ arthroplasty. The authors found that, as compared to the surface implant with a dorsal approach, the silicone implant placed through a volar approach was associated with significantly less extensor lag, better net gain in arc of motion, better overall post-operative motion, and a lower revision rate (6% vs 18%). They concluded that silicone implant via volar approach was associated with the best outcomes for PIPJ arthroplasty.

     

    Yamamoto et al. (2018) comprehensively review exposure and implant designs for PIPJ arthroplasty. They showed that the volar approach resulted in the best

    range of motion, with the least extension lag, and lowest compilation rates. The surface implant had the highest rate of surgical revisions. The authors underscored the importance for continued improvement of implant design.

     

    Incorrect answers:

    Answer 1: The volar approach has been shown to be superior to the dorsal approach in terms of less extensor lag, improved range of motion, and less complications.

    Answer 3: The revision rate of silicone implants placed through a volar approach was 33% that of a surface implant placed through a dorsal approach. Answer 4: The silicone implant was superior to the other implant types in terms of range of motion and revision rate.

    Answer 5: The volar approach and silicone implant have been shown to be superior.

     

     

     

  69. A 28-year-old jack-hammer operator presents with 4 years of wrist pain. The patient was seen three years ago and had normal wrist radiographs at that time. The PA who saw him diagnosed him with a wrist sprain, and he subsequently returned to work. His symptoms never improved, even after a period of casting, so he was referred to your clinic after an MRI was obtained (Figure A). On physical examination, he has dorsal wrist tenderness and limited wrist extension. What surgical procedure may offer him the most reliable pain relief while best preserving the native anatomy?

     

     

     

    1. Vascularized bone graft

    2. Proximal pole excision and tendon interposition

    3. Scapho-capitate (SC) fusion

    4. Proximal row carpectomy

    5. Scaphoidectomy and four corner fusion (4CF)

    CORRECT ANSWER: 1

    This patient has avascular necrosis of the proximal pole of the capitate. The surgical procedure which would offer him the most reliable pain relief while best preserving the native anatomy is a vascularized bone graft.

     

    Aseptic or avascular necrosis (AVN) of the capitate is a rare condition and often occurs secondary to a traumatic injury. AVN of the capitate may be further potentiated by a number of unique anatomical features such as irregularity of the blood supply (which is typically palmar-dominant) and retrograde perfusion (analogous to the scaphoid). This condition is also purportedly more common in ligamentously lax patients. Conservative treatment begins with cast immobilization, though recalcitrant cases may be treated with capitate decompression, SC fusion, 4-corner fusion, or vascularized bone graft. A local vascularized graft offers the best chance of restoring perfusion to the capitate and preserving native anatomy.

    Peters et al. reviewed 6 cases of capitate AVN. Ultimately 4 were treated with SC fusion and 2 had a 4CF. The authors reported good-excellent results in 3, fair in 2, and poor in 1. They concluded that arthrodesis most reliably produced the best results.

     

    Lapinsky and Mack reviewed a single case of capitate AVN in a 20-year-old female. Intraoperatively, the authors found that the proximal pole was cystic and non-viable. As a result, the proximal pole was excised and replaced a tendon interposed. They reported that she continued to have pain postoperatively and went on to an eventual wrist arthrodesis. The authors underscore the difficulty of the diagnosis and difficulties in management.

     

    Hattori et al. reported a case of treating capitate AVN with a vascularized pedicled bone graft from the dorsal distal radius, specifically the 4th extra-compartmental artery. At one year, the authors reported excellent symptomatic relief and that the patient was able to return to playing baseball.

     

    Figure A is a T1 coronal MRI of the wrist demonstrating low signal intensity within the proximal pole of the capitate consistent with avascular necrosis.

     

    Incorrect Answers:

    Answer 2: Although this procedure has been described, it does not restore native anatomy. The case of tendon interposition above required revision to a wrist arthrodesis.

    Answer 3: A SC fusion is described for this condition but would not restore native anatomy.

    Answer 4: A PRC would likely exacerbate the condition as the radio-carpal joint would now articulate with a dysvascular capitate.

    Answer 5: A scaphoidectomy and four-corner fusion may be performed but this would not restore native anatomy.

     

     

     

  70. After losing an arm-wrestling match, an enraged orthopaedic resident punches a wall and has immediate pain and swelling about his dominant right hand. His co-resident examines his hand under fluoroscopy and identifies the injury noted in Figures A and B. The muscle responsible for the primary deforming force in this injury is innervated by which nerve?

     

     

     

     

     

    1. Anterior interosseous nerve (AIN)

    2. Radial nerve

    3. Posterior interosseous nerve (PIN)

    4. Ulnar nerve

    5. Extensor branch of ulnar nerve

    CORRECT ANSWER: 3

    The patient has sustained a 4th and 5th carpometacarpal joint (CMCJ) dorsal fracture-dislocation with an associated dorsal hamate fracture. The 5th metacarpal is displaced dorsally and proximally by the pull of the extensor carpi ulnaris (ECU), which inserts on the base of the 5th metacarpal and is innervated by the PIN.

     

    CMCJ fracture-dislocations are often missed on initial presentation. Radiographs should be carefully scrutinized for shortening of the metacarpals on the AP view. On the lateral view, metacarpal base subluxation or small fragments of the hamate may be present dorsally. Reduction is typically obtained with a local block, wrist extension, and pressure over the base of the metacarpal. Reduction may often then be maintained within a well-molded splint. However with more extensive joint involvement or soft tissue disruption, maintenance of reduction by closed means may be difficult and unreliable. Moreover, cases in which there is delayed presentation or those which do not stay reduced following closed reduction and splinting require percutaneous fixation or potentially even open with internal fixation.

     

    Zhang et al. evaluated the management of patients presenting with acute (20) and chronic (6) ulnar CMCJ fracture dislocations. All of those who received acute treatment were successfully managed conservatively with reduction and splinting. However, 50% (3 of 6) of the injuries treated in a delayed fashion failed conservative management and necessitated operative stabilization. The authors concluded that early reduction was paramount in avoiding surgical intervention and attaining the best outcomes.

     

    Gehrmann et al. reviewed 16 cases of 4th/5th and 23 cases of 5th CMCJ fracture-dislocations. The authors managed all cases with reduction and percutaneous pinning. They found that the two cohorts had comparable DASH scores and functional outcomes. The authors concluded that reduction of the ulnar CMCJ consistently resulted in excellent outcomes at one year follow-up.

     

    Figure A shows an AP radiograph of an ulnar fracture dislocation at the CMCJ. Note the shortening of the ulnar two metacarpals.

    Figure B shows a lateral radiograph of an ulnar fracture dislocation at the CMCJ. The metacarpals are dislocated dorsally.

    Incorrect Answers:

    Answer 1: The AIN innervates the deep flexors, but not the extensors. Answer 2: The radial nerve innervates the ECRL but the remainder of the extensors are innervated by the PIN.

    Answer 4: The ulnar nerve innervates the FDP to the ulnar two digits as well as intrinsic muscles in the hand, but not the ECU.

    Answer 5: There is not a nerve by this name.

     

     

     

     

  71. A 28-year-old female ski instructor sustained a fracture-dislocation of her index finger proximal interphalangeal joint (PIPJ) 10 years ago in a roller derby contest. The fracture was treated with distraction arthroplasty. The patient never retained functional range of motion. Additionally, since the injury, the patient has had increasing pain and a mild, progressive rotational deformity. Her radiograph is shown in Figure A. The patient is healthy with the exception that she is homozygous for factor V Leiden. She has attempted non-operative management but her symptoms are unbearable. What surgical intervention is the most appropriate for this patient?

     

     

     

    1. Ray resection

    2. PIPJ silicone arthroplasty

    3. PIPJ pyrocarbon arthroplasty

    4. PIPJ arthrodesis

    5. Vascularized PIPJ transfer from the second toe

    CORRECT ANSWER: 4

    This patient has end-stage arthritis of the PIPJ with an associated angular and rotational deformity - which indicates insufficiency of her collateral ligaments. She has failed conservative measures but is too young and active for interphalangeal joint arthroplasty; therefore PIPJ arthrodesis is the best option.

     

    PIPJ arthritis most often presents with a painful flexion contracture and may be secondary to a post-traumatic mechanism or an inflammatory etiology. In an older, less active patient without rotational deformity, silicone or pyrocarbon PIPJ arthroplasty is an excellent option for the small, ring, and middle fingers. However, the patient in this vignette is young and active with index PIPJ arthritis, therefore arthroplasty would be expected to have an unacceptably low survivorship. This patient is best indicated for a PIPJ arthrodesis. This can be performed utilizing either a tension band technique, a single headless compression screw, or a sliding screw with fusion plate. From radial to ulnar, the PIPJ should be fused in increasing degrees of flexion (40-55 degrees) to optimize function.

     

    Wagner et al. reviewed 170 PIPJ pyrocarbon arthroplasties. The authors found that patients had no significant improvement in motion but did experience significant pain relief. After 5 years, they noted that 51% had evidence of subsidence, 36% were unstable, and 34% required reoperation. As a result, the authors emphasized caution in performing pyrocarbon PIPJ arthroplasty in young patients with posttraumatic arthritis.

     

    Bales et al. reviewed the surgical outcomes among 51 patients who underwent silicone-based PIPJ arthroplasty for osteoarthritis. The average age in the study was 60 years old, and the authors found that all patients demonstrated improvements in terms of pain and DASH scores. While patients generally would undergo the procedure again, responses were otherwise mixed in regards to satisfaction with physical appearance, function, and range of motion. At the time of final follow-up, the authors reported an average arc of motion of 55 degrees, and that nearly 60% had radiographic deformation while 40% had implant failure. They concluded that in an older, less active patient population, silicone PIPJ arthroplasty offered reliable pain relief.

    Vitale et al. compared arthrodesis and arthroplasty for osteoarthritis or posttraumatic arthritis in the index finger PIPJ specifically. Arthroplasty did not improve PIPJ ROM. There was no difference in pain relief, satisfaction, or Michigan Hand Questionnaire scores between groups. Finally, the authors underscored that arthroplasty patients had a 4.3 times increased risk of complications in the index finger.

     

    Yamamoto and Chung comprehensively reviewed exposure and implant designs for PIPJ arthroplasty. The authors showed that the volar approach resulted in the best arc of motion, with the least extensor lag, and lowest complication rates. They further noted that the surface implant had the highest rate of surgical revisions. The authors underscored the importance for continued improvement of implant design.

     

    Figure A is a lateral radiograph of the PIPJ showing post-traumatic end-stage arthritis.

     

    Incorrect Answers:

    Answer 1: A ray resection could be performed as a salvage procedure, but it is not the best initial choice.

    Answer 2: Although PIPJ silicone arthroplasty via a volar approach yields the best range of motion and lowest complications when compared to the surface implant (metal and plastic) performed through a dorsal approach, PIPJ arthroplasty is not recommended for young patients with post-traumatic arthritis due to high rates of complications and low survivorship.

    Answer 3: PIPJ arthroplasty is not recommended for young patients with posttraumatic arthritis. A pyrocarbon implant would furthermore be contraindicated in this case due to collateral ligament deficiency, given that collateral ligaments are required for a pyrocarbon implant.

    Answer 5: A vascularized toe transfer can be performed in younger patients with PIPJ arthritis, it is not recommended when a patient has a clotting disorder.

     

     

     

  72. A 35-year-old jack-hammer operator presents with progressive hand weakness over the last year. He has been reluctant to see a physician. He notes some loss of coordination but otherwise denies any numbness. When asked to pinch a piece of paper, his thumb IPJ and index finger DIPJ flex. Electrodiagnostic studies demonstrate positive sharp waves in the adductor pollicis. An MRI is obtained and is seen in Figure A. What is the physical relationship of the affected neurovascular bundle?

     

     

     

    1. The superficial branch of the radial nerve is dorsal and radial to the radial artery

    2. The median nerve is ulnar to the radial artery

    3. The ulnar nerve is dorsal and radial to the ulnar artery

    4. The ulnar nerve is volar and radial to the ulnar artery

    5. The ulnar nerve is ulnar and dorsal to the ulnar artery

    CORRECT ANSWER: 5

    The patient has a ganglion cyst within Guyon's canal, just ulnar to the hook of the hamate. Within Guyon's canal, the nerve is dorsal and ulnar to the artery.

     

    This location of the patient’s mass is concordant with his symptoms. The MRI shows a well-circumscribed, homogenous lesion which has high intensity on T2, consistent with a benign ganglion cyst. The patient’s occupation: a jack-hammer operator, also puts him at risk for developing similar symptoms through a different etiology, namely hypothenar hammer syndrome. In this case, the patient’s adductor pollicis has developed positive sharp waves, which is a sign of abnormal insertional activity on electromyography. Because of his history, physical exam, and electromyography demonstrating ulnar nerve denervation, the mass should be excised.

     

    Maroukis et al. review the clinical anatomy of Guyon’s canal. The authors note that the floor is comprised of the transverse carpal ligament, the roof formed by the volar carpal ligament, the radial border formed by the hook of the hamate, and the ulnar border comprised of the pisiform. They further note the subdivisions within the canal, specifically that zone 1 is proximal to the ulnar nerve bifurcation, zone 2 surrounds the deep motor branch, and zone 3 surrounds the superficial sensory branch.

     

    Wang et al. retrospectively reviewed 9 patients with ulnar nerve compression within Guyon’s canal who underwent decompression. They found that in

    almost all (8/9) cases, the ganglion cyst originated from the pisohamate joint, and from the midpalmar space in the last case. The authors reported that all patients improved in strength after excision and decompression. They advocate early surgical management for best results.

     

    Figures A is the T2 axial MRI image of the patient's wrist demonstrating a ganglion cyst within Guyon's canal.

     

    Incorrect Answers:

    Answer 1: This is true, but the SBRN and radial artery are not affected in this case.

    Answer 2: This is true, but the median nerve and radial artery are not affected in this case.

    Answer 3: The ulnar nerve is dorsal and ulnar to the artery. Answer 4: The ulnar nerve is dorsal and ulnar to the artery.

     

     

     

  73. An unemployed 60-year-old female, who smokes cigars daily, has 3 years of increasing wrist pain. She was initially seen by her primary care physician who referred the patient to a skilled hand surgeon. The hand surgeon diagnosed her with Kienbock disease and gave her an injection at that time. She has been managed conservatively but now her symptoms are increasingly severe. Her most recent radiographs are shown in Figures A and B. The surgeon recommends a proximal row carpectomy (PRC), but expresses that a total wrist arthrodesis would not be warranted due to the limited extent of carpal involvement. Based on the radiographs and surgeon’s

    recommendation, the patient’s lunate pathology is at what stage?

     

     

     

     

     

    1. Stage I - lunate edema

    2. Stage II - lunate sclerosis

    3. Stage IIIa - lunate collapse

    4. Stage IIIb - lunate collapse with scaphoid flexion

    5. Stage IV - lunate collapse with pancarpal arthritis

    CORRECT ANSWER: 4

    The patient presents with advanced Kienbock's disease, or avascular necrosis of the lunate. The patient has Kienbock's disease with a collapsed lunate and a flexed scaphoid. The radio-scaphoid angle >60 degrees in the Lichtman classification, moving the injury from a stage IIIa to IIIb.

     

    Kienbock's Disease disease is thought to be precipitated by a combination of anatomic and biomechanical factors such as repetitive trauma, ulnar negative variance, decreased radial inclination, and vascular congestion. Lichtman classified Keibock's disease into four stages, based on the severity of the disease in order to guide diagnosis and management. The earliest stage of Kienbock's disease (Lichtman stage I) may be treated with immobilization. As lunate sclerosis becomes evident on plain films (stage II) the condition is then managed with either radial or capitate shortening, STT fusion, or a revascularization procedure. The same treatment applies to isolated lunate disease as the lunate fragments but the scaphoid remains in appropriate position (stage IIIa). However, with lunate collapse and fixed scaphoid flexion (stage IIIb), a PRC is one of the recommended treatments with other options including lunate excision and limited intercarpal fusion (scaphotraezio-trapezoidal or scaphocapitate). Finally, with degeneration of the remainder of the carpus (stage IV) the patient’s options may be limited to a PRC, intracarpal fusion, or wrist fusion.

     

    Lichtman et al. describe the four-stage classification and treatment based on each stage. The authors noted that the most important point in the classification between IIIa (lunate collapse) and IIIb (lunate collapse with fixed scaphoid flexion). While the former can be treated as in stage II, with joint leveling procedures, stage IIIb will require a salvage procedure like a PRC.

     

    Diao et al. comprehensively review PRC and the utility of this procedure in cases of scaphoid nonunion advanced collapse, scapholunate advanced collapse, perilunate fracture-dislocations, lunate dislocations, and Kienbock's disease. The authors recommend not performing a PRC in patients younger than 35. Adjunct procedures may include capsule interposition, capsule excision, radial styloidectomy. Although longevity is notable, patients may undergo secondary procedures.

    Figure A is an AP radiograph of a patient with lunate sclerosis and collapse. Figure B is a lateral radiograph of the same patient which demonstrates scaphoid flexion with radio-scaphoid angle > 60 degrees.

     

    Incorrect Answers:

    Answer 1: In stage I, the patient would be offered immobilization. Answer 2: In stage II, the patient would likely be offered a joint-leveling procedure.

    Answer 3: In stage IIIa, the patient would likely be offered a joint-leveling procedure or a vascularized graft.

    Answer 5: In stage IV, the lunate collapses and the other carpal articulations become arthritic. In this case, a total wrist arthrodesis would be warranted.

     

     

     

  74. Figure A is a clinical photo of a 2 year old male with a right hand abnormality. In addition to his right hand deformity, he has a similar appearance of his left hand. He also has a cleft palate, craniofacial defects, and bilateral club feet. His upper extremities are otherwise normal proximal to his wrists. The diagnosis was suspected after a routine ultrasound. What is the inheritance pattern of this condition?

     

     

     

     

    1. Autosomal dominant

    2. Autosomal recessive

    3. X-linked recessive

    4. X-linked dominant

    5. Sporadic CORRECT ANSWER: 5

    The patient has amniotic band syndrome (ABS), which has a sporadic inheritance.

     

    ABS occurs when loose fibrous bands of ruptured amnion adhere to and entangle the normal developing fetal structures. It is present in 1:15,000 lives births, has equal male: female occurrence, and most typically affects the upper extremities. There is a wide spectrum of severity from a simple constriction ring to acrosyndactyly (which refers to fusion of digits distally with proximal digit fenestrations) or amputation. Other conditions (club foot, cleft palate, cleft lip, craniofacial defects) are found frequently with constriction band syndrome and patients typically have a low birth weight, are born preterm, and have a higher infant mortality rate.

     

    Kawamura and Chung report that ABS is an uncommon abnormality that occurs with early amnion rupture and entanglement of developing fetal anatomic structures by the amniotic strands. The authors note that management consists of improving function through soft tissue release and reconstruction.

     

    Moran et al. review ABS and highlight the array of digit deletions and deformations resulting from trapped amniotic tissue in the distal most aspect of the extremities. They discuss that the severity varies widely, thus treatment spans a wide spectrum from observation to soft tissue reconstruction. They note that improvements in prenatal diagnosis and fetoscopic surgical techniques may allow in utero treatment in the future.

     

    Koskimies et al. reviewed the 71 cases of congenital constriction band syndrome in Finland over a 13 year period and found an infant mortality rate of 4.6% and perinatal mortality rate of 12.7%. Moreover, 31% were preterm and 18% had a low birthweight. The authors noted that only upper limbs were affected in 49%, while 18% had isolated lower limb involvement. They reported that no cases presented within the context of a known syndrome, and that other features present included pes equinovarus (38%), cleft palate (24%), and cardiac defects (29%). The authors supported that there may be an undetermined genetic defect.

     

    Figure A is a clinical photo of a patient with constriction band syndrome. The hand has acrosyndactyly -digits fuse distally and proximally the digit has fenestrations.

     

    Incorrect Answers:

    Answer 1: Marfan’s syndrome, Cleidocranial dysostosis, Multiple Hereditary Exostosis, Achondroplasia, Kniest dysplasia, Ehlers-Danlos syndrome, and

    Multiple Epiphyseal Dysplasia are autosomal dominant.

    Answer 2: Diastrophic dysplasia, Friedreich’s ataxia, spinal muscular atrophy, sickle cell anemia, and Gaucher disease are autosomal recessive.

    Answer 3: Duchenne muscular dystrophy, Becker’s muscular dystrophy, Hunter’s syndrome, and Hemophilia are X-linked recessive.

    Answer 4: Leri-Weill dyschondrosteosis and hypophosphatemic rickets are X-linked dominant.

     

     

     

  75. A 30-year-old patient presents for evaluation of a volar radial wrist mass. Examination reveals a firm palpable 1 x 1 cm mass that transilluminates to light. It does not cause him any pain but he is concerned about its appearance. Which of the following is true with respect to the management of his pathology?

    1. Aspiration of the lesion is associated with a 33% recurrence rate

    2. It constitutes 40% of all hand masses

    3. Open surgical excision of the lesion is associated with a 20% risk of recurrence

    4. Observation is associated with a recurrence rate in about 75% of cases

    5. Open surgical excision leads to improved outcomes compared to arthroscopic excision

    CORRECT ANSWER: 3

     

    The patient presents with clinical signs and symptoms of a volar wrist ganglion. With surgical excision, there is a 15-20% risk of recurrence.

     

    Volar wrist ganglions are benign soft-tissue masses that occur secondary to trauma, mucoid degeneration, or synovial herniation from various carpal articulations. They are generally filled with mucin from a joint or tendon sheath. They are extremely common, mostly asymptomatic, and their sizes may increase and decrease over time. Examination generally reveals a firm well-circumscribed mass that transilluminates to light. Management involves non-operative measures (observation, aspiration) or operative measures (surgical excision).

     

    Mayerson et al. wrote a review of soft tissue masses detailing the importance of a thorough history and physical, appropriate imaging studies and diagnostic studies required to lead to the correct diagnosis. They noted that when the initial evaluation is inconclusive, a biopsy must be performed and a diagnosis must be established before definitive treatment. The authors also conclude that a multidisciplinary approach optimizes outcomes and diminishes comorbid

    complications in the management of soft-tissue masses.

     

    Head et al. performed a meta-analysis comparing open surgical excision and aspiration for the treatment of wrist ganglions. A total of 6 studies met criteria for meta-analysis, including 2 RCTs and 4 cohort studies. Across all studies, the mean recurrence for arthroscopic surgical excision (512 ganglions), open surgical excision (809 ganglions), and aspiration (489 ganglions) was 6%, 21%, and 59%, respectively. Furthermore, aspiration was not associated with a significant reduction in recurrence compared with reassurance alone. The authors conclude that open surgical excision offers a significantly lower chance of recurrence compared with aspiration in the treatment of wrist ganglions.

    Arthroscopic excision has yielded promising outcomes but data from comparative trials are limited and have not demonstrated its superiority to open excision.

     

    Incorrect Answers:

    Answer 1: Aspiration of the ganglion is generally associated with a 40-60% recurrence rate.

    Answer 2: Ganglion cysts are the most common hand mass and comprise about 60-70% of all hand masses.

    Answer 4: Observation/reassurance leads to a recurrence rate of 50-60% (similar to aspiration).

    Answer 5: There is a reported lower recurrence rate of arthroscopic surgical excision of ganglions compared to open excision.

     

     

     

  76. A 55-year-old male, construction worker, presents to your clinic with chronic left-hand pain. The pain is associated with paresthesias of the thumb and index finger. He has been wearing a removable wrist brace which has partially helped his symptoms. He has a positive Tinel's over his left wrist but otherwise unremarkable. He is mostly interested in non-operative management and asks you about a corticosteroid injection (CSI). With regards to this treatment, what advice can you give this patient?

    1. 80% of patients with mild symptoms will have improvement with a CSI, 20% of those patients will have lasting effects up to 1 year

    2. Methylprednisolone reduces symptoms at a greater rate than triamcinolone

    3. The dosage of the steroid directly correlates to the duration of its effect

    4. A poor response to the CSI is a good prognosticator for symptom relief from surgery

    5. 60% of patients will have improvement of symptoms with a CSI, 40% of those patients will have lasting effects up to 1 year

    CORRECT ANSWER: 1

     

    The patient in the vignette has clinical signs and symptoms of carpal tunnel syndrome (CTS). Corticosteroid injections are a well-proven diagnostic and therapeutic treatment and around 80% of patients with mild CTS generally have at least a temporary improvement in their symptoms, with 20% of these patients expected to have symptom relief last up to 1 year.

     

    CTS is a compressive neuropathy of the median nerve occurring at the transverse carpal ligament. Various studies have shown its prevalence to range from 0.1 to 10% of the population in various cohorts. Risk factors include obesity, activities with repetitive wrist motion, constant pressure on the volar wrist (typewriters, truck drivers, etc), hypothyroidism, diabetes, and mucopolysaccharidosis. Diagnosis is made primarily via history and physical notable for numbness/tingling in median nerve distribution, nocturnal numbness, positive Tinel's and Phalen's test at the wrist, loss of 2 point median nerve distribution ( > 5 mm) and in severe cases, weakness and atrophy of the thenar musculature. Nerve conduction studies are also used for diagnostic purposes as well as for measuring response to treatment. Nonoperative management begins with removable wrist splints and NSAIDs. A corticosteroid injection can assist in both diagnosis (in equivocal cases) and management. A good response to a CSI is considered to be a good prognosticator for symptom relief following carpal tunnel release.

     

    Kane et al. described the phenomenon of the double crush syndrome which is a compressive neuropathy at two or more locations along the course of a peripheral nerve that can coexist and synergistically increase symptom intensity. Lack of response to treatment at one site should prompt evaluation of other sites. As such, diagnostic tools such as nerve conductive studies (NCS), CSIs, and evaluation for concomitant cervical radiculopathy, should be utilized in such cases.

     

    Ponnappan et al. looked at the various etiologies of upper extremity pain and subdivided them into 4 categories: neurologic, musculoskeletal, vascular, and other (eg, tumor, infection). They discussed the importance of clinical presentation and physical examination findings to help in distinguishing the source of the patient's complaints to narrow in on the correct diagnosis to help expedite appropriate treatment.

     

    Illustration A shows the CTS-6 diagnostic tool used for carpal tunnel screening.

    Incorrect Answers:

    Answer 2: There is no evidence that a CSI comprised of methylprednisolone is superior to triamcinolone in duration or amount of symptom relief.

    Answer 3: The dosage of the steroid or amount of steroid has not been shown to correlate to the response of a CSI for CTS.

    Answer 4: A poor response to the CSI is directly correlated to the likelihood of not having a good outcome with carpal tunnel release.

    Answer 5: Around 80% of patients generally have at least a temporary improvement in their symptoms, with 20% of these patients expected to have the symptom relief last up to 1 year.

     

     

     

     

     

     

  77. A 35 year old infantryman sustained a scaphoid fracture 9 months ago during a training exercise in a foreign country. As a result, his surgery was delayed but he ultimately underwent surgical fixation 3 months after his injury. While your partner is deployed, he presents to your clinic complaining of persistent wrist pain and inability to perform push-ups. Radiographs obtained today are seen in Figures A and B. What is the most appropriate next step in management?

     

     

     

     

     

    1. CT scan along the scaphoid axis

    2. Occupational therapy for wrist range of motion

    3. MRI with contrast

    4. Pulsed electromagnetic field application

    5. Casting with repeat radiographs at follow-up

    CORRECT ANSWER: 1

    A patient with pain, limited function, and no radiographic evidence of healing 6 months after scaphoid fracture fixation may have a scaphoid non-union. To confirm the diagnosis, a CT scan along the scaphoid axis is the next best step in care.

     

    A scaphoid fracture non-union is defined as a failure to achieve union 6 months after cast immobilization or operative fixation. The longer a non-union persists, the higher the likelihood that secondary issues may develop, including arthritis or carpal instability. Union rates following primary scaphoid fracture fixation are >90%. Though there is some conflicting data, generally operative fixation of scaphoid fractures yields similar union rates to nonoperative management but with faster return to work and shorter time to union. A CT is considered the gold standard in the diagnosis of a scaphoid fracture non-union following fixation.

     

    Yin et al. performed a systematic review and meta-analysis of the diagnosis of suspected scaphoid fractures using bone scintigraphy, MRI, and CT. They found that bone scintigraphy and MRI demonstrated equally high sensitivity and high diagnostic value for excluding scaphoid fractures, however MRI was more specific. They concluded that both CT and MRI had >90% sensitivity and specificity in the diagnosis of a suspected scaphoid fracture.

     

    The same authors (Yin et al.) later performed a cost analysis on diagnostic strategies for suspected scaphoid fractures. They showed that the average cost per scaphoid fracture with immediate CT was $2,553 and MRI was

    $7,483. The authors concluded that immediate advanced imaging (CT or MRI) was the most cost-effective strategy for diagnosing suspected scaphoid fractures compared to delayed radiograph-based strategies.

     

    Lutsky and Matzon reviewed the best course of action when a persistent fracture line is noted after scaphoid fixation. The authors highlighted that interpretation of healing on 12-week post-operative radiographs demonstrated poor interobserver reliability and reproducibility, but that interobserver agreement was substantially improved with the addition of a CT scan. The authors held that a lack of bridging trabeculae on CT scan at 6 months after screw fixation for a scaphoid fracture was diagnostic for non-union.

    Figures A and B are the AP and lateral radiographs of a scaphoid fracture fixed with a headless compression screw. A fracture line is still evident.

     

    Incorrect answers:

    Answer 2: Persistent pain with wrist extension and axial load at 6 months post-operatively is concerning for non-union, which should be ruled out prior to further occupational therapy.

    Answer 3: A non-contrast MRI is useful in the initial diagnosis of a suspected scaphoid fracture, but an MRI with contrast would not best demonstrate nonunion.

    Answer 4: Pulsed electromagnetic field application following scaphoid fracture fixation and subsequent non-union has demonstrated some evidence supporting its utility, however the diagnosis of non-union must made first.

    Answer 5: CT is superior to radiographs in the assessment of scaphoid fracture non-unions following fixation.

     

     

     

  78. A 56-year-old male presents to the office with the clinical picture shown in Figure A. He elects treatment with a collagenase injection. The patient receives his injection and due to social reasons is unable to follow-up for one week. How will his current outcome compare to if he could follow-up the next day?

     

     

     

     

    1. He will have increased pain on cord manipulation

    2. He will have less than 50% contracture correction at 7 days post injection

    3. He will have greater contracture correction at 7 days post injection

    4. He will have equivalent contracture correction

    5. He will have decreased rate of skin tears

    CORRECT ANSWER: 4

    The clinical image in Figure A depicts a Dupuytren’s contracture of the small finger with a contracture of approximately 70 degrees. This patient elected treatment with collagenase injection. Current literature has shown that contracture correction is equivalent at 1 day or 7 days following collagenase injection.

     

    Dupuytren’s Disease is inherited in an autosomal dominant manner and more commonly seen in males, Caucasians, and people of northern European descent. The pathologic cell type is the myofibroblast with an abundance of Type III collagen. Current treatment options include observation, needle cordotomy, Collagenase injection (Xiaflex) or surgical resection.

     

    Mickelson et al. randomized patients to manipulation either one day or 7 days after collagenase injection and found no significant difference in contracture correction. In fact, 91% of patients with an MCP contracture had a contracture of 5 degrees or less at 30 days post-injection and 100% of patients had greater than 50% contracture correction. For the PIP joint, contracture correction greater than 50% was 100% for 1 day and 91% for 7 days. There were no significant differences in pain scores and increased skin tears were seen with severe contractions. Higher auto-ruptures were seen in the 7-day group with MCP contractions with near-complete correction. However, Mickelson et al. did not observe this with PIP contractions. They concluded that cord correction is preserved for manipulations one week from injection without increased pain and skin tearing.

     

    Hentz et al. found similar clinical success between one day and 7 days between injection and manipulation. For MCP contracture 1-day manipulations went from 47 to 11 degrees and 7 days went from 46 to 9 degrees. Similar success was found with PIP cords that went from 56 to 25 degrees and 53 to 16 degrees for one and 7 days, respectively. Additionally, there was a higher auto rupture rate at 7 days. Hentz et al. determined that delayed manipulation at one week achieved equivalent correction.

     

    Badalamente et al. looked at results from 4 phase 3 clinical trials of collagenase injections for PIP cords. A total of 1,165 injections were reviewed in 644 PIP cords. He found 60% of patients only required one injection, but those with more severe cords required more and has less correction. Common side-effects were edema (most common), contusion, bleeding, and pain, swelling, and tenderness at injection site. Only 2 tendon ruptures occurred.

    Ultimately, they determined that collagenase injections were safe and effective for PIP cords.

     

    Figure A is a clinical image of a small finger PIP Dupuytren's contracture of

    approximately 70 degrees.

     

    Incorrect Answers:

    Answer 1: VAS pain scores have not been shown to be different between manipulation one day and 7 days following injection

    Answer 2: Manipulation at 1 day and 7 days both achieved greater than 50% clinical correction and there was no difference in clinical success rates of correction at 30 days.

    Answer 3: Clinical correction at 30 days was found to be equivalent with manipulation at 1 day and 7 days following injection.

    Answer 5: Skin tear rates were not found to be decreased. In fact, increased skin tears were associated with the severity of the contracture and not the length of time between injection and manipulation.

     

     

     

  79. A 30-year-old male undergoes surgical management and subsequent hardware removal for the injury identified in Figure A. Plain radiographs two months after his injury demonstrate a radiodense appearance of the lunate concerning for ischemia (Figure B). What is the next best step in management?

     

     

     

     

     

    1. Temporary scaphotrapeziotrapezoidal pinning

    2. Observation

    3. Proximal row carpectomy

    4. Capitate shortening osteotomy

    5. Cast immobilization CORRECT ANSWER: 2

    The patient has sustained a lunate dislocation with subsequent ischemia, which is likely transient. Transient ischemia after lunate and peri-lunate dislocations is usually benign and self-limiting.

     

    Lunate and perilunate dislocations typically result from high energy mechanisms and are commonly missed on initial presentation. Acute injuries require emergent closed reduction and splinting with close observation for acute carpal tunnel symptoms. This is usually followed by open reduction internal fixation with a ligamentous repair. The ischemia and increased radiodensity that develops in some patients does not typically follow the clinical and radiographic progression associated with avascular necrosis, Kienböck’s disease.

     

    White et al. reported on the transient vascular compromise of the lunate after fractures and fracture-dislocations of the wrist. In their series of 24 patients who sustained dislocation or fracture-dislocation of the carpus, 3 (12.5%) developed an increase in radiodensity within one to four months of injury. The radiographic appearance of the lunate returned to normal in two of the three patients with partial resolution in the third and stable appearance 8 years after injury. None progressed to the classic signs of avascular necrosis. The authors recommend nonoperative treatment with observation.

     

    Stansbury et al. reviewed perilunate dislocations and perilunate fracture-dislocations. They recommend careful radiographic inspection in trauma victims with wrist pain. Carpal height and carpal arcs should be assessed on the AP while collinearity of the radius, lunate and capitate should be maintained on the lateral. The authors note the development of lunate transient ischemia should not be overinterpreted, as it is likely to resolve.

     

    Forli et al. report on long-term functional outcomes after perilunate dislocations and transscaphoid perilunate fracture-dislocations. The authors retrospectively reviewed 18 patients with a minimum of 10-year follow-up. The presence of arthritis identified on radiographs and static carpal instability did not cause reduced function. They conclude signs of posttraumatic arthritis after these injuries increase progressively, but are well tolerated at an average of 13-years after injury.

    Figure A demonstrates the volar dislocation of the lunate. Note the “spilled teacup sign” and loss of collinearity of the radius, lunate and capitate. Figure B demonstrates a radiodense lunate consistent with ischemia.

     

    Incorrect Answers

    Answer 1: Temporary scaphotrapeziotrapezoidal pinning is a treatment for adolescents with radiographic evidence of Kienböck’s disease and progressive wrist pain.

    Answer 3: Proximal row carpectomy is a treatment for significantly progressed Kienböck’s disease. In addition, PRC can be used to treat chronic perilunate injuries (defined as >8 weeks after initial injury).

    Answer 4: Capitate shortening osteotomy is a treatment for Lichtman stage II and III Kienböck’s disease.

    Answer 5: Immobilization is the initial management for Lichtman stage I Kienböck’s disease, but not indicated for transient ischemia. A majority of patients with Kienböck’s disease will undergo further degeneration despite nonoperative measures.

     

     

     

  80. A 75-year-old female presents with difficulty extending her fingers and thumb. Her exam is notable for subcutaneous nodules, ulnar deviation of the metacarpophalangeal joints, and swan neck deformities. The wrist radially deviates with extension. Extension cannot be actively initiated or maintained when her fingers are passively manipulated. Tenodesis is normal. What pathology would best explain her symptoms?

    1. Attenuation of the sagittal bands

    2. Tendon rupture

    3. Peri-elbow synovitis causing nerve compression

    4. Intersection syndrome

    5. Distal radioulnar joint synovitis

    CORRECT ANSWER: 3

    The patient has rheumatoid arthritis and an inability to activate muscles innervated by the posterior interosseous nerve (PIN). Nerve compression at the level of the elbow joint best explains her symptoms.

     

    The PIN is a branch of the radial nerve and provides innervation to the extensor compartment of the forearm. PIN compression leads to weakness of

    wrist, fingers, and thumb. Preserved innervation to the ECRL allows for wrist extension, but with concomitant radial deviation as a result of a denervated ECU. No cutaneous fields are supplied by the PIN, so diminished sensation would not be observed.

     

    Dang et al. review compression neuropathies of the forearm and draw attention to a variety of etiologies. Many cases of PIN neuropathy are attributed to microtrauma from repetitive movements, typically associated with a patient’s occupation and activity. Other causes include fibrous bands, space-occupying lesions, and peri-elbow synovitis associated with rheumatoid arthritis. The authors advocate for the use of electrodiagnostic studies to confirm the diagnosis and consider other etiologies such as tendon rupture or subluxation in addition to a more proximal nerve lesion.

     

    Millender et al. reported on three cases of PIN paralysis caused by anteriorly bulging synovitis in rheumatoid arthritis patients. All were initially diagnosed as extensor tendon ruptures. One resolved after steroid injection, the second resolved after synovectomy, and the third required tendon transfers. The authors note a history of elbow pain and shooting pains along the course of the radial nerve prior to paralysis. In addition, extensor weakness precedes paralysis, in contrast to tendon rupture.

     

    Incorrect Answers:

    Answer 1: Attenuation or rupture of the sagittal bands leads to subluxation of the extensor tendons and an inability to initiate digit extension. However, patients retain the ability to maintain extension after they are passively extended.

    Answer 2: Tendon rupture would result in an abnormal tenodesis exam. Answer 4: Intersection syndrome is due to inflammation at the crossing point of the 1st and 2nd dorsal compartments. It typically presents as pain over the dorsal forearm and wrist.

    Answer 5: DRUJ synovitis can cause ECU subluxation, volar carpal subluxation, and increased pressure over the extensor compartments. This can ultimately lead to extensor tendon rupture, which would result in an abnormal tenodesis exam.

     

     

     

  81. A 24-year-old stagehand fell 12 feet off of a ladder while preparing a set. As he tried to brace his fall, he landed directly on his extended and ulnarly deviated left hand. He was taken to the local teaching hospital where radiographs were taken, shown in Figures A

    and B. What additional data is most necessary to obtain before a reduction is attempted?

     

     

     

     

     

    1. Distal vascular exam

    2. Neurological exam

    3. Wrist MRI

    4. Doppler Allen test

    5. DRUJ stability assessment

    CORRECT ANSWER: 2

    Between 25-50% of perilunate injuries present with acute carpal tunnel syndrome. A neurological exam should be performed before and after the reduction of a perilunate dislocation.

     

    Perilunate dissociation (lunate dislocation) is most often sustained by an axial load to an extended wrist in ulnar deviation. As many as one-quarter of cases are missed initially. Prompt recognition and reduction are critical but a neurovascular exam should be performed both before and after the reduction. The reduction is obtained typically by extending the wrist, applying axial traction, and then flexing the wrist while applying dorsal pressure over the carpus. Surgical management is always necessary and involves ligament repair, carpal stabilization with pinning, fracture fixation if necessary, and possible carpal tunnel release in the presence of persistent median nerve paresthesias. Emergent management is indicated in the setting of acute carpal tunnel syndrome which persists following reduction.

     

    Budoff reviews the treatment of acute lunate and perilunate dislocations and underscores the devastating nature of these injuries. The author notes that closed reduction should be completed immediately upon presentation to relieve the pressure of the lunate on the median nerve, and if the reduction is successful, surgery can be delayed 4 days until the swelling subsides. He discusses that median nerve dysfunction is typically static and non-progressive due to swelling and hemorrhage within the carpal tunnel and does not mandate emergent release. However, he notes that delayed onset or progressive worsening of carpal tunnel syndrome is an indication for emergent carpal tunnel release combined with open reduction and carpus stabilization.

     

    Stanbury and Elfar review the evaluation and management of perilunate dislocations. They emphasize that the radiographic evaluation will show loss of carpal congruity, which can often be missed on initial evaluation. The authors underscore the urgency of closed reduction followed by appropriate fixation.

    They discuss post-injury sequelae, including arthritis, median nerve dysfunction, and carpal instability, though note post-injury arthritis does not correlate with symptomatology.

     

    Dunn et al. reviewed 40 perilunate dislocations in a military population. They

    noted that over one-quarter (27.5%) were missed initially. Furthermore, one-half (50%) presented with acute carpal tunnel syndrome, and 17.5% had persistent paresthesias. The authors cautioned that a high-demand patient population may experience inferior functional results due to a greater degree of postoperative limitation.

     

    Figure A is an AP radiograph demonstrating a lunate dislocation. Note the triangular shape of the lunate, disruption of natural arcs within the wrist, with the lunate out of plane in comparison to the capitate and triquetrum.

    Figure B is a lateral radiograph demonstrating a lunate dislocation. Note the lunate volar to the carpus.

     

    Incorrect answers:

    Answer 1: A distal vascular exam is important in every patient, but unlikely to be abnormal in a perilunate injury.

    Answer 3: A wrist MRI may be helpful in evaluation of a potential scapholunate ligament injury or occult scaphoid fracture. However, this is needed neither to make a diagnosis nor for guiding treatment in this case.

    Answer 4: A Doppler Allen test is important when removing a volar ganglion cyst, but is not required before a perilunate reduction.

    Answer 5: A DRUJ injury would be uncommon in a perilunate dislocation and therefore a DRUJ stability assessment is not required prior to reduction.

    to reduction.

     

     

     

  82. An 18-year-old male presents to your clinic after sustaining a left-hand injury while playing basketball with his friends. He has swelling over the left middle finger. Radiographs are normal. When you flex his PIPJ over the edge of a table and ask him to extend his finger against resistance, his DIPJ becomes taught. You discuss initial non-operative management with the patient. Which of the following is the most appropriate initial plan of care for the treatment?

    1. Full time DIP splinting for 4-6 weeks

    2. PIP splinting for 6 weeks while allowing for DIP motion

    3. MCP extension splinting for 2-4 weeks

    4. Dorsal blocking splint of the PIP joint for 4 weeks

    5. PIP and DIP splinting for 4 weeks, then allow gentle passive range of motion of DIP only

    CORRECT ANSWER: 2

    The patient has a left middle finger central slip injury (Zone III extensor tendon) as confirmed by a positive Elson's test. This can be managed with PIP extension splinting for 4-6 weeks while maintaining DIP motion.

     

    Central slip (Zone III extensor tendon) injuries can occur after hyper-flexion injuries, direct blunt trauma or penetrating trauma to the digit. Diagnosis can be made by a thorough physical exam with a positive Elson's test. This is performed by placing the PIP joint over a table at 90 degrees and asking the patient to extend PIP against resistance. If the central slip is intact, the DIP joint will remain supple while if the central slip is disrupted, the DIP will be rigid. Management involves immobilization of the PIP joint while allowing for DIP motion to prevent stiffness and adhesions. If left untreated, these injuries can progress to a Boutonniere deformity.

     

    Scott wrote an overview of the pathoanatomy and management of injuries to the flexion-extension system of the digits. He cites that any imbalance as a result of the injury will often lead to both visible and functional deformities. He stresses that while these deformities are initially supple they can become fixed and progress to IP joint osteoarthritis if not managed appropriately.

     

    Posner and Green performed a review of various finger extensor tendon injuries. They emphasized that diagnosis is contingent on the understanding of the anatomy of the extensor apparatus as well as the ability to perform the appropriate physical exam. They note that the majority of acute injuries can be managed non-operative with immobilization alone whereas chronic conditions generally require surgical management.

     

    Illustration A is a video demonstration of the Elson's test.

     

    Incorrect Answers:

    Answer 1: Full-time DIP splinting for 4-6 weeks is the preferred treatment for Zone 1 extensor tendon injuries (mallet finger).

    Answer 3: MCP extension splinting is the management for Zone V extensor tendon injuries (sagittal band rupture).

    Answer 4: Dorsal blocking splint of the PIP joint is indicated for unstable PIP joint dorsal dislocations.

    Answer 5: While extension splinting of the PIP joint is the appropriate management of central slip injuries, the DIP should be allowed to have free ROM to prevent adhesions.

     

     

     

     

     

  83. A 30-year-old female falls onto an outstretched hand and hyperextends her fingers during a roller-derby pile-up. She is taken to an urgent care center where radiographs are obtained. A hand surgeon is consulted who reports that she has a simple dorsal metacarpophalangeal joint (MCPJ) subluxation of the index finger. What is the next best course of action?

    1. Wrist neutral position with axial traction on the digit

    2. Wrist flexion with distally and volarly directed pressure on the proximal phalanx

    3. Open surgical release of the volar plate through a volar approach

    4. Open surgical release of the volar plate through a dorsal approach

    5. Wrist extension with axial traction on the fingers and dorsally directed on the proximal phalanx

    CORRECT ANSWER: 2

     

    To reduce a dorsal MCPJ subluxation, the wrist should be flexed and pressure applied distally and volarly to the proximal phalanx. This maneuver will in effect slide the proximal phalanx and its attached volar plate over the metacarpal head into a reduced position.

     

    Simple dorsal subluxation of the MCPJ differs from complex dislocation in that the volar plate is draped over the MC head instead of being incarcerated within the joint. If direct axial traction is applied, a simple subluxation may be converted into a complex dislocation. In this case, a noose effect is created around the MC head by way of the flexor tendons (ulnarly) and lumbricals (radially). Complex dislocations require surgical release of the volar plate.

    Indicators of a complex dislocation include interposed sesamoid within the MCPJ, joint space widening, skin dimpling, metacarpal head prominence within the palm, and a resting posture with the digit held in extension without any ability to flex at the MCPJ and with deviation toward the central digits.

     

    Afifi et al. developed a cadaveric model of irreducible MCPJ dislocations. In all 6 specimens, the flexor tendons moved ulnar to the MC head and the radial digital nerve moved superficially. In the majority of cases, the lumbricals moved radially. The division of the volar plate was required for reduction.

     

    Bohart et al. described the dorsal approach for reducing complex dorsal MCPJ dislocations. In the 5 cases described, the volar plate was draped dorsally over the MC head and its division allowed reduction of the joint. The authors maintain that this approach is simple and avoids dissection of digital nerves.

    Incorrect answers:

    Answer 1: This attempt will draw the entire volar plate dorsally where it becomes folded between the proximal aspect of the proximal phalanx and the metacarpal head, converting a simple subluxation into an irreducible complex dislocation.

    Answers 3 and 4: A closed reduction attempt should be attempted before surgery is considered. Either a volar or dorsal approach could be used if irreducible.

    Answer 5: Wrist extension will tighten the flexors exacerbating the noose effect around the MC head.

     

     

     

  84. Regarding wide-awake zone 1 and 2 flexor tendon repair under local anesthesia, all of the following are true EXCEPT:

    1. Intraoperative active motion allows identification of repair gaps

    2. Assessment of tendon gliding reduces need for postoperative tenolysis

    3. Allows inspection of tendon gliding through the A2 pulley and active excursion

    4. Facilitates postoperative early active motion

    5. Epinephrine is contraindicated due to the risk of digital necrosis

      CORRECT ANSWER: 5

      Local anesthesia augmented with an appropriate dose of epinephrine is safe in finger and hand surgery. Wide-awake repair has several advantages, including intraoperative inspection of the repair and assessment of gliding through the A2 pulley and tendon sheath.

       

      Wide-awake flexor tendon repair is performed under tumescent local anesthesia using lidocaine with epinephrine. Advantages of epinephrine include a bloodless field without tourniquet, reduced sedation, and cost savings. The dose is usually 7mg/kg lidocaine and 1:100,000 epinephrine if <50cc is needed, although the lidocaine/epinephrine ratios and addition of bupivacaine can be modified on a case-to-case basis. Phentolamine, an alpha antagonist, is the antidote to epinephrine-induced vasoconstriction and can be injected at the same site at a dose of 1mg diluted in 5mL saline.

       

      Lalonde et al. reviewed wide-awake flexor tendon repair and early tendon mobilization in zones 1 and 2 using local lidocaine with epinephrine.

      Advantages include decreased rupture rates due to identification of repair gapping; confidence of repair given by intraoperative exam to allow early

      active range of motion; and decreased tenolysis due to repair and pulley adjustments to facilitate gliding. The authors discontinued the use of tourniquet, sedation, peripheral nerve blocks, and general anesthesia. They report decreased rupture and tenolysis rates in addition to allowing protected midrange active flexion movement after surgery.

       

      McKee et al. performed a prospective, randomized, triple-blind study to analyze the optimal time delay between epinephrine injection and incision to minimize bleeding. Twelve volunteers were injected with either 1% lidocaine with epinephrine or 1% plain lidocaine. Relative hemoglobin concentration of skin and soft tissues was then measured using spectroscopy. The authors conclude 25 minutes is the ideal time to wait after injection of local anesthetic with epinephrine for optimal visualization.

       

      Incorrect Answers

      Answer 1: Wide-awake repair helps surgeons identify gapping at the repair site intraoperatively by preserving active motion

      Answer 2: Intraoperative assessment of tendon gliding allows on-the-spot debulking and pulley division or venting, which reduces tenolysis rates Answer 3: Performing a repair under local anesthesia allows active finger motion and intraoperative inspection of tendon gliding and excursion Answer 4: Postoperative early active motion is facilitated by intraoperative examination demonstrating adequate strength of the repair without gapping

       

       

       

  85. A 25-year-old cowboy falls onto an outstretched wrist while roping a steer. He has immediate pain and inability to move his wrist. He is taken to a local urgent care center and radiographs are taken. The physician assistant at the urgent care center sends you the image in Figure A. What is likely the first wrist ligament injured during this particular injury pattern?

     

     

     

    1. Scapholunate

    2. Capitolunate

    3. Lunotriquetral

    4. Dorsal radiocarpal ligament

    5. Short radiolunate CORRECT ANSWER: 1

    Perilunar instability progresses in a typical fashion with injury first to the scapholunate (SL) ligament, followed by the capitolunate (CL) articulation, the lunotriquetral (LT) ligament, and finally the dorsal radiocarpal (DRC) ligament, allow the lunate to rotate and dislocate.

     

    Perilunate dissociation (lunate dislocation) is most often sustained by an axial load to an extended wrist. As many as one-quarter of cases are missed initially and another quarter present with acute carpal tunnel syndrome. Lesser arc injuries are purely ligamentous, following the classic pattern described above, while greater arc injuries include a combination of ligamentous injury and fractures of the radius, ulna, and/or carpus. Injury radiographs will demonstrate disruption of Gilula's lines, a shortened carpal height, lunate-capitate overlap with a triangular appearance of the lunate on the AP view.

    The lateral view will show a loss of collinearity of the radius, lunate and

    capitate, with the lunate positioned volar to the capitate. Prompt recognition and reduction are critical. Surgical management is always necessary and involves ligament repair, carpal stabilization with pinning, fracture fixation if necessary, and possible carpal tunnel release.

     

    Mayfield et al. analyzed the pathomechanics of progressive perilunar instability in a cadaveric model. The authors discussed that the mechanism of lunar instability was an extension, ulnar deviation, and intercarpal supination. They found that ligamentous disruptions occurred sequentially, beginning with the SL and ending with the DRC ligament. They noted the importance of understanding the mechanism of instability, as the reduction was achieved with intercarpal pronation, radial deviation, and palmar flexion.

     

    Garcia-Elias reviewed scapholunate instability and contends that the diagnosis is made primarily on physical exam. The author notes that static and dynamic radiographs are helpful to evaluate SL dissociation and that arthroscopy can aid if the diagnosis is still unclear. He discusses that treatment is based on the integrity of the dorsal SL ligament, its healing potential, the status of other scaphoid stabilizers, reducibility of malalignment, and the cartilage surface.

     

    Herzberg et al. reviewed a series of 166 perilunate dislocations. The authors found that 25% were initially missed, and that open injuries and those which had a delay in treatment experienced inferior clinical outcomes. Cases treated in a timely fashion demonstrated satisfactory clinical results but over one-half (56%) had post-traumatic arthritis at a 6-year follow-up. The authors underscored the importance of prompt diagnosis.

     

    Dunn et al. reviewed 40 perilunate dislocations in a military population. They noted that over one-quarter (27.5%) were missed initially. Furthermore, one-half (50%) presented with acute carpal tunnel syndrome, and 17.5% had persistent paresthesias. The authors cautioned that a high-demand patient population may experience inferior functional results due to a greater degree of postoperative limitation.

     

    Figure A is a lateral radiograph of a lunate dislocation (Mayfield stage IV). The short radiolunate ligament remains intact, tethering the lunate to the distal radius.

     

    Incorrect answers:

    Answer 2: The CL articulation fails during the second stage of perilunar instability.

    Answer 3: The LT ligament fails during the third stage of perilunar instability. Answer 4: The DRC ligament fails in the final stage of perilunar instability,

    which permits lunate rotation and dislocation.

    Answer 5: The short radiolunate ligament does not typically fail in the setting of perilunar instability.

     

     

     

  86. A 25-year-old right-handed IV drug user presents with left forearm cellulitis and osteomyelitis of the radius. After a course of IV antibiotics and debridement of the radius, the patient is left with a 10cm bone defect. The patient is able to kick his drug habit. You consider a bone flap to span the defect and reconstruct the radius. Which artery will be used in the anastomosis for the required bone flap?

    1. Peroneal

    2. Longitudinal branch of the descending geniculate

    3. Transverse branch of the descending geniculate

    4. Posterior radial collateral

    5. Medial sural CORRECT ANSWER: 1

    A free vascularized fibula is required for the lengthy bone defect and it is based on the peroneal artery.

     

    The free vascularized fibula, supplied by the 1.5-4mm diameter peroneal artery, also carries two venae and can be raised with a skin paddle. The pedicle length is roughly 4-6cm, and the distal 5-6cm of bone is typically retained to preserve the ankle syndesmosis. This graft is ideal for managing larger defects (>6cm) in trauma, after debridement of infection, in the setting of congenital anomalies, or for avascular necrosis.

     

    Noamen reviewed surgical outcomes following a free vascularized fibula bone flap in 16 patients who had upper extremity bony defects, primarily following debridement of osteomyelitis of the forearm. The authors found that almost all patients (15/16) went on to union at an average of 3.5 months, and that there was no recurrence of infection.

     

    Adani et al. reviewed 12 cases of free vascularized fibula used in reconstruction following trauma. They noted that the size of the bony defect ranged from 6 to 13cm. At the time of final follow-up, the authors found that 8 of the 12 cases went on to union. They underscored the utility of this graft for large segmental defects of the forearm.

    Incorrect Answers:

    Answer 2: The longitudinal branch of the descending geniculate artery (DGA) is the supply for the medial femoral condyle flap, which may be used for scaphoid fracture non-unions, but cannot supply the length of bone required in this patient.

    Answer 3: The transverse branch of the DGA is the supply for the medial femoral trochlea flap, which may be used for scaphoid proximal pole nonunions, but also is insufficient for the length of bone required in this patient. Answer 4: The posterior radial collateral artery is the supply for the lateral arm flap, which may be raised with a small piece of bone, but cannot be used for a large diaphyseal defect.

    Answer 5: The medial sural artery supplies the medial gastrocnemius rotational flap, which is not used to address bony defects.

     

     

     

  87. A 23-year-old recreational rugby player gets his right index finger caught on a jersey while trying to make a tackle and feels immediate pain. The next day he presents to a hand surgeon. On exam he is unable to flex the distal interphalangeal joint of his right index finger. The hand surgeon advocates early repair of the injured tendon. The degree of urgency for fixation is most dependent on:

    1. Presence of distal phalanx avulsion fracture

    2. Size of potential bone fragment from avulsion

    3. Degree of flexor tendon retraction

    4. Proximal interphalangeal joint (PIPJ) range of motion

    5. Level of athletic activity

      CORRECT ANSWER:

      3

      A flexor digitorum profundus (FDP) tendon retracted into the palm shears the vincula, the tendon's blood supply. In these cases, urgent surgical repair is indicated. On physical exam, a mass in the palm may be palpated or this degree of proximal retraction may be determined on an MRI.

       

      Zone 1 FDP avulsion is described by the Leddy and Packer classification. Type I avulsions indicate FDP retraction into the palm. In these injuries, the vincula are disrupted and require urgent repair. Type II, the tendon stump retracts to the level of the PIPJ and some vincular blood supply is preserved and the tendon may be repaired up to 6 weeks after injury. In Type III injuries, a bone fragment is attached to the FDP. Because of the bone fragment, the FDP can only retract to the level of the distal aspect of the A4 pulley. Repair of a zone III injury can be completed by internal fixation with a screw or a k-wire.

      Freilich reviews the evaluation and treatment of jersey finger and pulley injuries in athletes. These not infrequent injuries are difficult to both diagnose and treat and are further complicated by often in-season injuries in athletes who expect an immediate return to play. Prompt recognition and treatment are necessary to prevent permanent disability to the hand.

       

      Ruchelsman et al. reviewed FDP avulsion injuries and highlighted how the FDP may retract into the palm, shearing the tendon's vascular supply. No repair technique is necessarily superior. However, gapping may be seen with pullout suture repairs (which are performed with a dorsal button) and suture anchors may fail by pulling out of the distal phalanx. Distal interphalangeal joint arthrodesis is a reasonable option.

       

      Incorrect answers:

      Answer 1: Degree of retraction, not necessarily the presence or absence of an avulsion fracture, may determine urgency.

      Answer 2: The size of the fracture fragment does not affect the vinacula and thus does not affect surgical timing. An FDP avulsion with a large fragment is unlikely to displace proximally, however.

      Answer 4: For stage II reconstruction, it is important to obtain adequate gliding and PIPJ range of motion. However, for acute jersey fingers, PIPJ motion is likely limited secondary to pain.

      Answer 5: Proximal retraction into the palm is urgent regardless of the patient's athletic prowess.

       

       

       

  88. A 56-year-old man is assaulted while minding his own business. In the process of defending himself, the robber lacerates the medial aspect of the man's upper arm. At the hospital, hand surgery is consulted to address his wound, and a complete ulnar nerve laceration is identified. Although the man's ulnar nerve is directly repaired, nerve function never returns. At a later date, the hand surgeon discusses transferring the anterior interosseous nerve (AIN) to the deep ulnar motor fascicle in a supercharge end-to-side (SETS) procedure. Which of the following is a contraindication to this transfer?

    1. Scarce fibrillation potentials

    2. Intrinsic atrophy

    3. Decreased preoperative sensory nerve action potentials

    4. Present preoperative compound muscle action potentials

    5. Absent preoperative compound muscle action potentials

    CORRECT ANSWER: 5

     

    Lack of compound muscle action potentials (CMAP) on nerve conduction studies (NCS) reflects prolonged and severe denervation and likely inability of a muscle to become reinnervated even after axonal regeneration. Motor axons are indirectly assessed by the amplitude, expressed in millivolts, of CMAPs after nerve stimulation during a NCS.

     

    The SETS procedure involves the transfer of the terminal aspect of the AIN from the muscle belly of the pronator quadratus (PQ) to the ulnar motor branch proximal to the wrist. Thus partial neuronal regeneration is stimulated at the expense of an expendable nerve (terminal AIN). The technique can be utilized for a variety of ulnar nerve injuries with the intention of augmenting recovery and preserving motor endplates of the intrinsic muscles in the hand. Of note, 15% of patients may have a Martin-Gruber communication between the median and ulnar motor nerves. This means that muscles that are normally innervated by the ulnar nerve will be innervated by the median nerve. As a result, these patients may have some function spared after a high ulnar nerve injury.

     

    Davidge et al. reviewed 55 patients who underwent SETS transfer for ulnar nerve palsy. Preoperatively, all patients had intrinsic weakness. The authors found that first dorsal interosseous strength, key pinch strength, grip strength, and DASH all improved postoperatively. They noted that absent CMAPs predicted significantly poor intrinsic muscle recovery.

     

    Woo et al. reviewed the management of ulnar nerve injuries. The authors underscore the devastating nature of high ulnar nerve injuries and advocate early exploration and neurorrhaphy. In addition, they conclude that consideration should be given for distal nerve transfers (SETS) in the setting of high ulnar nerve injuries.

     

    Barbour et al. review the indications, rationale, technique, and therapy for SETS. The authors recommended the transfer for the patients with fibrillations and motor unit potentials on EMG. The authors describe their 12-step surgical procedure which includes a Guyon canal release. The ulnar motor nerve at the wrist can be found in between the dorsal cutaneous and volar sensory components of the ulnar nerve. The authors have not experienced any patients with pronation deficits postoperatively.

     

    Incorrect answers:

    Answer 1: This indicates a mild nerve compression, and therefore a transfer would still be indicated.

    Answer 2: Intrinsic atrophy is an indication to perform the transfer.

    Answer 3: SNAPs are often affected before CMAPs, and thus their decreased amplitude may not indicate a chronic injury. In addition, decreased SNAPs have not been correlated with worse outcomes following SETS transfer.

    Answer 4: This would indicate that the endplates have not depolarized and a transfer is still indicated.

     

     

     

  89. A 24-year-old male computer programmer injures his wrist two years ago while LARPing. At the time he was diagnosed with a scaphoid fracture but he was finishing school and did not seek treatment. He presents to your office two years later. Figure A is the patient's X-Ray from your clinic. Advanced imaging confirms the cystic nature of the fracture site. To aid in fracture healing, you elect to harvest a vascularized graft and an intraoperative photo is seen in Figure B. What is the pedicle most often supplying this graft?

     

     

     

     

     

     

    1. Longitudinal branch of the descending geniculate artery

    2. Transverse branch of the descending geniculate artery

    3. Superomedial genicular artery

    4. 1,2 intercompartmental supraretinacular artery

    5. Peroneal artery CORRECT ANSWER: 1

    This patient has a scaphoid waist non-union. A medial femoral condyle (MFC) vascularized bone graft can supply this defect. The longitudinal branch of the DGA supplies the MFC.

     

    The scaphoid is nearly completely covered by articular cartilage, is entirely intra-articular, and has a retrograde blood supply that is poor. These factors

    pre-dispose scaphoid fractures to non-union. Many non-unions can be treated with cancellous graft and internal fixation (most frequently a single headless compression screw). However, in the presence of cystic changes or osteonecrosis at the fracture site, a vascularized graft is indicated. Pedicles can be raised from the distal radius or medial femoral condyle, where a large piece of cancellous bone can be raised from the periosteum.

     

    Jones et al. compared scaphoid non-unions treated with distal radius pedicle vascularized grafts versus MFC. The distal radius pedicled grafts (4/10) healed less reliable as compared to the MFC (12/12). The time to healing was also shorter in the MFC cohort. The authors concluded that MFC is the superior graft for scaphoid non-unions.

     

    Moon et al. review scaphoid fracture non-union management and underscore the importance of scaphoid healing to prevent carpal collapse. In the absence of osteonecrosis, internal fixation with non-vascularized bone graft is appropriate. However, although vascularized grafts are technically challenging they do improve union in the presence of osteonecrosis.

     

    Figure A is an AP X-Ray of a scaphoid non-union with sclerotic edges and cystic changes. Figure B is an intraoperative photo of an MFC graft on its pedicle.

     

    Incorrect answers:

    Answer 2: The transverse branch supplies the medial femoral trochlea (MFT) -which is used in proximal pole defects when a cartilage surface is needed.

    Answer 3: Superomedial genicular artery supplies the MFC in 11-20% of cases Answer 4: 1,2 ICSRA is harvested from the wrist and provides a much smaller and less robust graft.

    Answer 5: The peroneal artery supplies a free fibula graft.

     

     

     

  90. Collagenase Clostridium Histolyticum (CCH) is a FDA approved treatment for Dupuytren's disease. Neurovascular bundles are preserved during treatment due to the fact that CCH has limited effect on which type of collagen?

    1. I

    2. II

    3. III

    4. IV

    5. None of the above

    CORRECT ANSWER: 4

     

    CCH has little effect against type IV collagen, which forms the basement membrane of nerves and blood vessels.

     

    CCH is indicated for symptomatic cords causing >30 degrees of flexion contractures. Treatment with CCH typically entails a minimum dose of 10,000 units, suspended in 0.2-0.25mL, and injected with a 22-25g needle into pathologic cords. The hand is then manipulated 24-48 hours after the injection. Skin tears are common but usually asymptomatic. CCH is more effective against metacarpophalangeal joint (MCPJ) as compared to proximal interphalangeal joint (PIPJ) contracture. Although CCH treatment has revolutionized the management of Dupuytren's disease, there is some concern over potential increased surgical difficulty after a patient has received a CCH injection.

     

    Chung et al. present a review of the molecular mechanisms of collagenase. The authors describe how MMP-1, a prototypic collagenase, binds and unwinds the triple-helical collagen structure to permit bond hydrolysis. They conclude with the importance of understanding the underlying mechanisms in relation to the various biologic and disease processes.

     

    Gaston et al. prospectively evaluated 715 patients who received CCH injections for fixed flexion contractures (FFC) secondary to Dupuytren's disease present in two joints of the same finger. The authors found that at one month, FFCs decreased on average from 98 to 27 degrees, and range of motion improved.

    The clinical success rate in MCPJ contracture was 65% as compared to 29% for PIPJ contracture. The authors reported that the most common adverse events included skin tears (22%), edema, and pain. They recommended CCH as a treatment for Dupuytren's disease in those with 2 joints affected with FFC in the same finger.

     

    Desai and Hentz reviewed the treatment for Dupuytren's disease. The authors noted that treatment for Dupuytren's disease has historically been surgical, and limited palmar fasciectomy is currently the most common treatment option. However, they discuss that CCH is now an FDA approved treatment for Dupuytren's disease and provides an excellent non-surgical alternative.

     

    Incorrect answers:

    Answer 1: CCH is most effective against types I and III collagen.

    Answer 2: CCH has limited effect on type II collagen, HOWEVER this is not present in neurovascular structures but is instead the predominant type of collagen found in articular cartilage.

    Answer 3: CCH is effective against types I and III collagen. Type III collagen is

    furthermore the predominant type of collagen associated with Dupuytren's disease.

    Answer 5: CCH is effective against types I and III collagen, but not type IV.

     

     

     

  91. A 35-year-old female presents after a fall on an outstretched hand with the injury shown in figures A & B. What articulation is expected to develop osteoarthritis in the third stage of scapholunate advanced collapse (SLAC)?

     

     

     

     

     

     

    1. Radiolunate

    2. Capitolunate

    3. Radial styloid and scaphoid

    4. Entire radioscaphoid

    5. Lunotriquetral CORRECT ANSWER: 2

    Osteoarthritis resulting from scapholunate advanced collapse (SLAC) progresses in stages originally described by Watson: (1) radial styloid and scaphoid, (2) entire radioscaphoid, (3) capitolunate. A fourth stage involving the radiolunate articulation has since been described by other authors, although its association in SLAC wrist remains controversial.

     

    SLAC is a condition of progressive instability resulting from injury to the scapholunate (SL) ligament. Standard wrist PA radiographs reveal SL diastasis greater than 3mm while lateral radiographs reveal a dorsal intercalated segmental instability (DISI) deformity and subluxation of the capitate dorsally. Resultant scaphoid flexion and lunate extension create abnormal force distribution among the midcarpal and radiocarpal joints. This causes arthritis that advances in a predictable pattern. Surgical treatments include 4-corner fusion, capitolunate fusion, complete wrist fusion, proximal row carpectomy (PRC), radial styloidectomy, and wrist denervation.

     

    Watson et al. reviewed 210 wrist x-rays to establish the pattern of sequential changes in degenerative arthritis of the SLAC wrist in their 1984 landmark paper. Arthritis between the scaphoid, lunate and radius was most common (57%); second occurred between the scaphoid, trapezium, and trapezoid (27%); and a combination of these two patterns occurred in 15%. They noted sparing of the lunate-radius articulation. They concluded degenerative arthritis progresses in a predictable pattern beginning at the radioscaphoid articulation and advancing to the capitolunate articulation.

     

    Dr. Strauch reviewed SLAC and scaphoid nonunion advanced collapse (SNAC) patterns of wrist arthritis and treatment options. Surgical options include 4-corner fusion, capitolunate fusion, complete wrist fusion, proximal row carpectomy (PRC), radial styloidectomy, and wrist denervation. Four-corner arthrodesis is the classic surgical treatment using either k-wire or circular plate fixation. Alternatively, PRC has shown excellent results with the advantages of earlier motion, no need for fusion, and no hardware.

     

    Trehan et al. reviewed nomenclature and differential diagnosis for SLAC. The authors note pseudogout also affects the wrist in four progressive stages, termed scaphoid chondrocalcinosis advanced collapse (SCAC), with the final stage consisting of pancarpal arthritis involving the radiolunate articulation. The authors conclude clinicians should consider infection, idiopathic osteonecrosis (Kienböck’s disease, Preiser’s disease), crystalline arthropathy,

    inflammatory arthritis, scaphotrapezium-trapezoidal (STT) arthritis, and scaphoid nonunion as possible etiologies of wrist pain upon patient presentation.

     

    Figure A demonstrates widening of the scapholunate interval and the cortical ring sign caused by scaphoid flexion consistent with SL ligament disruption. Figure B demonstrates a DISI deformity and increased scapholunate angle.

     

    Illustration A measures the scapholunate angle at approximately eighty-seven degrees, normal SL values range from 30 to 60 degrees. Illustration B demonstrates Watson stage I SLAC. Illustration C demonstrates Watson stage

    II. Illustration D demonstrates Watson stage III.

     

    Incorrect Answers

    Answer 1: Osteoarthritis affecting the radiolunate articulation has been described as the fourth stage of SLAC, although pancarpal arthritis should alert the clinician to a different etiology of wrist arthritis

    Answer 3: Involvement of the radial styloid and scaphoid articulation is the first stage of SLAC

    Answer 4: Progression to the entire radioscaphoid articulation is the second stage of SLAC

    Answer 5: The lunotriquetral articulation is spared in SLAC

     

     

     

     

     

     

     

     

     

     

     

     

  92. A 25-year-old male injures his left index finger in a tortilla press at work. He is taken to the local teaching hospital where he is diagnosed with a transverse left index finger proximal phalanx fracture. The interossei and central slip work to deform the fracture in what manner, respectively?

    1. Proximal fragment extension, distal fragment radial deviation

    2. Proximal fragment extension, distal fragment extension

    3. Proximal fragment extension, distal fragment flexion

    4. Proximal fragment flexion, distal fragment flexion

    5. Proximal fragment flexion, distal fragment extension

    CORRECT ANSWER: 5

    In a transverse proximal phalanx fracture, an apex volar deformity often develops with proximal fragment flexion and distal fragment extension.

     

    Proximal phalanx fractures are difficult to treat secondary to displacement at the fracture site and the stiffness following operative treatment. Non-operative management is possible without rotation or angular displacement of the digit. However, less than 60% of active motion is maintained in younger patients following the non-operative management of proximal phalanx fractures. Soft tissue injury may further compromise soft digit mobility. The apex volar angulation at the fracture site, which can be difficult to manage in a closed manner, causes an extension lag at the PIPJ.

     

    Vahey et al. performed a cadaveric analysis on extensor lag following proximal phalanx fractures. After the typical apex volar fracture posture. The average slope was 12degrees of lag for every millimeter of bone-tendon discrepancy.

    In a simulated apex palmar displacement angulation of 16 degrees, the PIPJ lagged by 10 degrees. The authors underscore the importance of establishing the bone-tendon relationship following proximal phalanx fractures.

     

    The Meals family comprehensively review proximal phalanx fractures. Minimally displaced extra-articular fractures in compliant patients can be buddy taped for four weeks. Unstable fractures, intra-articular fractures (especially longitudinal unicondylar fractures of the proximal phalanx head) should be treated with fixation. Fixation allows for earlier range of motion which is critical to prevent stiffness. Open reduction internal fixation allows for immediate ROM a the risk of violating the extensor mechanism. The soft tissue sequelae of proximal phalanx fractures may be more significant than the

    boney injury.

     

    Kozin et al. reviewed the operative management of proximal phalanx fractures. These can be treated closed with percutaneous pinning or open reduction internal fixation. Volar, Dorsal, or lateral approaches may be used for ORIF. Interfragmentary screws alone can be used in oblique fractures. Soft tissue trauma should be limited as much as possible to optimize range of motion postoperatively.

     

    Incorrect answers:

    Answer 1: It is possible to develop a rotational deformity, but the classic deforming forces typically cause an apex volar position.

    Answer 2: The interossei pull the proximal fragment into flexion, while the central slip pulls the distal fragment extends from the pull of the central slip. Answer 3: This would represent an apex dorsal deformity, which is the opposite of the typical displacement.

    Answer 4: The interossei pull the proximal fragment into flexion, while the central slip pulls the distal fragment extends from the pull of the central slip.

     

     

     

  93. A lumberjack in rural Michigan falls 10 feet from an Evergreen branch onto an outstretched arm and develops immediate wrist pain. He is not able to see a physician for 4 months. When he finally does, he is diagnosed with a perilunate dislocation and indicated for a Proximal Row Carpectomy (PRC). Figure A is an intraoperative photo. The black dot in the photo is the capitate. The instrument touches a structure that prevents ulnar translocation of the carpus after a PRC. What is this structure?

     

     

     

    1. Short Radiolunate ligament (SRL)

    2. Lunotriquetral ligament (LTL)

    3. Radioscapholunate ligament (RSL)

    4. Scapholunate ligament (SL)

    5. Radioscaphocapitate ligament (RSC)

    CORRECT ANSWER: 5

    The RSC prevents ulnar translation of the carpus after a PRC.

     

    The RSC is a volar carpal ligament originating at the radial styloid and is positioned radial to the long and short radiolunate ligaments. The RSC runs obliquely and is a taught and stout ligament. It can be disrupted with too large of a radial styloidectomy, trauma, or iatrogenic injury during dissection. Its disruption destabilizes the carpus and ulnar carpal translocation inevitably occurs. In these cases, patients will have poor outcomes following a PRC and a total wrist fusion may be required.

     

    Green et al. review indications, techniques, and complications of the PRC. The PRC is indicated in arthritic conditions, Kienbock disease, or in a chronic perilunate injury. The scaphoid, lunate, and triquetrum are excised through a dorsal approach. The authors underscore the importance of identifying and

    preserving the stout RSC to maintain carpal stability. Progressive arthritic development after a PRC may indicate a total wrist fusion. Injury to the RSC will induce ulnar translation and lead to a poor outcome.

     

    Nakamura et al. performed a biomechanical stability analysis on the wrist with increasing size of an osteotomy of the radial styloid, the site of origin for the RSC. After 6mm of radial styloidectomy, the carpus was destabilized and ulnar translocation ensued. The authors recommend a radial styloidectomy of no more than 4mm to preserve the RSC and limit ulnar carpal translocation.

     

    Figure A demonstrates an intraoperative photo of a PRC through a dorsal incision. The black dot indicates the capitate and the forceps are touching the stout RSC.

     

    Incorrect answers:

    Answer 1: SRL stabilizes the lunate to the radius and is not typically injured in perilunate dislocations.

    Answer 2: LTL disruption causes a VISI deformity.

    Answer 3: The ligament of Testut is a neurovascular conduit with little structural importance.

    Answer 4: The SL is an important stabilizing structure in the carpus and prevents

    dorsal intercalated segment instability (DISI) posture.

     

     

     

     

  94. A 35-year-old software engineer sustains an avulsion of his nail plate, eponychial fold, and 5mm of dorsal skin about his right index finger during an American Civil War battlefield reenactment. Which of the following surgical procedures would best cover this defect?

    1. Reverse cross finger flap

    2. Cross finger flap

    3. First dorsal metacarpal artery (FDMA) flap

    4. V-Y advancement

    5. Moberg advancement flap

    CORRECT ANSWER: 1

    A reverse cross finger flap (RCFF) would best cover a dorsal fingertip wound involving the eponychial fold.

     

    The RCFF was initially described to cover a dorsal PIPJ wound. The indications

    now include dorsal fingertip wounds, particularly those including the eponychial fold and nailbed defects with an exposed tendon. Ideal donor areas include the dorsal aspect of the middle and proximal phalanges of the adjacent fingers. The hand is typically immobilized for 2 weeks at which time the flap is divided and range of motion is initiated.

     

    Atasoy describes the original technique for the RCFF. The indications include avulsions of eponychial skin folds or coverage of large nail bed defects with exposed bone. The author reported that the technique was successful in forming new eponychial skin folds with nail growth. He noted that the best donor sites were the dorsal aspects of the middle and proximal phalanges.

    Atasoy noted that the surgeon should avoid using the dorsum of DIP and PIP joints, because the flap at these locations may be too thin to elevate.

     

    Atasoy later updated his original paper and expanded the indications to include coverage of extensor tendons without paratenon and burns. He noted that the flap should be designed to be 5mm wider than the defect. The authors discusses the optimal technique, in which a thin skin flap is elevated to expose the subdermal vascular plexus. This fat flap is elevated with a base opposite to the skin flap and reversed over the defect. Finally, a thin full-thickness skin graft is used to cover the fat flap. He notes that a skin graft is not applied to cover a defect of the germinal matrix, as this would impair nail growth.

     

    Illustration A demonstrates the surgical technique of the RCFF. Note that that the base of the skin flap and the fat flap are opposite one another.

     

    Incorrect Answers:

    Answer B: A cross-finger flap would best cover a VOLAR fingertip wound. Answer C: A FDMA flap would be best suited to cover a dorsal THUMB wound. Answer D: A V-Y advancement would best cover a fingertip wound with a relatively small defect. It could not be advanced over the phalanx to cover a dorsal eponychial defect.

    Answer E: A Moberg advancement flap is used for coverage of thumb tip defects.

     

     

     

     

     

  95. A 26-year-old left-hand dominant male sustained a gunshot wound to the left forearm approximately 2 years prior. At that time, he was diagnosed with an ulnar nerve palsy. He was subsequently lost to follow-up and now presents desiring improved left-hand finger function. Based on his examination (Figure A), you suggest tendon transfers to improve his hand function. Which of the following correctly describes the physical examination maneuver to determine tendon transfer location?

     

     

     

    1. Flex PIP joint to 90 degrees and DIP extension is rigid

    2. PIP joints flex easily with MCP joints extended but not with MCP joints flexed

    3. Passive wrist extension results in loss of DIP flexion in the small finger

    4. Full MCP extension is blocked and PIP joint extension does not occur

    5. Thumb interphalangeal joint flexion while attempting to hold a piece of paper

    CORRECT ANSWER: 4

     

    This patient has an ulnar nerve injury based on the clinical image and has lost the function of his intrinsics in his hand. To properly determine tendon transfer location one must perform the Bouvier test, which is correctly described by blocking full MCP extension and PIP joint extension does not occur.

     

    Intrinsic minus hand or ulnar claw hand is a result of ulnar nerve injury. In this particular scenario, this patient has lost intrinsic hand function. A physical examination test to guide tendon transfer procedure is the Bouvier test. The Bouvier test is performed by holding the wrist in neutral and blocking full MCP joint extension and seeing if the patient can extend the PIP joints. If the patient is able to extend the PIP joints, then it is considered positive. A positive test indicates simple claw hand and an intact central slip. Tendon transfer procedures are then aimed at re-creating MCP flexion. To correct this, the tendons are either attached to the A1 pulley, A2 pulley or proximal phalanx. However, if the patient is unable to extend the PIP joints, then the test is considered negative. A negative test indicates complex claw hand and tendon transfers must correct both MCP flexion and PIP joint extension via insertion into the lateral bands.

     

    Sammer and Chung discuss the pertinent anatomy and function of the median and ulnar nerves in the forearm and hand. They discuss the different clinical deficits associated with median and ulnar nerve injuries based on their

    location. They describe various treatments including their risks, benefits, indications and post-operative care.

     

    Burkhalter and Strait in 1973 described a metacarpophalangeal flexor replacement for loss of instrinsic hand function. They found that it increased grip strength, reduced clawing and improved flexion pattern. They concluded that the procedure was biomechanically sound and can be done in the early course of ulnar nerve injury.

     

    Muzykewicz et al. compared the Zancolli-lasso and the House procedure in 12 cadaver hands. They found that the Zancolli-lasso procedure resulted in finger IP joint flexion then MCP joint flexion and the House procedure resulted in MCP joint flexion then IP joint flexion. They concluded that both procedures improved hand grasp and fingertip-to-palm distance, but the House procedure restored function similarly to an intrinsic-activated hand. Subsequently, the House procedure was more recommended for restoring intrinsic function in tetraplegic patients.

     

    Figure A is a clinical photograph depicting an intrinsic minus or ulnar claw hand.

     

    Illustration A is a clinical photograph depicting the Bouvier Test. The wrist is placed in neutral and full MCP joint extension is blocked. If the patient can extend the PIP joint (positive test) then the central slip is intact and indicates simple claw hand. The inability to extend the PIP joints (negative test) indicates complex claw hand.

     

    Incorrect Answers:

    Answer 1: This describes the Elson test, which is used to diagnose central slip injuries.

    Answer 2: This describes Extrinsic Tightness.

    Answer 3: This is describing the tenodesis effect, which can be used to identify flexor or extensor tendon injuries. In this example, there is likely a flexor digitorum profundus injury.

    Answer 5: This describes Froment’s sign, which is important to identify ulnar nerve pathology, but does not guide tendon transfer location for claw fingers.

     

     

     

     

     

  96. A 42-year-old male laborer sustains the injury shown in Figure A. He is taken to the operating room and found to have a partial laceration of the median nerve. Figure B shows a cross-sectional image of a nerve. Which of the following represents item D?

     

     

     

     

     

     

     

    1. Epineurium

    2. Perineurium

    3. Nerve fiber

    4. Myelin

    5. Endoneurium

    CORRECT ANSWER: 2

     

    Item D in Figure B represents the perineurium, the fibrous tissue covering fascicles.

     

    The basic building blocks of nerves are the axons of the individual nerve cells (neurons). The axons are covered by a myelin sheath that is produced by Schwann cells and speeds nerve conduction. These myelinated axons are bundled together to form a fascicle. The connective tissue and capillary network that supports the nerve fibers within each fascicle is called the endoneurium. Fascicles are grouped together into another connective tissue network, the perineurium, which provides mechanical strength to the nerve. Multiple groups of nerve fascicles are surrounded and sheathed by the epineurium, which provides support and vascularity.

     

    Lee et al. published a review on peripheral nerve injury and repair. They review the cross-sectional anatomy of a peripheral nerve, which includes the epineurium, perineurium, fascicle, endoneurium, and axon. They also review the classification, treatment, and outcomes of nerve injuries. They report that outcomes of nerve repair to date have been no better than fair, with only 50% of patients regaining useful function.

     

    Figure A is a clinical image of a deep laceration to the volar aspect of the proximal forearm. Figure B is an image demonstrating the cross-sectonal anatomy of a peripheral nerve, including the nerve fiber (A), myelin (B), epineurium (C), perineurium (D), endoneurium (E), and node of Ranvier (F).

     

    Incorrect Answers:

    Answer 1: The epineurium is indicated by Item C. Answer 3: The nerve fiber is indicated by Item A. Answer 4: Myelin is indicated by Item B.

    Answer 5: The endoneurium is indicated by Item E.

     

     

     

     

  97. A 16-year-old male presents with a gunshot wound to his right upper arm. On examination, he has a wrist drop and reports loss of sensation in his radial nerve distribution. There is a 1.5cm entrance wound on the lateral aspect of the arm with a 6.8 cm stellate exit wound anteromedially. Figures A and B are the radiographs of the injury. The patient is taken to the operating room for fixation, and the

    radial nerve is found to be completely transected. When is the optimal timing for definitive nerve repair?

     

     

     

     

    1. At the time of initial debridement

    2. At the time of definitive fracture fixation

    3. After 3 months from the injury

    4. After 1-3 weeks from the injury

    5. Once a neuroma has formed on MRI

    CORRECT ANSWER: 4

    The patient is presenting with a completely transected radial nerve after a gunshot wound (GSW) to the upper arm. Given the presence of neurotmesis and extensive soft tissue damage from cavitations created by the projectile, the optimal timing for assessment of the nerve is 1-3 weeks from the injury.

     

    Gunshot wounds are the second most common cause of death in patients under the age of 24 years old (the first being motor vehicle accidents).

    Management of these injuries involves bedside irrigation and antibiotics, with a vast majority of GSWs to the extremity being treated non-operatively. Surgical indications include fractures, vascular injuries, and nerve transections. Both direct tissue injury from the projectile and shockwaves into the surrounding

    tissue cause damage. Most nerve deficits can be monitored for up to three months if there is no evidence of nerve transection. In the setting of a transected nerve, the optimal time to definitive repair is within 1-3 weeks from injury in order to allow time for the zone of injury to declare itself.

     

    Hofmeister et al. reviewed the treatment of battlefield injuries from a military medicine standpoint during Operation Iraqi Freedom. The authors describe the multiple levels of care on the battlefield with level 1 and level 2 care consisting of basic stabilization with preparation to transfer to a higher level of care. With regards to nerve injuries, the authors recommend delayed treatment to ensure adequate soft tissue closure and declaration of the zone of injury.

     

    Carter et al. reviewed the treatment of gunshot wounds in pediatric patients. The authors described the mechanism by which projectiles cause damage to tissue by a permanent cavity (directly from the projectile) and the temporary cavity (from the shock wave). When nerves are transected (injury from the permanent cavity) there is further damage from the temporary cavity. From this, the authors recommend waiting 1-3 weeks from the injury to definitive surgical repair to allow full consideration of the extent of the injury.

     

    Figures A is an AP radiograph of the humerus with a humeral shaft fracture after a GSW. Figure B is a lateral radiograph of a humeral shaft fracture after a GSW. Video V is a demonstration of the shockwave and cavitary damage that occurs with projectiles passing through soft tissue.

     

    Incorrect Answers:

    Answer 1: Repair of nerve injuries at the time of initial debridement is not recommended as there is a large area of injury from GSW due to shock wave formation in the tissues. As such, adequate time is recommended for necrotic tissue to declare itself.

    Answer 2: A fracture such as this may need to be stabilized before the true extent of the injury is appreciated. As such, nerve repair may be done as a staged procedure after definitive fracture fixation.

    Answer 3: In the setting of a closed fracture of the humerus or no findings suggestive of nerve transection, a radial nerve palsy can be monitored for 3 months. Most nerve injuries in this setting are neurapraxias that resolve with observation.

    Answer 5: Waiting for neuroma formation is currently not recommended as an indication for nerve repair.

     

     

  98. When performing wrist arthroscopy, nerve injury may occur during portal placement. Which of the following combinations incorrectly pairs the portal and the nerve at risk?

    1. 1U; superficial sensory branch of the radial nerve

    2. 1-2; superficial sensory branch of the radial nerve

    3. 4-5; dorsal sensory branch of the ulnar nerve

    4. 6U; dorsal sensory branch of the ulnar nerve

    5. 6R; dorsal sensory branch of the ulnar nerve

    CORRECT ANSWER: 3

    The dorsal sensory branch of the ulnar nerve is not at risk during the establishment of the 4-5 portal.

     

    Complications of wrist arthroscopy occur in 1-2% of patients. The superficial sensory branch of the radial nerve is located 16mm from the 3-4 portal and may be injured during placement of the 1-2, 1U, and 1R portals. The dorsal sensory branch of the ulnar nerve is located 8mm from the 6R portal and may be injured during placement of the 6U and 6R portals.

     

    Shyamalan et al. performed a cadaveric study and systematic review of the structures at risk during wrist arthroscopy. They found that the dorsal sensory branch of the ulnar nerve was found to be at risk by performing the 6U, 6R and ulnar midcarpal (MCU) portals; the sensory branch of the radial nerve by the 1-2 and 3-4 portals; and the posterior interosseous nerve by the 3-4 and 4-5 portals.

     

    Longo et al. performed a study to determine the safety of dorsal wrist arthroscopy portals using MRI. They found that the 1-2 and 6U portals likely increase the risk of neurovascular damage because of their proximity to neurovascular structures. The 3-4 and 4-5 portals, on the other hand, are the farthest away from the neurovascular structures and likely reduce the risk of damage to neurovascular structures.

     

    Illustration A is a diagram depicting portal locations and their relationship to the superficial sensory branch of the radial nerve and dorsal sensory branch of the ulnar nerve.

     

    Incorrect Answers:

    Answers 1, 2, 4, and 5: Each of these combinations are correct. The superficial sensory branch of the radial nerve may be injured during placement of the 1-2, 1U, and 1R portals, while the dorsal sensory branch of the ulnar nerve may be injured during placement of the 6U and 6R portals.

     

     

     

     

     

  99. A 51-year-old man complains of elbow pain and numbness into the hand. He is an electrician and enjoys playing outfield in his competitive softball league, and was a minor league baseball pitcher. He has a multiyear history of numbness and tingling into his ring and small fingers. On physical exam, he has decreased 2-point discrimination in his small finger and a positive Jeanne's sign. His active elbow range of motion is 0-120 degrees with full pronosupination, but flexion elicits a snapping sensation over his medial elbow. His radiographs are shown in figure A. What is the best treatment for this problem?

     

     

     

    1. Arthroscopic elbow debridement

    2. In situ cubital tunnel decompression

    3. Medial ulnar collateral ligament reconstruction

    4. Cubital tunnel decompression with anterior transposition

    5. Counter-force bracing CORRECT ANSWER: 4

    This patient has a long history of compressive ulnar neuropathy with ulnar nerve subluxation, as such, he should be treated with cubital tunnel decompression and anterior ulnar nerve transposition.

     

    Cubital tunnel syndrome is the second most common compressive neuropathy involving the upper extremity. A number of sites within the region of the cubital tunnel can contribute to ulnar nerve compression, including the arcade of Struthers, the intermuscular septum, the flexor carpi ulnaris fascia, the anconeus epitrochlearis, the Osborne ligament, and fascial bands within the flexor carpi ulnaris distally. The superficial position of the ulnar nerve at the cubital tunnel and the increase in tension and traction that it experiences with elbow flexion combine to make it susceptible to compression neuropathy.

    Subluxation of the ulnar nerve is a result of increased traction across the elbow. Anterior transposition of the ulnar, either subcutaneous or submuscular, work to decrease the distance the ulnar nerve needs to travel to the hand, thus reducing traction and symptoms related to this.

     

    Nelson et al. reviewed 417 arthroscopic elbow procedures for early complications over a 13-year period. Overall there was a 14% complication rate. The most common complications were elbow stiffness, heterotopic ossification, infection, and transient ulnar neuritis. The rate of infection was increased with intraoperative steroid injections.

    Eberlin et al. review the history of ulnar neuropathy since its first description in 1817. Initial treatment of cubital tunnel syndrome focused on complete transection of the nerve at the level of the elbow, resulting in initial alleviation of pain but significant functional morbidity. A number of subsequent techniques have been described including in situ decompression, subcutaneous transposition, submuscular transposition, and most recently, endoscopic release.

     

    Zimmerman et al. retrospectively evaluated 142 patients surgically treated for cubital tunnel syndrome with decompression and anterior submuscular transposition. They concluded that submuscular transposition is a safe and durable option for primary ulnar neuropathy at the elbow. Overall, good or excellent results were achieved in 89% of patients with a low complication rate.

     

    Figure A shows plain radiographs of an elbow demonstrating olecranon osteophytes and calcification of the UCL consistent with valgus extension overload. Illustration A is Jeanne's sign of ulnar neuropathy which is MCP hyperextension with attempted pinch grip as a result of a denervated adductor pollicis.

     

    Incorrect answers:

    Answer 1: The patient has no limitation in range of motion, as such, arthroscopic debridement would be of limited benefit.

    Answer 2: In situ decompression alone would likely be inadequate as the patient demonstrates ulnar nerve instability.

    Answer 3: The patient does not demonstrate signs or symptoms of posteromedial elbow instability, therefore mUCL reconstruction would not be indicated.

    Answer 5: Counter-force brace is generally used for medial or lateral epicondylitis which this patient does not exhibit.

     

     

     

     

     

  100. A 32-year-old man sustains a traumatic amputation of his right thumb (Figure A). He undergoes the soft-tissue coverage procedure shown in Figure B. What artery does this flap rely on, and which nerve is at risk during this procedure?

     

     

     

     

     

     

    1. Medial femoral circumflex artery; femoral branches of genitofemoral nerve

    2. Medial femoral circumflex artery; lateral femoral cutaneous nerve

    3. Superficial circumflex iliac artery; femoral branches of genitofemoral nerve

    4. Superficial circumflex iliac artery; lateral femoral cutaneous nerve

    5. Superficial circumflex iliac artery; ilioinguinal nerve

    CORRECT ANSWER: 4

    Soft-tissue coverage of this patient's injury was achieved using a groin flap, which relies on the superficial circumflex iliac artery. The lateral femoral cutaneous nerve is at risk during this procedure.

     

    Groin flaps are axial flaps that are useful for providing soft-tissue coverage to the upper extremity. They are based on the superficial circumflex iliac artery. Caution must be taken when raising these flaps in order to avoid injury to the lateral femoral cutaneous nerve. The lateral femoral cutaneous nerve pierces the iliac fascia below the inguinal ligament, medial to the anterior superior iliac spine, and lies directly in the field of dissection when a groin flap is raised.

     

    Chuang et al. published a review on groin flap design and versatility. They report that the groin flap is a reliable and well-established reconstructive option for pedicled or free-tissue transfer. They discuss the "rule of two finger widths", a simplified guideline used to overcome concern regarding the flap's variable vascular origin, which is based on the transverse diameter of the patient's index and long fingers at the distal interphalangeal level.

     

    Yannascoli et al. published a review on the management of soft tissue defects of the hand. They report that groin flaps provide a large amount of soft tissue, offer the advantage of a conspicuous donor site, and are one of the thinnest options available.

     

    Figure A is a clinical image of a traumatic left thumb amputation. Figure B is a clinical image of a groin flap. Illustration A is a diagram depicting the anatomy of a groin flap.

     

    Incorrect Answers:

    Answers 1, 2, 3, and 5: Groin flaps are based on the superficial circumflex iliac artery and place the lateral femoral cutaneous nerve at risk.

     

     

     

     

     

  101. A 55-year-old patient presents with numbness and pain in the right ring and small fingers. He reports that his symptoms are worse at night. On examination, he has decreased sensation on the dorsal ulnar distribution of the hand with a positive Tinel sign at the medial elbow. There is no palpable subluxation at the medial elbow with flexion and extension. Electromyography from 3 months ago demonstrated decreased nerve conduction velocities in the ulnar nerve. Figures A and B demonstrate the radiographs of the right elbow. Symptoms continue to worsen despite nighttime extension splinting and NSAIDs. What is the best next step in treatment and the most likely site of compression for the patient?

     

     

     

     

     

     

    1. Repeat EMG; herniated nucleus pulposus

    2. In situ cubital tunnel release; flexor carpi ulnaris aponeurosis

    3. Medial epicondyle ostectomy; osteophyte

    4. Anterior submuscular transposition; anconeus epitrochlearis

    5. Anterior subcutaneous transposition; ligament of struthers

    CORRECT ANSWER: 2

    The patient has idiopathic cubital tunnel syndrome without evidence of an osteophyte at the medial elbow or ulnar nerve subluxation. The best treatment for this would be an in situ release of the cubital tunnel.

     

    Cubital tunnel syndrome presents with numbness and pain in the ring and small finger as well as intrinsic weakness of the hand. Compression most commonly occurs between the two heads of the FCU, with other possible sources including osteophytes, anconeus epitrochlearis, Osborne's ligament, intermuscular septum, Arcade of Struthers, heterotopic ossification from trauma or prior surgery, and ganglion cysts. Treatment consists of nighttime bracing and NSAIDs followed by surgical decompression if conservative treatment fails.

     

    Biggs et al. performed a randomized control trial of 44 patients undergoing in situ decompression versus submuscular transposition for idiopathic cubital tunnel syndrome. The authors found that there was no significant improvement with regards to functional recovery between the two groups, but there was a higher rate of wound complications, including deep infections, in the transposition group. The authors recommend isolated in situ decompression without transposition for the treatment of idiopathic cubital tunnel syndrome.

     

    Zlowodzki et al. performed a meta-analysis of four randomized control trials comparing in situ decompression to submuscular transposition for cubital tunnel syndrome. The authors found that there were no differences in motor conduction velocities and clinical outcomes scores between the two treatments. The authors recommend in situ cubital tunnel release as a reasonable alternative to transposition for cubital tunnel syndrome.

     

    Figures A and B demonstrate a normal AP and lateral radiograph of the right elbow. Illustration A depicts the course of the ulnar nerve at the level of the elbow with sites of compression at the cubital tunnel.

     

    Incorrect Answers:

    Answer 1: The patient has symptoms consistent with cubital tunnel syndrome. A repeat EMG in this patient will not demonstrate any new findings as the diagnosis is apparent clinically. A new EMG would be warranted if the patient developed new symptoms not consistent with cubital tunnel syndrome.

    Answer 3: Medial epicondyle ostectomy would be warranted if the patient had evidence of an osteophyte or heterotopic ossification on radiographs.

    Answer 4: An anterior submuscular transposition is an option for the treatment of idiopathic cubital tunnel syndrome; however, the literature suggests doing a transposition only in revision cases or when there is evidence of ulnar nerve

    instability. Anconeus epitrochlearis is a possible cause for cubital tunnel syndrome, but it is not the most common.

    Answer 5: An anterior subcutaneous transposition is not recommended for the treatment of idiopathic cubital tunnel syndrome as it has been associated with increased complications with equivalent outcomes. The ligament of Struthers has been implicated as the cause of pronator syndrome.

     

     

     

     

     

     

  102. A 22-year-old man sustained the injury shown in Figures A and B following an MVC. He underwent closed reduction and casting in the emergency department. Figures C and D show his radiographs in the clinic 1 week later. Which ligament attaches to the fracture fragment responsible for the loss of reduction in this patient?

     

     

     

     

     

     

    1. Scapholunate ligament

    2. Radioscaphocapitate ligament

    3. Short volar radiolunate ligament

    4. Lunotriquetral ligament

    5. Ulnolunate ligament CORRECT ANSWER: 3

    The short volar radiolunate ligament attaches to the volar lunate facet, which has fractured in this patient resulting in carpal subluxation.

     

    A subset of distal radius fractures involves the volar ulnar corner or critical corner. When the fracture is not appropriately identified and stabilized, carpal subluxation may occur. The small fragment, which supports the volar lunate, is at risk for inadequate fixation in addition to unfavorable biomechanical shear forces. Anatomically, the important short radiolunate ligament originates from the volar rim of the lunate facet. Fixation of this facet is integral to prevent volar subluxation of the carpus.

     

    Harness et al. performed a case series on 7 patients who lost fixation following open reduction and internal fixation of a distal radius fracture. They emphasize the importance of the volar lunate facet fragment in maintaining carpal stability. They add that the unique anatomy of this region may prevent standard fixation devices for distal radial fractures from supporting the entire volar surface effectively.

     

    O'Shaughnessy et al. analyzed 25 patients who underwent fragment-specific plating of the volar lunate fragment using a volar hook plate. Fragment-specific fixation using a volar hook plate designed specifically for these fragments was combined with other fragment-specific fixation techniques.

    There was no loss of fixation of the critical corner in this series.

     

    Figures A-D demonstrate an intra-articular distal radius fracture with a large radial styloid fragment and a volar ulnar lunate facet fragment (best seen on the lateral). Figure D, in particular, shows loss of reduction and volar carpal subluxation with the lunate facet fragment. Illustration A and B show postoperative radiographs of the same patient. The operative technique used in this patient included a dorsal bridge plate, a fragment specific radial styloid plate, and suture anchors repairing the volar radiolunate ligament. This is one of many techniques that can be used to address this fracture, but the importance is to recognize and address the pathology.

     

    Illustration C and D are labeled injury radiographs Incorrect Answers:

    Answers 1, 2, 4, and 5: These ligaments do not attach to the volar ulnar rim of the distal radius.

     

     

     

     

     

     

     

  103. A 55-year-old patient presents with numbness and tingling in the right small and ring fingers and associated hand weakness. On examination, there is decreased sensation on the volar, ulnar and radial aspect of the small finger and the volar, and ulnar aspect of the ring finger, but the sensation on the dorsum of the hand is preserved. Symptoms are reproduced with pressure over the wrist during flexion. There is weakness of finger adduction and grip strength, but flexor digitorum profundus strength of the ring and small finger remains preserved. The patient denies any previous trauma to the right hand or wrist. What study would most likely identify the causative lesion?

    1. MRI of the cervical spine

    2. EMG/NCV of the upper extremity

    3. MRI of the wrist

    4. Carpal tunnel view radiographs

    5. CT and MRI of the elbow

    CORRECT ANSWER:

    3

    The patient is presenting with ulnar neuropathy consistent due to ulnar tunnel syndrome. The most common cause of compression is a ganglion cyst in Guyon's canal which would be visualized on an MRI of the wrist.

     

    Ulnar tunnel syndrome is due to compression of the ulnar nerve at Guyon's canal and can lead to numbness and tingling in the ring and small finger without the involvement of the dorsoulnar hand. Three zones of compression have been described with Zone 1 leading to both motor and sensory deficits, Zone 2 leading to only motor deficits, and Zone 3 leading to only sensory deficits. The presence of a ganglion cyst in Guyon's canal is the most common cause of ulnar tunnel syndrome and accounts for 80% of cases. Magnetic resonance imaging will demonstrate a T2 hyperintense lesion in Guyon's canal in these cases. Ulnar tunnel syndrome results in motor weakness of the intrinsic muscles of the hand without affecting the ulnar nerve innervated flexor digitorum profundus muscle bellies of the ring and small finger.

     

    Subin et al. reported on a case of ulnar tunnel syndrome caused by a ganglion cyst that was preoperatively diagnosed by MRI. The authors reported that ganglion cysts can be a common cause of ulnar compression in Guyon's canal. They concluded that preoperative MRI can be useful for determining the location of ulnar nerve compression.

    Bachoura et al. reviewed the diagnosis and treatment of ulnar tunnel syndrome. They reported that the presence of a ganglion cyst in Guyon's canal is the most common reason for ulnar tunnel syndrome. The authors recommend surgical decompression with removal of any space-occupying lesions as nonoperative treatment has shown to have little efficacy.

     

    Illustration A depicts the zones of compression in Guyon's canal. Illustrations B and C demonstrate coronal and axial T2 MRI sequences depicting a ganglion cyst in Guyon's canal.

     

    Incorrect Answers:

    Answer 1: MRI of the cervical spine would potentially identify neuroforaminal stenosis at C8-T1 would present with radiculopathy down the patient's arm with the involvement of dermatomes of the ulnar nerve.

    Answer 2: EMG and NCV would likely present abnormal findings of ulnar nerve neuropathy, but would not identify the causative lesion.

    Answer 4: Carpal tunnel view radiographs would identify hamate fractures can sometimes lead to ulnar tunnel symptoms, but this is not the most common cause of ulnar tunnel syndrome.

    Answer 5: Corticosteroid injection at the carpal tunnel is a diagnostic, therapeutic, and prognostic tool for carpal tunnel syndrome. This patient is presenting with findings of ulnar tunnel syndrome.

     

     

     

     

     

     

     

     

     

     

  104. A 39-year-old construction worker presents to your clinic with a complaint of decreased ability to use his right hand at work. He sustained a crush injury to his hand 7 months ago and reports persistent swelling in the hand for 1-2 months after the injury. A new radiograph is shown in figure A. On physical exam, he can passively flex the proximal interphalangeal (PIP) joint when the metacarpophalangeal (MCP) joint is flexed but not when the MCP joint is extended. Based on this information, where is his stiffness most likely originating from?

     

     

     

    1. PIP joint capsule

    2. MCP joint capsule

    3. Extrinsic musculature

    4. Intrinsic musculature

    5. Bone deformity CORRECT ANSWER: 4

    The physical exam maneuver described above is Bunnell's test and this patient demonstrates intrinsic tightness.

     

    The examination of the stiff joint requires a fundamental understanding of the interdependence and relative roles of the joint capsule, extrinsic tendon adherence, and intrinsic function. If passive motion exceeds active motion, the musculotendinous unit is incompetent or adherent, or both. Posttraumatic interossei contracture is usually caused by edema, prolonged immobilization, or muscle ischemia. Hematoma and edema fluid envelop the interossei muscles after severe hand injuries. This fluid fills the interosseous muscle compartment and is trapped by the firm dorsal and volar interosseous fascia. When the edema finally subsides, fibrosis supervenes. Contracture of the intrinsic muscles of the hand can occur secondary to trauma, inflammation, and neurologic disorders.

    Lee and Gellman review their preferred methods of operative treatment of specific intrinsic deformities caused by intrinsic tightness. They note that intrinsic muscle contractures are a frequently overlooked cause of hand dysfunction.

     

    Espiritu et al. performed a cadaveric study to quantitatively assess the amount of release needed to overcome intrinsic tightness. Index, middle, ring, and little fingers regained flexion following the release of 59%, 65%, 26%, and 33% of each extensor hood, respectively.

     

    Figure A is a normal hand X-ray.

    Illustration A demonstrates how to perform Bunnell's test.

     

    Incorrect answers:

    Answers 1-2: MCP joint position does not influence PIP joint motion when a PIP joint contracture is present. PIP joint motion is not influenced by an MCP joint contracture.

    Answer 3: Extrinsic tightness results in more PIP joint flexion with MCP joint extension than with MCP joint flexion. This is caused by extensor tendon adhesions.

    Answer 5: This patient has no bony deformity.

     

     

     

     

     

  105. A 23-year-old professional rower presents with pain over the radial wrist when pronating and supinating his forearm. He denies any recent history of trauma and reports that the pain has been worsening over the past few months. On physical examination, there is crepitus over the distal forearm with wrist and thumb extension. Which of the following matches the correct extensor tendon compartment with the structures most likely affected in this pathology?

    1. Compartment 2; Abductor pollicis longus (APL) and extensor pollicis brevis (EPB) muscle bellies

    2. Compartment 2; Extensor carpi radialis brevis (ECRB) and extensor carpi radialis longus (ECRL) tendons

    3. Compartment 2; APL and EPB tendons

    4. Compartment 1; ECRB and ECRL muscle bellies

    5. Compartment 1; Extensor pollicis longus (EPL) tendon

    CORRECT ANSWER: 2

    This patient is presenting with intersection syndrome (IS), which affects the ECRB and ECRL.

     

    IS in an inflammatory tendon disorder occurring at the area where the muscle bellies of the APL and EPB cross the underlying ECRB and ECRL tendons. It is an overuse disorder that is seen with sporting activities such as racket sports, rowing, weight lifting, canoeing, and skiing. Patients with IS have pain and tenderness on the radial side of the distal forearm that is associated with crepitus when flexing and extending the wrist. IS must be differentiated from de Quervain's tenosynovitis (DQT), which is a disorder of the first dorsal compartment (APL and EPB tendons). DQT is more common in women and often presents in patients over 40. Pregnant or lactating women also commonly present with DQT.

     

    Skinner performed a review of intersection syndrome, also known as crossover tendinitis. The author reported that symptoms may worsen after pregnancy.

    He stated that IS is differentiated from DQT as it affects the first dorsal compartment. He concludes that having squeaking sensation may also differentiate the two diagnoses.

     

    Sato et al. performed a study to determine the demographic characteristics of patients who were diagnosed with IS and compared that with DQT. They found that, compared to DQT, IS occurred more frequently in men and in the dominant hand. IS also occurred in a younger age group and presented with a

    much shorter duration of symptoms. They conclude that, regardless of age, a peripartum woman likely has DQT.

     

    Illustration A is a photograph detailing the structures involved in IS and DQT. Illustrations B and C are photographs detailing the anatomic locations of pain in IS and DQT, respectively.

     

    Incorrect Answers:

    Answers 1 & 3: The APL and EPB are affected in DQT and are located in the first dorsal wrist compartment.

    Answer 4: The ECRB and ECRL muscle bellies are located more proximally. The tendons are affected in IS and these structures are located in the second dorsal compartment.

    Answer 5: The EPL is located in the third dorsal compartment and is not affected in IS.

     

     

     

     

     

     

     

     

     

     

  106. A 52-year-old construction worker presents with pain in the right proximal dorsoradial forearm over the last 8 months. He reports that his pain is worsened when using a screwdriver and lifting heavy objects. On exam, there is tenderness over the radial head and mobile wad, and pain with resisted supination and resisted third finger extension. There is no tenderness over the lateral epicondyle. There is no appreciable motor weakness or sensory deficits. Electromyography reveals no abnormalities. His radiographs are shown in Figures A and

    B. The patient has not sought any treatment up until this point. What is the likely diagnosis and first line treatment?

     

     

     

     

     

     

    1. Radial tunnel syndrome; physical therapy and activity modification

    2. Lateral epicondylitis; debridement of the extensor carpi radialis brevis

    3. PIN Syndrome; surgical release of the leash of Henry and arcade of Froshe

    4. Radial tunnel syndrome; steroid injection into the radial tunnel

    5. Lateral epicondylitis; steroid injection into the lateral epicondyle

    CORRECT ANSWER: 1

    The patient is presenting with radial tunnel syndrome, which is initially treated with exhaustive physical therapy and activity modification.

     

    Radial tunnel syndrome results from pathologic compression of the radial nerve in the proximal dorsoradial forearm. This presents without any of the motor or sensory changes that are characteristic of PIN compression syndrome and Wartenberg's syndrome. Despite the similar presentation of both PIN syndrome and radial tunnel syndrome the treatment for both remains the same. Initial treatment involves physical therapy and activity modification, and night bracing for a minimum of 1 year. Patients with persistent pain may be amenable to surgical release of the Arcade of Froshe, the distal edge of the supinator, and fibrous bands superficial to the radiocapitellar joint. A posterior approach is preferred as this allows safe access to the supinator and the distal edge. An anterior approach is usually performed when there a mass demonstrated on MRI that would be inadequately decompressed posteriorly.

     

    van den Ende et al. reported on a case of radial tunnel syndrome. The authors proposed using the rule-of-9 test, which includes self-reporting of symptom location, with localization in the two proximal lateral boxes being suggestive of the diagnosis(see illustration A). The authors also suggested using a motor latency of 0.3 ms as a diagnostic cut-off for radial tunnel syndrome, but mentioned further studies are needed to verify this.

     

    Sotereanos et al. performed a retrospective study on surgical release outcomes for radial tunnel syndrome in mostly workman's compensation cases. They found that surgical release was associated with 54% fair-poor outcomes and only 21% of patients returned to their previous level of employment. The authors concluded that surgical release of the radial tunnel in workman's compensation patients to be associated with poor outcomes.

     

    Figures A and B are AP and lateral radiographs of the right elbow without any apparent pathology. Illustration A depicts a clinical photograph of a left elbow with the rule-of-9 boxes present. Diagnosis can be made if there is the localization of symptoms to boxes 1 and 2, which are highlighted red in the figure.

     

    Incorrect Answers:

    Answer 2: Pain centered over the lateral epicondyle and is worsened with wrist extension is characteristic of lateral epicondylitis. Debridement of the extensor carpi radialis brevis is the surgical treatment for refractory lateral epicondylitis. Answer 3: PIN syndrome typically presents with dorsal wrist pain with associated thumb, wrist, and finger extension weakness and pain reproducible with resisted supination. The leash of Henry and the arcade of Froshe are

    potential sites of compression in radial tunnel syndrome and PIN syndrome; however, surgical treatment is not indicated until exhaustive conservative treatment has failed.

    Answer 4: Steroid injection into the radial tunnel is a second line treatment option for radial tunnel syndrome, as well as a diagnostic test. Diagnosis can be confirmed if there is relief of symptoms in PIN palsy and radial tunnel syndrome after the injection.

    Answer 5: Steroid injection into the lateral epicondyle is a treatment option for lateral epicondylitis. However, at 1-year there have been no outcome differences between physical therapy alone and corticosteroid injections for the treatment of lateral epicondylitis.

     

     

     

     

     

     

  107. For which of the metacarpal fractures shown in Figures A-E is buddy taping and an optional follow-up appropriate?

     

     

     

     

     

     

     

     

     

     

    1. Figure A

    2. Figure B

    3. FIgure C

    4. Figure D

    5. FIgure E CORRECT ANSWER: 2

    Figure B is a radiograph of an angulated fifth metacarpal neck fracture, which can be treated with buddy taping and an optional follow-up.

     

    Fifth metacarpal fractures that simply are angulated without rotational deformity generally have good outcomes. Acceptable functional outcomes have been reported even for angulation up to 70°. Most fractures can be treated without surgery. Additionally, evidence exists showing that optional follow-up is sufficient; patients reported better satisfaction when managed with buddy taping without scheduled follow-up. Reduction, casting, and scheduled return visits for examination and additional radiographs may represent unnecessary burdens to the patient and the health care system.

     

    van Aaken et al. performed a prospective randomized study comparing soft wrap/buddy taping to reduction and casting of fifth metacarpal neck fractures. They found that there was no significant difference between both groups' secondary outcomes of pain, satisfaction with the esthetic appearance, mobility of the metacarpophalangeal-joint at flexion and extension, or power grip. They concluded that the use of soft wrap and buddy taping for treatment of a boxer's fracture with palmar angulation ≤70° and no rotational deformity was supported by the results of their study.

     

    Finger et al. studied the results of offering patients optional follow-up for simple upper extremity fractures (adequately aligned metacarpal fractures, non-or minimally displaced distal radius fractures, and isolated non- or minimally displaced radial head fracture). They found that a majority of patients preferred optional follow-up for simple upper extremity fractures with a good prognosis. They concluded that hand surgeons can consider offering patients with low-risk hand fractures an optional second visit.

     

    Bansal et al. performed a study evaluating whether follow-up after fifth metacarpal neck fractures is unnecessary after initial assessment in a hand clinic. They found that patients reported better satisfaction when managed with buddy taping and without regular follow-up visits.

     

    Figures A-E are radiographs of the left hand demonstrating an intraarticular second metacarpal base fracture, an angulated fifth metacarpal neck fracture,

    a transverse second metacarpal shaft fracture with shortening, a spiral third metacarpal shaft fracture with shortening, and spiral fractures of the second through fourth metacarpal shafts, respectively.

     

    Incorrect Answers:

    Answers 1, 3, 4, and 5: Buddy taping and an optional follow-up is not appropriate for these fractures

     

     

     

  108. In which of the following scenarios would a periarterial sympathectomy +/- arterial reconstruction be indicated?

    1. A patient with Raynaud's who has failed medical management with an ischemic digital ulcer

    2. A patient with thromboangiitis obliterans who continues to smoke with an ischemic digital ulcer

    3. A patient with complex regional pain syndrome with a crush injury who has exhausted medical treatment

    4. A patient with a diabetic Charcot neuropathy with a non-healing plantar ulcer despite total-contact casting

    5. A patient with thoracic outlet syndrome who would like to return to competitive weightlifting

    CORRECT ANSWER: 1

     

    A patient with Raynaud's who has failed medical management with an ischemic digital ulcer may benefit from a periarterial sympathectomy +/-periarterial reconstruction.

     

    Raynaud's phenomenon describes the inappropriate vasospasm resulting from inadequate vascular structures and/or homeostatic mechanisms. Nonoperative modalities are often appropriate for patients without occlusive disease or abnormal vasculature. These include avoiding the cold, smoking cessation, activity modification, protective garments, biofeedback, and medications (calcium channel blockers, tricyclic antidepressants, serotonin reuptake inhibitors, and alpha antagonists). There is some evidence to support the use of botulinum toxin A injection in the treatment of Raynaud's refractory to other non-surgical modalities, as it can improve digital perfusion. When these modalities have failed, especially in the setting of a concomitant collagen vascular or occlusive disease, periarterial sympathectomy +/- arterial reconstruction is often beneficial.

     

    Thibaudeau et al. provided an algorithmic approach to the surgical treatment

    of chronic ischemia of the hand. They performed a systematic literature review. They concluded that sympathectomy, arterial bypass, and venous arterialization are efficacious but not without moderately high complication rates.

     

    Shammas et al. evaluated the outcomes of sympathectomy +/- vascular bypass for digital ischemia in connective tissue disorders. They reviewed all patients (36 hands) at their institution undergoing this procedure for connective tissue disorders over an 18 year period. They recommended that periarterial sympathectomy be performed in conjunction with vascular bypass, as this resulted in a significantly higher rate of durable ulcer resolution.

     

    Illustration A is an ischemic digital ulcer in a patient with Raynaud's. Illustration B is an intra-operative photo demonstrating a digital periarterial sympathectomy.

     

    Incorrect Answers:

    Answer 2: Periarterial sympathectomy in the setting of thromboangiitis obliterans (Buerger's Disease) is controversial, but would not be indicated in a patient who continues to smoke.

    Answer 3: A stellate ganglion block may play a role in the treatment of complex regional pain syndrome (CRPS); there is no role for periarterial sympathectomy.

    Answer 4: Periarterial sympathectomy plays no role in the treatment of diabetic neuropathy.

    Answer 5: Periarterial sympathectomy plays no role in the treatment of thoracic outlet syndrome.

     

     

     

     

     

     

     

     

  109. A 39-year-old patient presents with a left thumb mass at the interphalangeal joint of eight months duration. The patient denies any pain but is bothered by the appearance and limited range of motion. The patient opts to undergo surgical excision. Figure A depicts a clinical photograph of the patient's hand. Figures B, C, and D depict axial T1, T2, and T1 with contrast MRI sequences of the thumb, respectively. Figure E depicts the histology of the lesion at the time of marginal excision. What is the next appropriate step?

     

     

     

     

     

     

     

     

     

     

     

     

     

    1. Ray resection

    2. Initiate chemotherapy

    3. Routine follow-up

    4. Return to OR for wide tumor bed resection

    5. Radiation of the tumor bed

    CORRECT ANSWER: 3

    The patient is presenting with a giant cell tumor the tendon sheath (GCTTS), which is treated with a marginal excision. In this case, no further treatment is necessary.

     

    Giant cell tumor of the tendon sheath is the second most common tumor of the hand next to synovial cysts. The lesion is histologically characterized by many giant cells, lipid-laden macrophages, and sheets of polygonal cells.

    Patients often present with an enlarging mass and pain in the affected digit. There are high recurrence rates with these lesions, specifically if there is involvement of the tendon, intra-articular involvement, or bone involvement. Marginal excision remains the mainstay of treatment.

     

    Lancigu et al. performed a retrospective study on factors associated with recurrence following treatment for GCTTS. They found that intra-articular involvement and tendon destruction were associated with recurrence of GCTTS. Further, there was no association between histological findings and recurrence rates.

     

    Lanzinger et al. presented a case and reviewed the literature regarding GCTTS. The preferred treatment remains marginal excision of the lesion, with several studies reporting lower recurrence rates after radiation therapy.

    However, the authors concluded further research into this is required to make any recommendations.

     

    Figure A depicts a clinical photograph of a left thumb with a mass on the volar surface at the level of the interphalangeal joint. Figure B depicts an axial T1 MRI of thumb with a hypointense mass of the radial side consistent with a giant cell tumor of the tendon sheath. Figure C depicts an axial T2 MIR of the thumb with a hyperintense lesion on the radial aspect consistent with a giant cell tumor of the tendon sheath. Figure D is an axial T1 MRI of the thumb with contrast enhancement of a lesion on the radial aspect consistent with giant cell tumor of the tendon sheath. Figure E depicts a histologic slide of giant cell tumor of the tendon sheath with multinucleated giant cells and hemosiderin-laden histiocytes.

     

    Incorrect Answers:

    Answer 1: Ray resection is not necessary for GCTTS. Marginal resection is sufficient treatment for GCTTS.

    Answer 2: Chemotherapy is not necessary for GCTTS.

    Answer 4: Wider surgical resection is not necessary for GCTTS.

    Answer 5: Radiation treatment has been reported to have lower recurrence rates, but there are potential complications. Currently, radiation therapy is not recommended as part of the standard treatment protocol.

     

     

     

  110. A patient presents with a hand injury after an accidental self-inflicted gunshot wound. The ring finger is traumatically amputated at the proximal phalanx. Which treatment option will provide optimal cosmesis and grip strength?

    1. Early revision amputation

    2. Early replantation

    3. Delayed fourth ray resection

    4. Early fourth ray resection with fifth ray transposition

    5. Delayed fourth ray resection with fifth ray transposition

    CORRECT ANSWER: 4

    Mutilating ring finger injuries have better cosmetic and grip strength outcomes with early fourth ray resection with a fifth ray transposition.

     

    Traumatic amputation of the ring finger at the level of the PIP joint and proximal is a relative contraindication to replantation due to post-operative stiffness resulting in a nonfunctional digit. Revision amputation is an option but frequently leads to decreased grip strength and a stump that is often repeatedly traumatized. Complete resection of the fourth ray with transposition of the fifth ray allows for a more cosmetic appearance of the hand by closing the gap between the remaining digits while also increasing grip strength.

     

    Monreal performed a retrospective review of patients with mutilating ring finger injuries undergoing fourth ray resection and radial transposition of the fifth ray. The author reported that grip strength and Result Assessment Scores (RAS) improved after resection and transposition. He concluded that performing a fourth ray resection with fifth ray transposition and hamate capitate arthrodesis to be technically easier than metacarpal osteotomy while improving cosmesis and hand function.

     

    Bhat et al. performed a retrospective review of functional outcomes and

    cosmesis following single-digit amputations. For patients that had significant gapping after central digit amputations, adjacent digit transposition provided improved cosmesis with significantly higher Disabilities of the Arm, Shoulder, and Hand (DASH) scores. They concluded that despite the management of finger amputations, most patients are able to return to the same level of work despite the loss of baseline grip strength.

     

    Sadek et al. prospectively reviewed patients that sustained a mutilating fourth ray injury that was treated either with early or delayed fourth ray amputation and fifth ray transposition. They found that patients treated with early fourth ray resection with fifth ray transposition had significantly higher grip strength, pronation strength, and cosmesis compared to delayed treatment. The authors supported the early treatment of mutilating fourth ray injures with fourth ray resection and fifth ray transposition.

     

    Illustration A depicts an AP radiograph of the hand with resection of the 4th ray and transposition of the 5th ray with screw fixation through the capitohamate joint. Illustration B is the clinical radiograph of the right hand with a 3rd ray resection and transposition of the 4th ray.

     

    Incorrect answers:

    Answer 1: Revision amputation will result in a less cosmetic appearance of the hand with decreased grip strength compared to ray resection and transposition.

    Answer 2: Early replantation is likely to result in a stiff and nonfunctional digit due to the injury through Zone, also known as"No man's land." Replantation and repair in this region is fraught with stiff and nonfunctional flexor tendons due to the scarring of FDS and FDP.

    Answer 3: Delayed fourth ray resection has been associated with decreased grip strength, pronation strength, and cosmesis. Therefore, early ray resection with adjacent ray transposition will yield a more favorable outcome.

    Answer 5: Sadek et al. found that early fourth ray resection resulted in higher cosmetic appearance and greater grip strength compared to delayed treatment.

     

     

     

     

     

     

     

     

  111. A space-filling lesion leading to nerve compression in the area marked "X" in Figure A would manifest by which of the physical examination findings?

     

     

     

    1. Inability to flex thumb

    2. Inability to extend thumb

    3. Inability to cross index and middle fingers

    4. Inability to extend the proximal interphalangeal (PIP) joints of the index, long, ring, and small fingers

    5. Inability to sense light touch at dorsal aspect of ring and small fingers

    CORRECT ANSWER: 2

    A space-filling lesion causing compression of the PIN at the arcade of Frohse (the thick fibrous leading edge of the supinator) would manifest as an inability to extend the thumb, which is performed by the extensor pollicus longus (EPL) muscle.

     

    While the arcade of Frohse is the most common site of PIN compression, other sites include the recurrent radial vessels (vascular leash of Henry), the distal aspect of the supinator, and the extensor carpi radialis bevis tendon.

     

    The PIN is a branch of the radial nerve that originates at the radiohumeral joint line. It typically dives under the supinator muscle at the arcade of Frohse, courses around the radial neck to the posterior forearm compartment and through the posterior interroseous membrane of the forearm, and ends at the dorsal wrist capsule where it provides capsular sensation. However, several variations in the site of variation do exist. The PIN provides motor innervation to the following muscles: extensor digitorum communis, extensor digiti minimi, extensor carpi ulnaris, supinator, EPL, abductor pollicus longus, extensor pollicus brevis, extensor indicis proprius, and extensor carpi radialis brevis (most commonly innervated by radial nerve, but sometimes PIN depending on variation in bifurcation). A PIN palsy may manifest as wrist drop and inability to extend the thumb.

     

    Meng et al. performed an anatomical study to examine the utility of

    ultrasound-guided PIN injections at the arcade of Frohse. They reported a 95% success rate with injection of the PIN. They concluded that with ultrasound guidance, safe PIN injection within the arcade of Frohse is possible.

     

    Urch et al. performed a cadaveric analysis of 3 surgical approaches used for decompression of the PIN within the radial tunnel. They reported that the anterior and anterolateral approaches were best for visualizing the Arcade of Frohse, fibrous bands of the radial head, leash of Henry, and the origin of the ECRB, whereas the posterior approach was best for visualizing the distal border of the supinator. They recommend decompression through multiple approaches, as no single approach was adequate for full visualizing of all PIN compression sites in the radial tunnel.

     

    Figure A demonstrates the bifurcation of the radial nerve to form the PIN and the superficial radial nerve. The "X' in Figure A depicts the arcade of Frohse, a common site for PIN compression.

     

    Incorrect Answers:

    Answer 1: Inability to flex the thumb would be caused by an anterior interosseous nerve (AIN) palsy.

    Answer 3: Inability to cross the index and middle fingers would be caused by an ulnar nerve palsy.

    Answer 4: The ability to extend the proximal interphalangeal (PIP) joints of the index, long, ring, and small fingers would be preserved by the hand interossei and lumbrical muscles, which are innervated by the deep branch of the ulnar nerve.

    Answer 5: Inability to sense light touch at the dorsal aspect of the ring and small fingers would be caused by an ulnar nerve palsy.

     

     

     

  112. A 36-year-old male injured his right arm after a fall. His injury radiograph is shown in Figure A. On physical examination, he was found to have a nerve injury. Which function is the last to return?

     

     

     

    1. Pain sensation to the radial portion of the dorsum of the hand

    2. Temperature sensation to the radial portion of the dorsum of the hand

    3. Sensation of light touch to the radial portion of the dorsum of the hand

    4. Wrist extension

    5. Elbow extension CORRECT ANSWER: 4

    This patient sustained a humeral shaft fracture which is associated with radial nerve palsy. Motor function is also last function to return. In the case of a radial nerve injury, this is manifested in a loss of wrist extension.

     

    Peripheral nerve injury causes a loss of distal function in the following order: motor function, proprioception, light touch sensation, temperature, pain, and sympathetic activity. Nerve function recovery occurs in the reverse order.

    Therefore, if a patient has a nerve injury, motor function is lost first and is the last to recover. The injury this patient sustained is also known as a Holstein-Lewis fracture.

     

    Yan et al. performed a study to determine the role of precisely matching fascicles in the quick recovery of nerve function in long peripheral nerve defects. Recently tissue-engineered nerve grafts have shown potential in

    peripheral nerve injuries. They tested them in an animal nerve injury model to study the role of the precise matching of fascicles in the effectiveness of nerve function recovery. They found that in 6 weeks after surgery fascicle matched nerve grafting had good functional recovery and showed excellent physiological outcomes.

     

    Arbat-Plana et al. performed a study to determine the effects of forced, passive, and voluntary exercise on spinal motoneurons after peripheral nerve injury. The aim of their study was to investigate whether passive exercise is able to mimic the effects induced by forced exercise on the changes that axotomized motoneurons suffer after peripheral nerve injury. They found partial preservation of synapses only in axotomized motoneurons from animals subjected to high-intensity cycling and the ones that freely ran long distances, but not when low-intensity exercise protocols were applied. They conclude that intensity, not the time of exercise is the key element to prevent loss of perineuronal nets in motoneurons after axotomy.

     

    Figures A is an AP of the humerus demonstrating a distal-third humeral shaft fracture.

     

    Incorrect Answers:

    Answers 1, 2, & 3: Motor function is the first to be lost with a peripheral nerve injury, and the last to return.

    Answer 4: With a radial nerve injury at this level, elbow extension would be maintained.

     

     

     

  113. A 42-year-old intoxicated man presents with hand pain and redness after being involved in a fight. On physical exam, there is a puncture wound with minimal drainage, which is shown in Figure A. He undergoes incisional irrigation and debridement, and is started on intravenous antibiotics. Had a culture been performed, what would have been the most common organism(s) isolated?

     

     

     

    1. Corynebacterium diphtheria

    2. Eikenella corrodens

    3. Moraxella catarrhalis

    4. Pasteurella multocida

    5. Polymicrobial CORRECT ANSWER: 5

    This patient has sustained a human fight bite and is presenting with signs of infection. If cultured, the most common isolate is often polymicrobial.

     

    Severe hand infections after human bites are not uncommon. Over 600 bacterial species have been identified in human saliva. Bacteria typically isolated from infected hand wounds from human bites are frequently polymicrobial, with 4 isolates per culture, typically. Other common bacteria include Streptococcus viridans, Staphylococcus aureus, Eikenella corrodens, and anaerobes.

     

    Shoji et al. reviewed the literature regarding the treatment outcomes of fight bite injuries. They reported that the human mouth contains as many as 50 species of bacteria, with the most common isolates being polymicrobial, Staphylococcus aureus, Streptococcus species, Corynebacterium species, and Eikenella corrodens. As such, they recommended broad-spectrum antibiotics

    and combination therapies, such as cephalosporins or a combination of gentamicin and a penicillinase-resistant penicillin for coverage.

     

    Shewring et al. performed a prospective study outlining the management of clenched fist fight bite injuries. They reported that 100% of patients with such injuries had surgical exploration with further débridements as necessary in a 4-year period. While all patients with metacarpophalangeal (MCP) joint injury had satisfactory or good outcomes, 42% of patients with proximal interphalangeal (PIP) joint injuries had poor results. They recommended a tendon-splitting approach to the MCP joint for excellent exposure during irrigation and debridement of MCP injuries. For fight bites to the PIP joint, a more intense approach with serial debridements and a protracted period of antibiotics should be considered, as these have a high likelihood of a poor outcome.

     

    Figure A depicts a human fight bite with a puncture wound overlying the 3rd MCP joint.

     

    Incorrect Answers:

    Answers 1, 3 & 4: Corynebacterium diphtheria, Moraxella catarrhalis, and Pasteurella multocida are more commonly seen in dog and cat bites.

    Answer 2: Although Eikenella corrodens is isolated in about a third of cases in several studies, polymicrobial isolates are more typical.

     

     

     

  114. Which of the following findings would be present in a patient with a sagittal band injury?

    1. Locking of the digit in a flexed position

    2. Laxity of the PIP joint to valgus stress when the PIP is flexed 30° and the MCP is flexed 90°

    3. Rigidity of the DIP joint when the PIP is flexed 90° over edge of a table and the patient is asked to extend the digit

    4. Inability to actively flex the DIP joint

    5. Inability to actively extend the MCP joint but able to maintain MCP extension after passive extension

    CORRECT ANSWER: 5

     

    Patients with a sagittal band injury typically present with an inability to actively extend the MCP joint but the ability to maintain MCP extension after passive extension.

    Sagittal band injuries represent zone 5 extensor tendon injuries. The sagittal band attaches dorsally to the extensor hood, runs palmar to insert on the volar plate, and serves as the primary lateral stabilizer of the extensor tendon over the metacarpophalangeal (MCP) joint. Sagittal band injuries, therefore, present with tendon subluxation or dislocation, and the inability to actively extend the MCP joint but the ability to maintain MCP extension after passive extension.

     

    Grandizio et al. published a review on sagittal band, boutonniere, and pulley injuries in the athlete. They report that sagittal band injuries most frequently occur in the long finger due to its greater length and the relative weakness of its fibrous attachments to the extensor hood in comparison to the other digits. Additionally, a radial-sided rupture of the sagittal band typically occurs and results in pain, swelling, and ulnar subluxation of the extensor tendon with MCP flexion.

     

    Kleinhenz et al. published a review on closed sagittal band injury of the metacarpophalangeal joint. They reported that sagittal band injury is a rare injury that often results in subluxation or dislocation of the extensor digitorum communis tendon. They note that many acute injuries can be managed nonsurgically with extension splints.

     

    Incorrect Answers:

    Answer 1: This describes a trigger finger, which results from the entrapment of the flexor tendons at the level of the A1 pulley.

    Answer 2: This describes a collateral ligament injury.

    Answer 3: This describes a positive Elson's test, which is seen in patients with a central slip injury.

    Answer 4: This describes jersey finger, an avulsion injury of FDP from the insertion at the base of distal phalanx

     

     

     

  115. A 63-year-old woman sustains the closed injury shown in Figure

    A. In the emergency room, she has full active range of motion of all digits. She undergoes surgery and her postoperative images are shown in Figure B. She follows up 12 months later with a concern of a recent popping sensation near the wrist. She is now unable to flex her thumb interphalangeal joint but has full active and passive range of motion of remaining finger joints. Radiographs reveal fully healed fracture. What is the most appropriate next step in treatment?

     

     

     

     

     

     

    1. Change plate position and perform flexor pollicis longus repair

    2. Change plate position and perform flexor pollicis brevis repair

    3. Flexor pollicis longus tenolysis

    4. Remove plate and perform flexor pollicis longus reconstruction

    5. Remove plate and perform flexor pollicis brevis reconstruction

    CORRECT ANSWER: 4

    This patient sustained a flexor pollicis longus (FPL) attritional rupture after open reduction internal fixation (ORIF) of a distal radial fracture (DRF). Tendon reconstruction is a viable treatment option for FPL attritional rupture.

     

    The most common cause of tendon rupture after ORIF of a DRF is chronic tendon attrition. It is often due to placement of the plate distal to the watershed line or prominent volar distal radius plate placement, which results in chronic tendinopathy and eventual rupture of the FPL. Surgical options for attritional FPL tendon ruptures include FPL reconstruction with free tendon graft and 4th digit flexor digitorum superficialis transfer to the thumb.

     

    Monaco et al. performed a study to investigate the rate of tendon ruptures in a 12-month period after volar plating of DRF. They reported that the FPL was the most commonly reported tendon injury and that palmaris longus grafting and tendon transfer were the most commonly reported treatments following FPL tendon rupture. They recommended a critical assessment of hardware position at the time of the index procedure to avoid tendon complications.

     

    Azzi et al. published a systematic review on tendon rupture and tenosynovitis following ORIF of DRF. They reported a 7% rate of tendon-related adverse events after dorsal and volar plating of DRF, with tendon complications including extensor pollicis longus (EPL) tenosynovitis, EPL rupture, FPL tenosynovitis, FPL rupture, flexor digitorum profundus (FDP)/FDS tenosynovitis, FDP/FDS rupture, extensor digitorum communis (EDC) tenosynovitis, and EDC rupture.

     

    Figure A depicts a displaced comminuted distal radial fracture and ulnar styloid fracture. Figure B depicts volar plate fixation after ORIF of the DRF.

     

    Incorrect Answers:

    Answers 1, 2, 3, & 5: The most common cause of tendon rupture after ORIF of a DRF is chronic tendon attrition of the FPL, not flexor pollicis brevis (FPB). The attritional nature of this injury makes FPL repair and tenolysis difficult. Plate removal is reasonable given the fracture is well healed.

     

     

     

  116. What is the mechanism of action of the first-line prophylactic antibiotic used when utilizing leech therapy?

    1. Inhibits the 30S subunit

    2. Inhibits cell wall synthesis

    3. Inhibits the 50S subunit

    4. Inhibits DNA gyrase

    5. Inhibits RNA synthesis CORRECT ANSWER: 4

    The first-line chemoprophylactic treatment in patients undergoing leech therapy is ciprofloxacin, a fluoroquinolone. The mechanism of action of fluoroquinolones is through inhibition of DNA gyrase, which ultimately inhibits DNA synthesis.

     

    Medical-grade leeches release Hirudo medicinalis, a powerful anticoagulant used to treat venous congestion after replantation, grafts, or soft-tissue flaps. The offending microbe is Aeromonas hydrophilia, a gram-negative rod that is dormant in the intestines of the leeches and that also assists with clot digestion. However, Aeromonas hydrophilia can also cause infection in humans, ranging from local soft tissue infections to severe sepsis. As such, prophylactic antibacterial coverage is often recommended, with ciprofloxacin recognized as the first line chemoprophylaxis. A third-generation cephalosporins or sulfamethoxazole-trimethoprim (TMP-SMX) are recommended in cases where fluoroquinolones are contraindicated or in fluoroquinolone-resistant strains.

     

    Verriere et al. reviewed medicinal leech therapy (MLT) and Aeromonas spp. infection over a 2 year period. They reported 3 infections in 28 cases. They found that the Aeromonas spp. strains were susceptible to fluoroquinolones, TMP-SMX, aminoglycosides, and third-generation cephalosporins but resistant to amoxicillin-clavalunic acid and second-generation cephalosporins. They recommended routine tank water microbiological analyses in order to identify leeches containing resistant strains and to promote the selection of appropriate antibiotic prophylaxis.

     

    Kruer et al. performed a multicenter study to evaluate the efficacy of antimicrobial prophylaxis during medical leech therapy. Prophylaxis was given in all patients undergoing therapy. In cultures of all 4 surgical site infections after leech therapy, they isolated Aeromonas spp. resistant to the chosen prophylactic agent. They concluded that ciprofloxacin and TMP-SMX were equally effective at preventing leech-associated infections.

     

    Wilmer et al. performed a study to investigate the role of leech water sampling in the choice of prophylactic antibiotics in medical leech therapy. In 14 surveillance cultures with Aeromonas species, they reported 71.4% were susceptible to ciprofloxacin and 100% were susceptible to TMP-SMX. They recommended regular surveillance to detect resistant Aeromonas species in medical leeches.

    Incorrect Answers:

    Answer 1: While some studies have demonstrated the susceptibility of some Aeromonas strains to aminoglycosides, these are not the first line of treatment. Tetracyclines are also 30S subunit inhibitors.

    Answer 2: While some studies have demonstrated the susceptibility of some Aeromonas strains to third-generation cephalosporins, these are not the first line of treatment.

    Answer 3: 50S subunit inhibitors (including macrolides and Linezolid) have no role in Aeromonas infection prophylaxis.

    Answer 5: RNA synthesis inhibitors, like Rifampin, have no role in Aeromonas infection prophylaxis.

     

     

     

  117. A 30-year-old woman sustains an acute injury to her right long finger while building new cabinets in her kitchen. On exam, she has pain and swelling at the distal interphalangeal (DIP) joint and lacks active DIP extension. Figure A is her current radiograph. What is the next best step in treatment?

     

     

     

    1. Extension splinting of DIP joint 24 hours daily for 6-8 weeks

    2. Extension splinting of DIP joint 12 hours daily for 6-8 weeks

    3. Extension splinting of DIP joint 24 hours daily for 2-4 weeks

    4. Extension splinting of DIP joint 12 hours daily for 2-4 weeks

    5. Extension splinting of PIP joint 24 hours daily for 6-8 weeks

    CORRECT ANSWER: 1

    The treatment for an acute mallet finger without a fracture is full-time (24 hours) extension splinting of the DIP joint for 6-8 weeks.

     

    A non-bony mallet finger injury is caused by terminal extensor tendon disruption distal to the DIP joint, most commonly from forceful DIP joint flexion. The fingertip remains in a flexed position, with an extensor lag at the DIP. Acute tendinous injuries and non-displaced fractures may be treated with full-time DIP joint extension splinting for 6-8 weeks, with closed reduction and percutaneous pinning (CRPP) or open reduction internal fixation (ORIF) reserved for displaced fractures. After nonoperative treatment, patients may experience a mild DIP extensor lag but without functional deficit.

     

    Gruber et al. performed a prospective randomized controlled trial comparing night splinting with no splinting after full-time splinting treatment of mallet finger for 6-8 weeks. They reported no significant differences in final extensor lag and patient satisfaction between patients that did and did not receive a night splint. They concluded that supplemental night splinting after mallet finger injuries do not change the outcomes.

     

    Altan et al. performed a prospective study to compare the results of early (within 2 weeks of injury) and delayed (2-4 weeks of injury) extension orthosis fabrication in closed tendinous mallet injuries. They reported no significant differences between the groups in initial extension lag, arc of flexion, and functional outcomes after treatment. They concluded that conservative management of tendinous mallet finger injuries with treatment delays up to 4 weeks can be treated as well as those injuries in patients presenting within the first 2 weeks of injury with low long-term complication rates.

     

    Figure A depicts the normal bony anatomy of the third middle and distal phalanges. Illustration A depicts an extension splint used for non-operative management of mallet injuries.

     

    Incorrect Answers:

    Answers 2, 3, and 4: The treatment for tendinous and minimally displaced mallet finger injuries is full-time (24 hours daily) extension splinting of the DIP joint for 6-8 weeks.

    Answer 5: In mallet finger injuries, the DIP joint must be immobilized, not the PIP joint.

     

     

     

     

     

     

  118. A child's hand is seen in Figure A. In addition, her mother complains that the ipsilateral forearm lacks rotation. No one else in the family has this constellation of symptoms. Of the following computed tomography studies shown in Figures B through F, which is more frequently associated with this patient's condition?

     

     

     

     

     

     

     

     

     

     

     

     

     

     

     

     

     

    1. Figure B

    2. Figure C

    3. Figure D

    4. Figure E

    5. Figure F

    CORRECT ANSWER: 2

     

    This child has symbrachydactyly of the left hand and radioulnar synostosis. These findings are associated with Poland syndrome, which is characterized by the unilateral absence of the sternocostal head of the pectoralis major and ipsilateral chest wall hypoplasia (shown in Figure C).

     

    Poland syndrome is a congenital disorder which includes unilateral chest wall hypoplasia, absence of the sternocostal head of pectoralis major, hypoplasia of the hand and forearm, symbrachydactyly, and breast or nipple aplasia. It is three times more common in males, and three-quarters of cases involve the right side.

     

    Kramer et al. retrospectively compared syndactyly release in Poland syndrome vs idiopathic syndactyly. The authors found significant persistent differences in function between operated and normal hands in Poland syndrome, but not among patients with idiopathic syndactyly. They concluded that functional deficits in hands affected by Poland syndrome are attributable to more than the syndactyly alone, whereas hands with idiopathic syndactyly are likely to have little postoperative functional deficit.

     

    Van Heest reviewed congenital disorders of the hand and upper extremity. She characterized Poland syndrome, discussing ipsilateral hand anomalies such as brachydactyly, syndactyly, or ectrodactyly. Other associated anomalies include hypoplastic scapula or ribs, or shoulder girdle muscles (e.g. latissimus dorsi), scoliosis and renal anomalies.

     

    Figure A shows symbrachydactyly, with shortening of the central digits, which may be seen in Poland's syndrome. Patients may have an absence or shortening of the middle phalanx of the affected fingers.

     

    Incorrect Answers:

    Answer 1: Figure B shows pectus excavatum, which is associated with Marfan syndrome but not with Poland syndrome.

    Answer 3: Figure D shows a normal CT chest (bone window).

    Answer 4: Figure E shows pectus carinatum, which is not associated with Poland syndrome.

    Answer 5: Figure F shows a rib osteochondroma on the right side. This is not associated with Poland syndrome.

     

     

  119. Which of the following factors is inconsistent with a retinacular cyst of the flexor tendon sheath?

    1. Small size (2-8mm)

    2. Palpable under metacarpophalangeal flexion crease

    3. May be excised with a small portion of the flexor sheath to prevent recurrence

    4. Aspiration may be a successful treatment

    5. Mass moves with the tendon

    CORRECT ANSWER: 5

    Retinacular cysts (RC) arise from the flexor pulley system and therefore do not move with the tendon.

     

    Volar RCs are infrequently tender. However, patients often complain of discomfort from these pea-sized masses that arise from the A1 or A2 flexor pulley sheath in the MCPJ flexion crease. These masses are very small.

    Aspiration may be technically difficult and patients may not wish to undergo manipulation in the office. However, aspiration is recommended by some sources prior to surgery.

     

    Bittner et al. compared aspiration versus surgical management of volar RCs. The recurrence rate after two aspiration attempts was higher as compared to surgical excision (34% vs. 7%). All patients who recurred after a single aspiration attempt and then elected for surgery were cured with surgery.

    Because the aspiration can be performed in the clinic and patients may not require surgery, the authors of this particular paper advocate trying aspiration before surgical excision.

     

    Korman et al. prospectively evaluated the efficacy of immobilization following aspiration of carpal and digital cysts. Nearly half (49%) of ganglion cysts recurred regardless of treatment. Immobilization did not reduce recurrence after aspiration. The authors do not advocate for immobilization of the joint following aspiration of a volar RC.

     

    Thornburg et al. review ganglion cysts about the hand and wrist. A portion of the article focuses specifically on volar RCs which account for 7-12% of all hand and wrist ganglions. In addition, the masses range from 3-10mm, arise off of the A1-A2 pulley, and are diagnosed on physical exam. The masses can be successfully aspirated (70%) but the attempt places the digital nerve at risk. The masses are also safely and effectively removed surgically.

     

    Incorrect Answers:

    Answer 1: Volar RCs are often small and pea-shaped.

    Answer 2: The RCs are often palpable in the MCPJ flexion crease. These become symptomatic when patients grasp objects.

    Answer 3: It is believed that excising a portion of the pulley may reduce the likelihood of recurrence.

    Answer 4: Aspiration may be attempted. If unsuccessful or the mass reforms, open excision is the next best option.

     

     

     

  120. An active 66-year-old pastry chef falls in her kitchen and notes pain and deformity of her small finger. She is taken to the hand clinic for assessment. There are no open wounds. Pre-reduction films are obtained prior to reduction and shown in Figures A and B. After reduction, there is neither rotation, shortening, nor angulation. What is the next most appropriate step in management?

     

     

     

     

    1. Immediate range of motion without immobilization

    2. Intramedullary screw

    3. Reduction and percutaneous fixation

    4. Open reduction and internal fixation

    5. Short term immobilization and range of motion at 3 weeks

    CORRECT ANSWER: 5

    Proximal phalanx fractures which are well-reduced can be treated nonoperatively with 3 weeks of immobilization followed by range of motion exercises.

     

    Fracture displacement with an apex volar position, secondary to the pull of the central slip, is possible and routine follow-up is necessary for this fracture pattern. However, extra-articular fractures with < 10° angulation or < 2mm shortening and no rotational deformity can be effectively treated with 3 weeks of immobilization followed by aggressive motion. Operative intervention is indicated for open fractures and those that cannot maintain a reduction.

    Proximal phalanx fractures can be fixed with CRPP or ORIF. Very proximal phalanx fractures may be secured by passing a k-wire through the metacarpal head; Eaton-Belsky pinning.

     

    Gaston and Chadderdon reviewed the treatment of phalangeal fractures. The authors recommend nonsurgical management for stable, extra-articular fractures of the proximal phalanx and for surgical management displaced fractures, unicondylar fractures, or those in high level athletes.

     

    Kozin et al. reviewed operative treatment for phalangeal shaft fractures. The authors advocate for non-operative management for stable fracture configurations for those that are well reduced in a position of MCPJ flexion for up to 3 weeks. The fracture may be displaced in an apex volar angulation by the interossei insertion on the proximal phalanx base and as the central slip pulls the distal fragment into hyperextension. Transverse fractures may be pinned or secured with a small plate while oblique fractures may be secured with lag screws or pins.

     

    Figure A is a lateral view of the hand which demonstrates an apex volar angulation of the small finger proximal phalanx fracture.

    Figure B is an AP view which demonstrates the displaced and angulated proximal phalanx fracture.

     

    Incorrect Answers:

    Answer 1: The fracture would likely displace without any immobilization secondary to the pull of the central slip and interossei.

    Answer 2: An intramedullary screw can be used to fix phalangeal and metacarpal shaft fractures - but this fracture is too proximal to use this technique.

    Answer 3: The fracture is well-reduced and stable. A trial of non-operative management is the next best step in management. If the reduction fails, pinning would be one option to maintain the reduction.

    Answer 4: The fracture is well-reduced and stable. A trial of non-operative

    management is the next best step in management. If the reduction fails, a plate and screw construct would be one option to maintain the reduction.

     

     

     

  121. Figure A is the radiograph of an 8-year-old female who presents with complaints of right shoulder pain for the past 3 weeks. She has been unable to perform gymnastics during this time. She is treated conservatively for a proximal humeral stress fracture. In gymnasts, which upper extremity stress fracture is most likely to lead to growth arrest?

     

     

     

     

    1. Scaphoid

    2. Distal humerus

    3. Clavicle

    4. Shoulder

    5. Distal radius CORRECT ANSWER: 5

    Gymnasts are prone to upper extremity stress fractures. Among the choices listed, a stress fracture of the distal radius has the greatest potential for growth arrest.

     

    This patient has a proximal humeral stress fracture as evident by the wide proximal humeral physis. Other upper extremity stress fractures in which gymnasts are prone to include the scaphoid, distal radius, elbow, and clavicle. Among the choices listed, a stress fracture of the distal radius is most likely to

    result in growth arrest. Repetitive stress causes inflammation at the growth plate of the distal radius and microtrauma can lead to premature closure of the distal radial physis resulting in secondary overgrowth of the ulna. Several cross-sectional studies have demonstrated significant correlations between training intensity and ulna-radial-length-discrepancy, suggesting a dose-response relation.

     

    Fallon et al. reported on a 10-year-old female gymnast who sustained a stress fracture of the medial clavicle. In this report, the patient was successfully managed with conservative treatment and pathologic causality was excluded. They concluded that when an acute trauma has been excluded and bony injury is suspected, stress fracture should be considered even if the symptoms and signs appear to arise from an unusual site.

     

    Sinha et al. reviews stress fracture in a non-weight-bearing location of the upper extremity or ribs in athletic individuals. In this multicenter study, they report on 44 patients diagnosed with an upper extremity or rib stress fracture. They concluded that upper extremity weight bearers (e.g., gymnastics, diving, cheerleading) all had fractures occurring distal to the elbow.

     

    Caine et al. reviewed stress-related injuries of the distal radius in female gymnasts to determine if repetitive physical loading inhibits growth of the radius in female gymnasts. They reported radiographic abnormalities consistent with distal radius physeal-stress reaction in up to 85% of gymnasts studied. Several cross-sectional studies showed significant correlations between training intensity and growth disturbance. An inconclusive association of stress-related distal radius physeal arrest with repetitive gymnastic activities was made.

     

    Figure A: AP radiograph of the right shoulder demonstrating widening of the proximal humerus physis consistent with a stress reaction

    Illustrations Aand B: AP and lateral radiographs demonstrating widening of the distal radius physis found in "gymnast's wrist"

     

    Incorrect Answers:

    Answer 1: Scaphoid stress fractures have been associated with gymnasts, however, the risk of growth arrest is less than a stress fracture of the distal radius

    Answer 2: Distal humerus stress fractures have been reported in gymnasts. The risk of growth arrest is less than a stress fracture of the distal radius Answer 3: Clavicle fractures are extremely common in the pediatric population. Stress fractures of the clavicle are rare but have been reported in the gymnast.

    Answer 4: Shoulder stress reactions are more common in throwing and overhead athletes. Widening of the proximal humeral physis, as illustrated in Figure A, suggests proximal humeral physeal separation.

     

     

     

     

     

     

  122. A 47-year-old female presents with a pulsatile mass in the palm of the hand and intermittent paresthesias a few weeks after a traumatic laceration in a kitchen accident. A contrast-enhanced MRA of the mass is seen in Figure A. What is the most appropriate treatment?

     

     

     

     

    1. Aspiration

    2. Surgical exploration

    3. Compression bandage

    4. Steroid injection

    5. Strict elevation and observation

    CORRECT ANSWER: 2

    The contrast-enhanced MRA shown in Figure A depicts a false aneurysm of the superficial palmar arch. The most appropriate treatment for this symptomatic lesion is surgical exploration.

     

    Traumatic palmar artery aneurysms and pseudoaneurysms are rare and can be caused by penetrating or blunt trauma (e.g., during endoscopic carpal tunnel release). The case reports in the literature all support surgical exploration with either ligation, excision, or repair depending on the extent of the lesion.

     

    Yajima et al. report on the management of three cases of digital artery aneurysms after traumatic lacerations. Two cases were treated with excision only, while one was treated with excision and vascular reanastomosis.

     

    Figure A is a contrast-enhanced MRA showing a false aneurysm of the superficial palmar arch.

     

    Incorrect Answers:

    Answer 1, 3-5: Aspiration, compression bandage, steroid injection and observation are not the most appropriate modalities of treatment for false aneurysms of the superficial palmar arch.

     

     

     

  123. Your partner, a pediatric hand surgeon, sees a child with a congenital upper extremity deformity. A radiograph is shown in Figure

    A. What is a relative contraindication to performing a hand centralization procedure in this patient?

     

     

     

    1. Diminished grip strength

    2. Unstable carpus

    3. Cardiac defects

    4. Limited terminal elbow extension

    5. Current patient age CORRECT ANSWER: 5

    In a patient with radial longitudinal deficiency (RLD), a centralization procedure is not generally offered until 12 months, when it is considered safer for the child to undergo anesthesia. While the patient's age is not given, he is much younger than 12 months based on the absence of any ossification of the carpus (as the capitate should show some calcification at 1-2 months and the lunate at 2-4 months, with both very prominent by 12 months of age).

     

    RLD, or radial clubhand is relatively rare, with a cited incidence of 1:30,000-100,000 live births. There is a slight male predilection (3:2), and RLD is bilateral in 38-58% of cases. RLD is often associated with syndromes including VATER/VACTERL, TAR, Fanconi's anemia, and Holt-Oram. There is a spectrum of RLD, including a short radius with the distal epiphysis present (type 1), a hypoplastic radius with defective distal epiphysis (type 2), partial absence of radius with a proximal defect (type 3), and a completely absent radius (type 4, which is the most severe but most common). While associated syndromes must be ruled out, mild forms may be treated with simple observation or splinting and stretching. Complete absence however may necessitate a hand centralization procedure unless otherwise contraindicated. Centralization involves resection of a portion of the carpus, shortening of the extensor carpi

    ulnaris, and potentially an angular osteotomy of the ulna, sparing the distal ulna physis. The contraindications to centralization include older patients with good function, limited elbow flexion (less than 90 degrees), and severe soft tissue contracture.

     

    Geck et al. evaluated the outcomes of 29 limbs with RLD who underwent radialization or modified centralization. The authors found that all patients had improvement on hand-forearm angle and hand-forearm position, regardless of the procedure. They noted that risk factors for revision surgery included positive hand-forearm angle and young age at the time of index procedure.

    Furthermore, for every 6 months younger that the procedure was performed, the risk of revision was doubled. As such, the authors advised on waiting to centralize until after 12 months.

     

    Maschke et al. presented a comprehensive review of RLD. They distinguished between isolated and syndromic RLD, specifically noting that while patients with TAR lack a radius, they often do have an intact thumb. The authors highlighted that all patients presenting with RLD should undergo a renal ultrasound, echocardiogram, and a complete blood count to evaluate for the potentially severe associated medical conditions. They concluded that the long-term health of the patient should guide the goals for limb reconstruction.

     

    Incorrect answers:

    Answer 1: Patients with radial clubhand have reduced grip strength, but this is not a contraindication to centralization

    Answer 2: An unstable carpus is not a contraindication, - on the contrary it can be improved with a forearm centralization procedure.

    Answer 3: Many patients will have cardiac defects, especially in those with Holt-Oram, but this is not a contraindication to centralization.

    Answer 4: Limited elbow FLEXION and not extension would be a contraindication, because a centralization procedure may further prevent the patient from being able to bring their hand to their mouth for feeding or hygiene.

     

     

     

  124. A 2-year-old infant presents to your nationally-renowned pediatric hand clinic with the hand deformities shown in Figure A. As the foremost expert in the field, you counsel the parents that which of the following is associated with this patient's condition:

     

     

     

    1. Unremarkable gestational history

    2. Anterolateral tibial bowing

    3. Congenital oblique talus

    4. Congenital vertical talus

    5. Congenital talipes equinovarus

    CORRECT ANSWER: 5

    This child presents with Amniotic Band Syndrome (also called constriction ring syndrome, or Streeter's dysplasia), which has been shown to be associated with congenital talipes equinovarus, or club foot.

     

    Streeter's dysplasia has been found to occur in 1:15,000 live births and affects both sexes equally. While the etiology is contended, the condition is known to be sporadic and not hereditary. The most supported theory involves the formation of membranous strands from premature amnion rupture that then circumstantially wrap and constrict the developing fetus in utero. Appendages are most often affected, but constriction of the torso or head may result in a variety of severe cardiac, pulmonary, gastrointestinal, or cranial abnormalities. Of the limbs, the hands are most commonly affected, resulting in a spectrum of deformity ranging from amputation to syndactylization. Streeter's dysplasia has additionally been shown to be associated with congenital talipes equinovarus. Prognosis is related to the severity of the constricting bands. In severe cases, constriction bands may need to be released, require Z-plasties to address contracture or necessitate reconstruction of involved digits.

     

    Foulkes and Reinker reviewed 71 cases of children with congenital constriction band syndrome. They noted that patients averaged three involved limbs, most often the distal aspects of central digits of the upper extremity. Abnormal gestational histories were found in 60% of cases and 33% of these patients had club feet. Furthermore, the average patient had a 20% whole body impairment and required a mean of three reconstructive procedures. The authors believe that the term "early amnion rupture sequence" accurately reflects the pathogenesis of congenital construction band syndrome.

     

    Moran et al. comprehensively review amniotic band syndrome. The authors

    attribute the etiology to strands of amniotic tissue which form bands, entrapping and deforming fetal tissue, most often involving the hand. They noted that due to the varied manifestations, treatment is considered on a case-by-case basis. In cases of distal acrosyndactyly and edema of the hand, earlier soft tissue reconstruction or release likely improves prognosis. The authors concluded that improvements in prenatal diagnosis may one day lead to in utero treatment at some point in the future.

    Figure A shows a left hand with constriction band syndrome. Incorrect Answers:

    Answer 1: Abnormal gestational history is present in the majority of cases.

    Answer 2: Anterolateral tibial bowing is associated with Neurofibromatosis type I but has not been demonstrated in patients with amniotic band syndrome.

    Answers 3 and 4: Congenital oblique talus and congenital vertical talus are not associated with congenital constriction band syndrome, however, one-third of patients will have congenital talipes equinovarus.

     

     

     

  125. All of the following are considered the cardinal signs of flexor tenosynovitis EXCEPT:

    1. Tenderness along the flexor tendon sheath

    2. Flexed resting posture of the finger

    3. Fusiform swelling of the finger

    4. Pain on passive extension of the finger

    5. Pain on passive flexion of the finger

      CORRECT ANSWER: 5

      The cardinal signs of pyogenic flexor tenosynovitis (Kanavel signs) include:

      1. partially flexed resting posture of the involved finger

      2. pain with passive extension

      3. fusiform swelling of the finger

      4. volar tenderness along the flexor sheath

       

      Patients commonly present 24 to 48 hours after onset of symptoms. The standard of care is “urgent surgical drainage” to avoid tendon scarring or necrosis with subsequent impairment of finger function followed by intravenous antibiotic administration.

       

      According to Hand Surgery Update 3, open sheath irrigation has been replaced

      largely by closed sheath irrigation. These authors cite a retrospective study that showed no statistical difference in resolution of infection using open sheath irrigation or closed sheath irrigation, however, there was a trend towards more frequent complications and reoperations in the open drainage group.

       

      Lille et al reviewed the records of 75 patients with pyogenic flexor tenosynovitis and found that there was no difference in outcomes between those who received intraoperative irrigation only versus those receiving intraoperative irrigation and continuous postoperative irrigation.

       

       

       

  126. A 50-year-old woman is diagnosed with carpal tunnel syndrome. She is prescribed a cock-up wrist splint at 30 degrees of extension to wear at night. This splint has what effect on the carpal tunnel?

    1. Decreases carpal tunnel pressure

    2. Increases carpal tunnel pressure

    3. No effect on carpal tunnel pressure

    4. Enlarges the carpal tunnel volume

    5. Improves nerve conduction studies

    CORRECT ANSWER: 2

    This question is based on the fact that carpal tunnel canal pressure varies with wrist position. Use of neutral wrist splints for carpal tunnel syndrome is most useful for improving noctural symptoms. The reason for this is the functional position of the wrist is approximately 30 degrees of extension, and the neutral splints can be functionally limiting when used during productive daytime hours.

     

    The reference by Gerritsen et al is a randomized controlled study of splinting versus surgery for carpal tunnel. They found a 80% success rate for surgery at final follow-up versus 54% for splinting at 3 months, which increased to 90% at 18 months for surgery and 75% for splinting.

     

    The reference by Omer is a review of carpal tunnel, and it covers the diagnosis, treatment, and follow-up care of these patients. They note the need for careful diagnosis to avoid unnecessary or inappropriate surgery.

     

    Weiss et al showed that carpal tunnel pressures are elevated when the wrist is in extension, and are lowest at near neutral. If one couples this with the

    inherent tunnel pressure increase from the disease itself, its easy to see that extension splinting is a double hit and can lead to increased symptoms.

     

     

     

  127. A 28-year-old factory worker has his ring finger caught in the machinery at work. A photograph of the injury is shown in Figure A. Which of the following procedures will best supply coverage of the wound?

     

     

     

     

    1. Amputation through the proximal interphalangeal joint

    2. Shortening of the distal phalanx, nail bed removal, and V-Y flap

    3. Cross-finger flap

    4. Groin flap

    5. Thenar flap CORRECT ANSWER: 2

    The clinical presentation is consistent with a transverse fingertip amputation. Shortening of the distal phalanx, nail bed removal, and dorsal V-Y flap would be the most appropriate treatment.

     

    The V-Y flap is useful for extending dorsal skin to cover a transverse or dorsally angulated fingertip injury. They are typically used for finger tip amputations which have more dorsal soft tissue loss than palmar loss. Nail bed removal is important to prevent a subsequent hook nail deformity.

     

    Fassler reviews the proper management of fingertip injuries including the different flap coverage outlines discussed above. If there is no exposed bone, healing by secondary intention is acceptable. If bone is exposed in the wound but there is enough nail bed remaining to support a nail plate, then a local advancement flap is a good option. If local flap coverage is not feasible due to

    the extent of the soft tissue injury, a regional flap (cross-finger or thenar) is necessary. If the wound is proximal to the nail bed, then shortening and primary closure is most reasonable.

     

    Figure A shows a transverse fingertip amputation. Illustration A shows the different zones of fingertip amputations: Zone I is distal to the phalanx; Zone II is distal to the lunula; Zone III is proximal to the lunula. Illustration B shows the basic steps of a V-Y flap.

     

    Incorrect Answers:

    There is too much exposed bone remaining to simply cover with sterile dressings.

    Answer 1: An amputation would not be indicated.

    Answer 3: Cross-finger flaps are useful for fingertip injuries with volar tissue loss only.

    Answer 4: A groin flap would not be indicated in this clinical situation.

    Answer 5: Thenar flaps are reserved for index and middle fingertip injuries and carry a risk of postoperative flexion contractures.

     

     

     

     

     

     

     

     

  128. Which of the following amputations may be considered a relative contraindication for a replantation?

    1. Ring finger through the proximal phalanx shaft

    2. Mid forearm

    3. Thumb through the proximal phalanx

    4. Middle, ring, and small fingers through the middle phalanx shaft

    5. Complete hand just proximal to the distal palmar crease

    CORRECT ANSWER: 1

    Replantation of a single finger amputated proximal to the insertion of the flexor superficialis tendon is a relative contraindication because of the severe stiffness and poor function encountered after repairs in this location.

     

    The FDS insertion is in the middle of the middle phalanx and is also what

    defines the distal border of Zone II in flexor tendon injuries. If the finger is cut proximal to the insertion of FDS, that means that FDS is also cut and needs to be repaired leading to severe stiffness and worse PIP function. The exceptions are when there are multiple digits injured or in young children.

     

    Urbaniak et al found replantation of a single finger amputated distal to the insertion of the flexor superficialis tendon is justified, but that replantation of a single finger that was amputated proximal to this insertion is not indicated.

     

    Tamai et al found that an amputation of the hand, forearm, arm, and thumb, as well as multiple digits are all criteria for replantation.

     

     

     

  129. A 27-year-old professional cowboy is thrown from a bull during the rodeo and lands on his hand. No deformity is identified and the hand is completely neurovascularly intact. Pain is present upon palpation of the anatomic snuffbox. A radiograph is provided in Figure

    A. The cowboy wants to return to competitive riding tomorrow. Which of the following is the best next step in management?

     

     

     

     

    1. Cock-up wrist splint and immediate return to sport as tolerated by pain

    2. Steroid injection of the snuffbox, taping of the wrist and return to sport

    3. Wrist MRI

    4. Percutaneous screw fixation of the nondisplaced fracture

    5. Scapholunate ligament repair and percutaneous pin fixation

    CORRECT ANSWER: 3

    Tenderness with palpation of the anatomic snuffbox should raise suspicion of a scaphoid fracture. The radiograph does not show any findings, but scaphoid fractures are often not initially visualized on plain radiographs. Appropriate treatment for any patient with snuffbox tenderness entails cast immobilization with repeat radiographs at 2-3 weeks or advanced imaging with MRI to evaluate for a fracture that is not identified with plain radiographs.

     

    The MRI that correlates with this patient's radiograph is provided in Illustration A and demonstrates a nondisplaced scaphoid fracture.

     

    Gaebler et al performed an MRI study of 32 consecutive patients who were clinically suspicious for a scaphoid fracture, but no fracture could be indentified on wrist radiographs. The MRI was 100% sensitive and specific in diagnosing scaphoid fracture.

     

    Treatment for this patient following the MRI would be debatable. Cast immobilization would be appropriate, but screw fixation may allow earlier return to sport. A percutaneous compression screw would be an appropriate technique for this scaphoid fracture.

     

     

     

     

     

  130. All of the following are possible sites of compression for the ulnar nerve EXCEPT:

    1. arcade of Struthers

    2. ligament of Struthers

    3. flexor carpi ulnaris fascia

    4. medial intermuscular septum

    5. Osborne's ligament CORRECT ANSWER: 2

      There are five sites of potential ulnar nerve entrapment around the elbow: arcade of Struthers, medial intermuscular septum, medial epicondyle, cubital tunnel, and deep flexor pronator aponeurosis.

       

      The ulnar nerve emerges from the medial intermuscular septum, under the arcade of Struthers, and lies on the medial head of the triceps. At the level of the elbow, the ulnar nerve continues distally toward the posterior aspect of the condylar groove, passing between the medial epicondyle and olecranon to enter the cubital tunnel. The roof is formed by the arcuate (Osborne’s) ligament. This ligament blends distally with the antebrachial fascia superficial

      to the aponeurosis and connects the ulnar and humeral heads of the FCU. The ligament of Struthers is a fibrous band extending from the supracondylar process of the humerus to the medial epicondyle which can cause compression of the median nerve.

       

      Elhassan et al discuss the pathogenesis, evaluation, and treatment of entrapment neuropathy of the ulnar nerve.

       

      Illustration A shows the various site of compression at the elbow. Illustration V shows a submuscular ulnar nerve transposition performed Dr. Susan E. Mackinnon

       

       

       

       

       

       

  131. Repair of a nailbed injury with 2-octylcyanoacrylate (Dermabond) provides what distinct advantage over standard suture repair?

    1. Better ultimate cosmesis

    2. Better functional outcome

    3. Faster procedure

    4. Less pain at follow up

    5. Lower infection rate CORRECT ANSWER: 3

    Octylcyanoacrylate (Dermabond) has been found to be a viable method in nailbed repair, and has the advantage of being a faster procedure.

     

    Strauss et al performed a level 1 randomized trial of 2-octylcyanoacrylate (Dermabond) versus suture repair of nailbed injuries. They found the only

    significant difference was that 2-octylcyanoacrylate had a shorter procedure time. All other indices were similar.

     

    Richards et al provide a description of their technique using dermabond to secure the nail following a nailbed repair. They found good results with no complications in their cohort of 22 patients, and recommend it as a technique.

    Illustration A shows a nail bed repair. Incorrect Answers

    Answer 1,2,4,5: There is no difference in these outcomes when comparing 2-

    octylcyanoacrylate (Dermabond) versus suture repair

     

     

     

     

     

     

  132. A 50-year-old man complains of numbness and tingling along his right small finger. Physical exam is notable for the finding demonstrated in Figure A. Elbow flexion reproduces the numbness and tingling. Physical therapy and splinting have failed to relieve the symptoms. Which of the following is the most appropriate surgical intervention to alleviate the symptoms while minimizing complications?

     

     

     

    1. Simple ulnar nerve decompression at the cubital tunnel

    2. Ulnar nerve decompression at the cubital tunnel with anterior submuscular transposition

    3. Ulnar nerve decompression at the cubital tunnel with anterior subcutaneous transposition

    4. Open carpal tunnel release

    5. Endoscopic carpal tunnel release

    CORRECT ANSWER: 1

    The patient's clinical presentation and physical exam are consistent with cubital tunnel syndrome. The clinical photograph demonstrates Froment's sign; compensatory IP hyperflexion of FPL (AIN) to compensate for the loss of adductor pollicis (ulnar nerve) during key pinch. Simple decompression of the ulnar nerve is less invasive and achieves clinical outcomes equivalent to decompression with transposition.

     

    Zlowodzki et al conducted a meta-analysis evaluating anterior transposition and simple decompression of the ulnar nerve. No difference in motor nerve-conduction velocities or clinical outcome scores was found.

     

    Bartels performed a prospective randomized trial (included in the Zlowodski meta-analysis) on 152 patients comparing simple decompression to transposition. No difference in clinical results at 1 year were reported, but a significantly higher complication rate occurred in the transposition group (31%) compared to simple decompression (9.6%).

     

    Nabhan et al performed a level 1 study randomizing 66 patients to simple decompression or subcutaneous ulnar nerve transposition. No differences were

    found with respect to clinical outcome or nerve conduction velocities.

     

    Illustration V is an educational presentation discussing ulnar nerve transposition at the elbow.

     

     

     

  133. A 22-year-old male snowboarder falls on an outstretched hand and presents with the radiograph shown in Figure A. Which of the following techniques is MOST important in optimizing biomechanical fixation?

     

     

     

     

    1. Using a screw placed in the central axis of the scaphoid into the subchondral bone

    2. Using a supplementary K-wire transfixing the distal pole of the scaphoid to the capitate

    3. Using a screw placed in the dorsal axis of the scaphoid into the subchondral bone

    4. Using a larger diameter screw placed in the dorsal axis of the scaphoid

    5. Using a larger diameter screw placed in the volar axis of the scaphoid

    CORRECT ANSWER: 1

    Several studies have shown a longer screw placed in the central axis of the scaphoid optimizes biomechanical fixation of scaphoid waist fractures. Many studies have discussed the amount of compression generated by various internal fixation screws (e.g headless vs. headed, variable pitch, partially vs.

    fully threaded, cannulated vs. noncannulated), but it is believed that rigidity of fixation is probably the most important factor in promoting healing of scaphoid fractures.

     

    The first reference by McCallister et al is a cadaveric, biomechanical study that demonstrated a centrally placed screw had 43% more stiffness than an eccentrically placed screw. They recommend using surgical techniques that optimize central placement and screw length, such as using a cannulated screw.

     

    The study by Dodds et al supported these findings and added that a longer screw with 2mm of bone coverage provided greater stability than a shorter screw. A more centrally placed screw is generally longer and has more length of screw on each side of the fracture than does a peripherally placed screw due to the anatomic dimensions of the scaphoid.

     

     

     

  134. A 32-year-old male sustains a 100% tear of his flexor tendon in the Zone 2 region after cutting his finger with a knife. You plan a one-stage repair of the flexor tendon. Which of the following variables has the greatest effect on increasing the strength of the tendon repair?

    1. The size of the core suture

    2. Number of core strands crossing the repair site

    3. Use of epitendinous suture

    4. Active range of motion during the immediate postoperative period

    5. Repair of the flexor tendon sheath

    CORRECT ANSWER: 2

    The single most effective intervention for increasing strength of a flexor tendon repair is to increase the number of core sutures crossing the repair site.

     

    Hatanaka and Manske found that locking loops were better than grasping loops, and that a higher core suture diameter led to an increase in strength. It is well known that adding an epitendinous suture increases the repair up to 10-50% in strength depending on the depth of the suture. There is no

    evidence that fixing the flexor sheath after repair increases the strength of the repair nor does it lead to improved outcome. Postoperative active range of motion would increase excursion thus decreasing the number of potential adhesions. Active range of motion of a repaired tendon can facilitate intrinsic over extrinsic tendon healing and increase tendon tensile strength, but the

    magnitude of this effect is secondary to the number of sutures crossing the repair site.

     

     

     

  135. A 6-year-old boy sustained a finger tip amputation shown in Figure A after grabbing a broken glass out of the dishwasher. Your plan was to perform a bedside irrigation and debridement of the finger after digital anesthetic block and apply antibiotic ointment with a sterile dressing. Upon exploration of the wound you notice that distal phalanx is exposed. Your plan should change to include which of the following treatments?

     

     

     

     

     

    1. Homodigital island flap

    2. Thenar flap

    3. Volar flap advancement

    4. V-Y advancement flap

    5. No change from your initial plan of ointment and dressing

    CORRECT ANSWER: 5

    In young children with a fingertip amputation, ointment and dressing changes is the most appropriate treatment even if bone is exposed.

     

    When deciding on a treatment, consideration of a "reconstruction ladder" is helpful in determining the least invasive procedure to obtain the optimal outcome. The ladder includes primary closure, healing by secondary intention, split-thickness skin grafts, full-thickness skin grafts, random pattern local flaps, axial pattern local flaps, island pattern local flaps, distant random pattern flaps, distant axial pattern flaps, and free tissue transfer.

    Lamon et al reviewed 25 patients, with an average age of 30 years old, with fingertip injuries treated with dressings and warm soaks started 2 days after injury and noted no healing complications. Only one patient in this cohort had bone exposed.

     

    Soderberg et al performed a Level 3 study of 36 operative and 34 conservatively treated fingertip amputations with bone exposure and found no benefit to surgery.

     

    Farrell et al conducted a Level 4 review of 21 fingertip amputations with 6 having exposed bone and concluded that they healed with excellent results in regards to contour, sensation, and finger length.

     

    Illustration A shows a homodigital island flap. Illustration B shows a thenar flap. Illustration C shows a volar flap advancement. Illustration D shows a volar V-Y flap advancement.

     

     

     

     

     

     

     

     

     

     

     

     

     

     

  136. The Bilhaut-Cloquet procedure for thumb duplication, where the central portions of bone and nail are removed and the radial half of one thumb is combined with the ulnar half of the other to create one thumb, is most appropriate in which Wassel Type shown in Figure A?

     

     

     

    1. Type 2

    2. Type 4

    3. Type 5

    4. Type 6

    5. Type 7

     

    CORRECT ANSWER: 1

     

    The Bilhaut-Cloquet procedure has been shown to be successful in Wassel Type 1, 2, and 3 deformities. Asymmetrical Type I or II duplications with distinct components may also be treated with ablation of the smaller thumb with transfer of the collateral ligament and centralization of the extensor tendon. Types 3 and 4 duplications (Type 4= most common duplication) are often treated with selection of a dominant thumb and ablation of the lesser counterpart after preservation and transfer of intrinics and collateral ligaments.

     

    The article by Ogino states that the ablation of the radial thumb is most common and Miller's Review states radial thumb ablation is preferred.

    Treatment of Types 5 and 6 duplication utilizes similar principles as Type 4 deformities, with the added complexity of additional intrinsic reconstruction. Type 7 is a triphalangeal thumb and has variable treatment based on the presentation, and reconstruction includes excision versus fusion of the extra phalanx.

     

    Baek el al conducted a Level 4 review of 7 patients with Type 3 and Type 4 duplication treated with an extra-articular modified Bilhaut-Cloquet and found excellent IP joint range of motion, no nail deformities, and no episodes of growth arrest.

     

     

     

  137. An open dorsal approach for antegrade screw fixation of a nondisplaced scaphoid waist fracture differs in which of the following ways compared to a percutaneous dorsal approach?

    1. Decreased risk of proximal pole AVN

    2. Increased risk of posterior interosseous nerve injury

    3. Decreased risk of injury to the APL tendon

    4. Increased risk of injury to the EPL tendon

    5. Decreased risk of screw prominence above subchondral bone

    CORRECT ANSWER: 5

    Scaphoid screw fixation should be just below the subchondral bone; this is best judged by direct visualization.

     

    Adamany et al in an anatomic study using fluoroscopy to insert a scaphoid screw via a percutaneous approach found that the scaphoid screw "was prominent (above the subchondral bone) in 2 of 12 specimens and flush with or buried in the remaining 10 specimens." As a result, they recommend using a limited dorsal incision to verify full seating of the screw. In addition, they found the percutaneous approach was within 2.2-3.1 mm of the PIN, EDC, and EIP. Thus, all of these structures are at increased risk of injury in a percutaneous approach. The APL tendon is not in the surgical field. Illustration A shows the AIN(arrowhead) is deep in relation to pronator quadratus.

    Sensory remnant of posterior interosseous nerve (straight thick arrow) is adjacent to interosseous membrane. White arrow is median nerve. Shaded open arrow is ulnar nerve, and long thin arrow is superficial radial nerve.

     

    Tumilty et al inserted a Herbert screw through a dorsal approach in 12 cadaveric wrists. They then imaged them with AP/Lateral xrays, and 360 degree fluoroscopic views. The wrists were then dissected to evaluated for subchondral penetration, and plain x-ray films were accurate in 5 of 6 specimens. Fluoroscopy was accurate in all 6. They concluded that fluoroscopy during placement of the Herbert screw may decrease the rate of subchondral penetration.

     

     

     

     

     

     

  138. You are taking care of an adult patient with significant scar contracture in her first web space after a thermal burn. Which of the following techniques will allow you to lengthen her scar approximately 75%?

    1. Cross-finger flap

    2. Two-flap Z-plasty with 60 degree limbs

    3. Two flap Z-plasty with 25 degree limbs

    4. Island pedical flap

    5. Split-thickness skin graft

    CORRECT ANSWER:

    2

    One of the most commonly used techniques for lengthening scar contracture in hand surgery is the Z-plasty. When the two 60 degree triangular flaps are transposed and closed, the original direction of the scar is rotated and the scar length is increased by approximately 75% Because of its history the 60 degree Z-plasty is the technique to which other methods of contracture lengthening are compared.

     

    Hove et al describe the technique, various applications, and different types of Z-plasty used today. Neither the cross-finger flap nor island pedical flap are useful for this amount of scar release. Two flap Z-plasty with 25 degree limbs does not offer enough lengthening. Split-thicknes skin grafts are not useful for either lengthening or the volar aspect of the hand due to the significant contracture they experience.

     

    Illustration A and B depict the Z-plasty technique.

     

     

     

     

     

     

     

     

  139. A 10-year-old boy presents with a painless mass on the dorsal aspect of his wrist that has been present for 3 weeks. A clinical image is shown in Figure A. T1 and T2 magnetic resonance images are shown in Figure B and C, respectively. On your exam, the mass transilluminates and Allen test reveals patent radial and ulnar arteries. What is the most appropriate next step in management?

     

     

     

     

     

     

     

     

    1. Referral to a orthopaedic oncologist

    2. Surgical excision with wide margins

    3. Observation

    4. Autologus bone marrow aspirate injection

    5. Injection of N-Butyl-Cyanoacrylate

    CORRECT ANSWER: 3

    This child has a ganglion cyst on the dorsal aspect of his wrist. Imaging provided shows a well-marginated, homogenous signal intensity mass consistent with a ganglion cyst. Physical examination findings of a mass transilluminating corroborate the MRI findings of a ganglion cyst. Performing an Allen's test to evaluate radial and ulnar artery collateral blood flow is especially important when evaluating ganglion cysts on the volar aspect of the wrist as they are often adjacent to the radial artery. Wang et al. peformed a Level 4 review of 14 children with hand and wrist ganglion cysts and found that 79% of these cysts resolved spontaneously within 1 year. Autologus bone marrow aspirate injection is a treatment option for unicameral bone cysts and N-Butyl-Cyanoacrylate injections have been described for treatment of hemangiomas. Referral to an orthopaedic oncologist is not indicated.

     

     

     

  140. Chronic injury to what anatomic structure can lead to a boutonnière deformity of the finger?

    1. terminal extensor tendon

    2. sagittal band

    3. volar plate

    4. flexor digitorum profundis tendon insertion

    5. central slip of the extensor tendon

    CORRECT ANSWER: 5

    Rupture of the central slip of the extensor tendon and subsequent subluxation of the lateral bands leads to a boutonnière deformity, which is characterized by PIP flexion and DIP extension. Central slip injuries can be caused by a laceration or traumatic avulsion. In the listed reference, Imatami et al treated a series of central slip injuries associated with attachment fractures successfully with ORIF. As stated by Tuttle et al, rupture of the terminal extensor tendon leads to a mallet finger. Sagittal band injury can lead to subluxation of the extensor tendon at the level of the MCP joint. Chronic volar plate injuries can lead to swan neck deformities. Avulsion of the FDP insertion leads to a jersey finger. Illustrations A and B are a clinical photograph and anatomic diagram of a boutonneire deformity.

     

     

     

     

     

     

     

     

  141. You are seeing a 26-year-old man after he was involved in a knife fight. He has pain when flexing and extending his index finger. You explore a 2 centimeter wound in zone 2 and find his flexor tendons to the index are 40% lacerated. What is the preferred method of treatment?

    1. Trim the frayed tendon edges and begin early range of motion

    2. Trim the frayed tendon edges and cast in an intrinsic positive position for 2 weeks

    3. Peritendinous 6/0 and Core 4/0 suture repair

    4. Core 4/0 suture repair

    5. Core 6/0 suture repair CORRECT ANSWER: 1

    This patient has a partial flexor tendon laceration involving < 60% of the width of the tendon, therefore, the preferred management would be to trim the frayed tendon edges and begin early range of motion.

     

    Flexor tendon injuries are classified into five anatomic zones. Injuries in zone II, which ends at the insertion of the FDS tendon at the middle phalanx, are particularly challenging because the tendon gliding must be restored within a tight fibro-osseous sheath while minimizing the formation of adhesions in surrounding tissues.

     

    Bishop et al. developed a nonweightbearing canine model to examine partial tendon lacerations and found early motion improved tendon excursion and stiffness, resulting in more normal tendon morphology. They concluded that partial tendon lacerations less than 60% cross-sectional area be treated without tenorrhaphy and with early mobilization.

     

    McGeorge et al. compared the results of repair versus non-repair in patients with zone II tendon lacerations and concluded that tendons lacerated by 60% or less should not be repaired.

     

    Illustration A depicts the palmar view of the hand highlighting the anatomical classification of flexor tendon injuries. Illustration B shows a lateral view of a finger demonstrating the relationship between FDP and FDS within zone II.

     

    Incorrect Answers:

    Answer 2: Casting is not indicated in this patient. Early range of motion is preferred as it allows for improved tendon excursion and morphology.

    Answer 3, 4 & 5: Flexor tendon repair is not indicated in this patient as it is

    recommended in patients with lacerations >60% of the tendon width.

     

     

     

     

     

     

     

     

     

     

  142. A 60-year-old man has chronic pain at the base of this thumb and weakness on attempted thumb pinch. A radiograph is shown in Figure

    A. Which injection would likely most offer the greatest degree of symptomatic improvement?

     

     

     

    1. Saline

    2. Steroid

    3. Hylan

    4. All of the above are equally effective

    5. All of the above are detrimental

    CORRECT ANSWER: 4

    The patient has basal joint arthritis of the thumb and randomized controlled trials have failed to demonstrate an advantage of steroid or hylan over saline.

     

    Heyworth et al demonstrated that all three injections were similarly effective for approximately 3 months at reducing pain and increasing thumb function over baseline levels.

     

    Stahl found that steroid and hylan were equally effective, but did not control with saline.

     

    Henderson found no advantage of hyaluronan over saline for knee arthritis during a 5 week treatment course.

     

     

  143. Which of the following upper extremity congenital anomalies, represented in the figures below, if found in an individual of Caucasian descent requires a work-up for chondroectodermal dysplasia or Ellis-van Creveld syndrome?

     

     

     

     

     

     

     

     

     

     

     

     

     

     

    1. Figure A

    2. Figure B

    3. Figure C

    4. Figure D

    5. Figure E CORRECT ANSWER: 2

    Postaxial polydactyly is rare in Caucasian individuals and deserves further workup for underlying syndromes. Postaxial polydactyly is ten times more common in African Americans and does not require further workup.

    Constriction band syndrome or amniotic band syndrome is a type of pseudosyndactyly(Figure A) and is not the result of failure of differentiation during embryogenesis, but a result of injury by bands after the fingers are formed. Preaxial polydactyly (Figure C) is more common in Caucasians and is usually sporadic except for triphalangism which is associated with Holt-Oram and Fanconi's Anemia. Syndactyly (Figure D) is defined as an abnormal interconnection between adjacent digits and syndactyly variations are associated with Apert syndrome and Poland syndrome. Macrodactyly (Figure E) represents overgrowth of all structures of the involved digit and is associated with neurofibromatosis and Klippel-Trenaunay-Weber syndrome. The referenced article by Kozin is an excellent Current Concepts Review that summarizes upper limb embryogenesis and the most common anomalies.

     

     

     

  144. A 6-year-old boy presents to your office with a forearm deformity and the radiographs shown in Figure A. What additional tests must be ordered for this patient?

     

     

     

    1. CBC

    2. CBC and renal ultrasound only

    3. CBC, renal ultrasound, and echocardiogram

    4. CBC and MRI

    5. No additional testing needed

    CORRECT ANSWER: 3

    The clinical presentation is consistent with radial clubhand or radial longitudinal deficiency. A CBC, renal ultrasound, and echocardiogram should be obtained to screen for associated congenital abnormalities.

     

    Radial clubhand is commonly associated with thrombocytopenia-absent radius (TAR), Fanconi's anemia, Holt-Oram syndrome (congenital heart disease), and VACTERL (vertebral, anal, cardiac, tracheal, esophageal, renal, and limb) syndromes. A CBC, renal ultrasound, and echocardiogram are the basics that must be ordered to screen for the other associated conditions. If Fanconi's anemia is suspected, chromosomal breakage studies using diepoxybutane or mitomycin C allow for detection of this disorder prior to the onset of pancytopenia. It is important to obtain complete blood counts as the hematopoietic system is most commonly involved.

    Maschke et al. emphasize that all children presenting with radial longitudinal deficiency, regardless of severity, require a renal ultrasound, echocardiogram, and complete blood count to evaluate the potential for associated systemic conditions. They report addition genetic testing may be warranted to rule out specific conditions, such as Fanconi's anemia.

     

    Goldfarb et al. reviews thumb function in these patients. They report that while the thumb in TAR syndrome patients is of relatively normal size and shape, due to its position of metacarpophalangeal flexion, function is significantly impaired.

     

    Goldfarb et al. evaluated 164 patients with radial clubhand or associated congenital defects. Of these 25 patients had thrombocytopenia absent radius syndrome; 22 patients had vertebral, anal, cardiac, tracheoesophageal, renal, and limb abnormalities association; 7 patients had Holt-Oram syndrome; and 1 patient had Fanconi anemia. There were 32 patients with cardiac abnormalities and 60 patients with spinal or lower-extremity musculoskeletal abnormalities. They therefore recommend, in addition to a CBC, renal ultrasound, and echocardiogram, that spinal radiographs be obtained.

     

    Figures A demonstrates the radiographs of a radial clubhand or radial longitudinal deficiency.

     

     

     

  145. What is the most frequently encountered form of osseous injury associated with dorsal proximal interphalangeal joint(PIP) fracture-dislocations?

    1. Middle phalanx palmar lip fractures

    2. Proximal phalanx extraarticular fractures

    3. Middle phalanx dorsal lip fractures

    4. Middle phalanx dorsal and palmar lip fractures (pilon)

    5. Proximal phalanx palmar lip fractures

    CORRECT ANSWER: 1

    Middle phalanx palmar lip fractures are the most frequently encountered form of osseous injury associated with dorsal PIP joint fracture-dislocations. Pure PIP joint hyperextension often disrupts the palmar plate either at its distal insertion or by creating a tension fracture at the palmar lip of the middle phalanx.

     

    The review article by Kiefhaber and Stern detail that the restoration of the

    middle phalangeal base to glide around the proximal phalangeal head during the flexion arc is the primary goal. Hinging (instead of articular gliding) at the fracture site must be avoided by eliminating joint subluxation and then re-establish joint stability to prevent recurrent subluxation. Early motion of the PIP and anatomic restoration of the fractured joint surface is a desirable but is secondary compared to reduction of the middle phalanx on the condyles of the proximal phalanx.

     

    Illustration A demonstrates a middle phalanx palmar lip fracture.

     

     

     

     

     

     

  146. A 35-year-old professional football player complains of severe wrist pain after making a tackle. He reports paresthesias in his thumb and index finger. AP and lateral radiographs of the wrist are shown in figures A and B respectively. What is the most appropriate next step in management?

     

     

     

     

     

     

    1. short arm thumb spica cast

    2. long arm thumb spica cast

    3. urgent closed reduction and splinting

    4. MR arthrogram of the wrist to assess ligamentous injuries

    5. bone scan to assess vascularity

    CORRECT ANSWER: 3

    This patient is presenting with a perilunate dislocation with carpal tunnel symptoms. The most important next step in treatment is reduction of the dislocation. This is generally performed in the emergency room, and if unsuccessful, immediate reduction and stabilization in the operating room is indicated.

     

    Kozin et al note that these injuries can be overlooked and have variable propagation patterns through the carpus/carpal ligaments. This patient has a radial styloid fracture due to avulsion of the radiocarpal ligaments.

     

    Melone et al note that these injuries were historically treated with closed reduction and pinning, but more recently the trend is for open reduction and fixation, for optimal anatomic restoration.

     

    Figure A is an AP radiograph that shows obvious scapholunate diastasis due to a perilunate dislocation. Figure B shows the 'empty teacup' sign due to the empty articulation of the distal lunate.

     

     

     

  147. A 32-year-old carpenter has a 6-month history of ulnar-sided wrist pain that is worsened opening a jar, squeezing a wet towel, typing, or changing a gearshift. Radiograph and MRI images are detailed in Figures A through C. All of the following concerning ulnar shortening osteotomy are true EXCEPT:

     

     

     

     

     

     

     

     

    1. Care should be taken to avoid the dorsal sensory branch of the ulnar nerve

    2. Results are encouraging even for those with degenerative changes in the distal radioulnar joint

    3. Placement of the plate to the dorsal surface of the ulna can cause tendinitis of the extensor carpi ulnaris

    4. Concomitant arthroscopy may be indicated for patients with concurrent tears of the triangular fibrocartilage complex

    5. Degenerative cystic changes of the ulnar carpal bones can resolve after the ulnar shortening osteotomy

    CORRECT ANSWER: 2

     

    For patient's with ulnar impaction syndrome, concomitant arthrosis in the distal radioulnar joint (DRUJ) is a contraindication to ulnar shortening osteotomy.

     

    Ulnar impaction syndrome is caused by abutment of the ulnar head into the carpal bones. It is worsened by activities that have wrist rotation and ulnar deviation. A positive ulnar variance with or without cystic changes of the carpus is often seen on plain radiographs. Coexisting central TFCC tears are common and can be addressed by simultaneous arthroscopic or open débridement.

     

    Baek et al. describes 31 patients that had improved Gartland and Werley scores following ulnar shortening osteotomy. They also noted that all patients with degenerative cystic changes had resolution of the cysts at 1-2 year

    followup and they include a detailed outline of their surgical technique.

     

    Chun et al. similarly reviewed 30 wrists of 27 patients with ulnar impaction syndrome with very good outcomes with minimal complications and no ulnar nonunions following ulnar shortening osteotomy. Exclusion criteria included any exisiting arthrosis in the DRUJ

     

    Figure A is a plain radiograph noting ulnar positive variance and mild cystic changes in the lunate. Figures B and C are T1 and T2 MRI images of the wrist noting increased signal in both the lunate and ulnar head.

     

    Incorrect answers:

    Answer 1, 3, 4, and 5 are all important factors to consider when performing ulnar shortening osteotomy.

     

     

     

     

     

     

  148. Which of the following statements is TRUE about force transmission based on wrist position?

    1. neutral wrist position decreases force through the lunate fossa

    2. extended wrist position increases force through the lunate fossa

    3. neutral wrist position increases force through the scaphoid fossa

    4. extended wrist position increases force through the scaphoid fossa

    5. wrist position has no effect on force transmission

    CORRECT ANSWER: 4

    With the wrist extended force transmission is shifted to a greater extent through the scaphoid and the scaphoid fossa.

     

    The position of the wrist can change the pattern of force transmission across the joint. With the wrist in neutral force is transferred across the joint via the lunate fossa and scaphoid fossa almost equally (slight predominance to the scaphoid fossa). However, with the wrist extended force transmission is shifted to pass even more via the scaphoid fossa, and less via the lunate fossa. This is a proposed explanation for scaphoid fractures resulting from falls onto an outstretched hand with the wrist extended.

     

    Majima et al. tested force transmission through the wrist in different positions in a 3 dimensional rigid body spring model from CT images. They found that force through the scaphoid fossa increased from 52% to 62% with extension of the wrist (p<0.05). Similarly, in the midcarpal joint force transmitted across the scaphoid increased from 60% to 69% (p<0.05).

     

    Weber et al. performed a cadaveric study of wrist injury. They found that with the wrist extended, the force applied to the palm was increased 4 times as it transmitted across the scaphoid. They describe the waist of the scaphoid as a location of concentrated force and theorize this is why waist fractures are common in injuries with wrist extended.

     

    Incorrect answers:

    Answers 1, 2, 3, 5: These do not appropriately describe force transmission based on wrist position.

     

     

     

  149. A collegiate baseball player injures his left small finger sliding into third base. He complains of pain and swelling. The finger is ecchymotic, swollen throughout, and painful with attempted range of motion of the PIP joint. No sensory or vascular deficits are present. A radiograph is provided in Figure A. Which of the following interventions will provide the best outcome?

     

     

     

    1. Buddy taping the small finger to the ring finger

    2. Immobilization of the MCP in flexion and the PIP and DIP in extension with a custom splint

    3. External fixation

    4. Open reduction internal fixation

    5. PIP joint arthrodesis CORRECT ANSWER: 4

    The radiograph shows an oblique fracture of the distal proximal phalanx that extends into the joint with an articular step off. Open reduction internal fixation will correct the deformity, expedite finger rehabilitation, and prevent early degenerative arthritis. Closed treatment without fixation will not reliably hold the fracture reduced while the bone heals. Arthrodesis is unnecessary in this young, active patient.

     

     

     

  150. You are seeing a 24-year-old male in the emergency room after he was involved in a knife fight. He has severed the common digital nerve to the index finger on his dominant hand, with an 8mm gap between nerve ends. In counseling him about repair, which of the following options is as good as autologous nerve grafting?

    1. Glycolide trimethylene carbonate conduit

    2. Collagen conduit

    3. Silicone sleeve

    4. Primary end-to-end repair

    5. Polyglycolic acid conduit

    CORRECT ANSWER: 2

     

    Repair of segmental nerve loss in the hand using collagen conduits allows for nutrient exchange and accessibility of neurotrophic factors to the axonal growth zone during regeneration. While the other listed answers have been used, none has shown the efficacy of collagen conduits or autograft.

     

    Li et al. describe the repair of peripheral nerves with a tubular collagen conduit and review supporting data from in vitro and in vivo primate studies to this regard.

     

    Bertleff et al. describe the recovery of sensory nerve function after treatment of traumatic peripheral nerve lesions with a biodegradable poly(DL-lactide-epsilon-caprolactone) neurolac nerve guide, compared to their control of end-to-end repair, no autologous grafting. They show equal results between primary end-to-end repair and their synthetic graft.

     

    Waitayawinyu et al. compared 2 synthetic polyglycolic acid conduits to autogenous nerve grafting using histopathologic and neurophysiologic analyses in a segmental defect rat model. They found that collagen conduits and autografts produced comparable results, which were significantly better than polyglycolic acid conduits.

     

     

     

  151. The parents of a newly adopted 3-year-old boy bring the child to the office for evaluation of his thumb. A clinical photograph is provided in figure A. Which of the following is the most important factor in determining thumb reconstruction versus ablation and pollicization?

     

     

     

    1. Stability of the carpometacarpal joint

    2. Functional ROM of the wrist

    3. Functional ROM of the index, middle, ring, and small fingers

    4. Skin contracture of the first web space

    5. Absence of a thumb interphalangeal joint

    CORRECT ANSWER: 1

    The clinical photograph demonstrates a hypoplastic thumb. The incidence of thumb hypoplasia is 1 in every 100,000 births and associated anomalies including radial aplasia, thrombocytopenia, and renal/cardiovascular/CNS anomalies are frequent. Stability of the carpometacarpal joint is essential for success of thumb reconstruction procedures. If CMC stability is deficient, then ablation and pollicization is preferred.

     

    Light et al describe the evaluation and surgical technique involved in treating the hypoplastic thumb. They note that severe thumb hypoplasia and aplasia are best treated by thumb ablation and pollicization of the index finger.

     

     

     

  152. A 50-year-old woman sustains an open both bone forearm fracture seen in Figure A and undergoes the treatment seen in Figure

    B. During surgery the posterior interosseous nerve was transected and primary repair was attempted. One year following surgery the patient continues to have no posterior interosseous nerve function. Which of the following treatments will best restore function?

     

     

     

    1. Sural nerve grafting to the posterior interosseus nerve

    2. Wrist fusion with transfer of the flexor carpi ulnaris to the finger extensors

    3. Transfer of the flexor carpi radialis to extensor digitorum and the palmaris longus to the extensor pollicis longus

    4. Transfer of the pronator teres to the wrist extensors and the palmaris longus to the finger extensors

    5. Transfer of the flexor carpi ulnaris to the wrist extensors and the palmaris longus to the extensor pollicis longus

    CORRECT ANSWER: 3

     

    Figures A and B show a pre and post-operative radiograph of a both bone forearm fracture. The posterior interosseus nerve is at risk during surgical approaches to this fracture pattern and care should be taken.

     

    Ropars et al retrospectively reviewed 15 patients who underwent treatment for radial nerve and PIN palsy. For PIN palsy, they concluded the most beneficial transfers included transferring the flexor carpi radialis to the finger extensors (to restore finger extension) and palmaris longus to the extensor pollicis longus (to restore extension of the thumb). In contrast with a radial nerve palsy, with a PIN palsy the patient has adequate wrist extension due to intact ECRL (providing radial wrist extension) supplied by the radial nerve proximal to the PIN.

    Ustün et al in their cadaveric studies were able to show that it is possible to perform posterior interosseous nerve neurotization via the median nerve.

     

    Hirachi et al reviewed the results of 17 traumatic PIN palsies that were treated either with nerve repair, tendon transfers, or nonoperatively. They noted that associated muscle damage resulted in poorer results.

     

    The muscles involved in the suggested transfer (FCR, ED, PL, EPL) are shown in illustration A-D.

     

     

     

     

     

     

     

  153. Which of the following skeletal dysplasias is caused by a sex linked mutation of the short stature homeobox (SHOX) gene?

    1. Cleidocranial dysplasia

    2. Leri-Weil dyschondrosteosis

    3. Pseudoachondroplasia

    4. Ellis-van Creveld (EVC) syndrome

    5. Achondroplasia CORRECT ANSWER: 2

    Leri Weill dyschondrosteosis is a skeletal dysplasia characterized by short stature and bilateral Madelung deformities of the wrist. The SHOX gene is located on the X and Y chromosomes and a mutation on either of the sex chromosomes leads to the dysplasia (sex linked dominant).

     

    Madelung deformity is a result of disruption of the volar ulnar physis of the distal radius (leading to radial inclination and a radiopalmar tilt). Symptoms from the wrist deformity include ulnocarpal impaction, restricted forearm rotation, and median nerve compression.

     

    Cleidocranial dyplasia is an autosomal dominant defect in CBFA-1, a transcription factor that activates osteoblast differentiation.

    Pseudoachondroplasia is an autosomal dominant mutation in COMP on chromosome 19. Ellis-van Creveld (EVC) syndrome is an autosomal recessive mutation in the EVC gene causing acromesomelic shortening and postaxial polydactyly among other anomalies. Achondroplasia is an autosomal dominant mutation of FGFR-3.

     

    Illustration A demonstrates a Madelung deformity of the wrist with characteristic increased radial inclination and increased volar tilt.

     

     

     

     

     

  154. A 32-year-old carpenter complains of progressively worsening wrist pain for the last 2 months. He denies any recent history of trauma to the wrist or hand. An MRI is obtained and a representative image is provided in Figure A. Which of the following surgical interventions is thought to be effective for this condition by inciting a local vascular healing response?

 

 

 

 

  1. Wrist fusion

  2. Ulnar shortening osteotomy

  3. Distal radius core decompression

  4. Proximal row carpectomy

  5. Scapholunate ligament reconstruction